You are on page 1of 228

‫‪Correction of first 135pages ,‬‬

‫‪gd luck everyone..‬‬

‫‪Collections of SDLE 2019 for last‬‬


‫‪prodcast‬‬
‫) ‪All blue dots was in exam ( 14 nov 2019‬‬
‫‪+‬‬
‫‪arrow‬‬

‫تصحيح ومراجعة اول ‪ 135‬صفحة‪٫‬‬


‫اجتهادات شخصية من قبل زمالئكم‪ ..‬التنسونا‬
‫من دعواتكم ‪ ,‬وفقنا اهلل وياكم‬
‫‪Emtiaz‬‬
‫‪Text‬‬

‫التصحيح وبعض ا‪-,‬حظات بتكون باللون‬


‫ا‪6‬زرق بجانب السؤال‬

‫‪Good luck …..‬‬

‫تم مراجعة اغلب االسئلة من قبل زمالئكم الى صفحة ‪ ,١٣٠‬ان شاء اهلل مايكون فيه اخطاء ولكن محد معصوم من الخطأ‪.‬‬
‫وفقنا اهلل واياكم ‪6th Nov‬‬
1/ Over protected parents Affect on child :
A/ worse child behavior
A/ make the dentist uncomfort ***

2/ Most important colour :


A/ Avalue *** first one and important one
B/ chroma
C/hue

3/ When add violet stain to color with yellow chroma :


A/ increase value increase chroma
B/ decease value decrease chroma
C/ decrease value increase chroma and if we add white color to yellow it will increase value
D/ increase value decrease chroma

4/ prevent amalgam discoloration :


Put varnish

5/ alginate impression put in water for long time :


Imbibition

6/ impression cab poured after 24 hour :


A/ PVS
B/ reversible hydrocolloid
C/ irreversible hydrocolloid
D/ condensation silicon in 30 minute

7/ which sealer is AH plus :


A/ resin ***
B/ glass inomer
C/ calcium hydroxide
D/ Silica

8/ which sealant is AH 26 :
A/ zinc oxide eugenol free ** contain eooxy resin
B/ glass inomer

9/ which obturation material can be used when the patient allergy to latex or zinc oxide :
A/ Gp
B/ active GP
C/ Resilon ***
B / carrier based

10/ patient with missing #21 Kennedy classification :


A/ class l
B/ class ll
C/ class lll ***
D/ class lV

11/ patient complain of rotation of denture .. which part when it present can prevent this :
A/ direct retainer
B/ indirect retainer *** and not used in class 3
C/ minor connector

12/ 2/3 of the crown of upper 6 fracture and 1/3 remain ,, the tooth respond normally to tests what’s the
management :
A/ elective RCT then post and core then crown ***
B/ composite restoration

13/ definition of veracity { Truthfulness }

14/ definition of autonomy (patient right)

15/ guid plane


A/ parallel to path of insertion *** guide denture in and out
B/ parallel to lond axis of tooth
C/ perpendicular on the occlusal plan

16/ Investmrnt material of metal fused porcelain:


A-gypsum bond for gold
B-silica bond
C-phosphate bond ***

17/ Mechanism of action of chlorahexadine :


( Anti-septic ) destruction of bacteria cell wall

18 / Most support in maxillary denture achieved from :


A/ hard palate *** primary bearing area
B/ rogae area ( Relief )
C/ maxillary tuberosity

19/ pic of swelling ,, and asked about inferior border of submandibular space infection :
A/ platesma muscle *** and superfacial layer of deep facia Digastric muscle
B /sternocleidomastoid muscle

20/ patient with clenching habit and many cavities with weake functional cusp best restoration :
A/ cast inlay
B/ porcelain inlay
C/ composite
D/ amalgam

21/ lateral incisor with fracture of disoincisal angle and glass inomer restoration in mesiolabial angle best
restoration for esthetic :
A/ casted ceramic veneer
B/ etched porcelain veneer
C: composite restoration

22/ cause of orofacial cleft in pregnancy


A/ painkiller (. Save )
B/ antibiotic
C/ vitamin c deficiency
D/ folic acid deficiency ****

23/ best test to determine asmetic patient :


A/ heart volume
B/ pulse rate
C/ inspiration
D/ expiration *** peak expiratory flow (PEF) + spirometry

24/ when you use G.G to enlarge canal , what’s the complication could occur in dangerous zone :
A/ strip perforation *** distal side of mesial root in mandibular molar
B/ zipping
C/ ledge

25/ patient came with pain and discomfort in the tooth after RCT ,, in X-ray there is very wide post ,,
what’s the most common cause of pain :
A/ vertical root fracture

26/ AIDS pt has asymptomatic broken tooth ,, what’s the management :


A/ wear double gloves and face mask
B/ refer to infectious disease center
C/ request CD4 test ,, and blood test .... ***

27/ TB patient with active disease ,, you started treat him what’s the management :
A/ take all precaution

28/ TB patient but it’s not infectious :


A/ treat as normal patient

29/ TB patient under medication ,, what is the least time to treat him after medication
A/ 2 weeks
B/ 3 weeks
C/ 4 weeks
D/ 8 weeks

30/ Fluoride in tooth paste :


1000-1450 Adult
Child 500
Mouthwash 250-900 type of fluoride : sodium fluoride +monofluoride phosphate

31/ when dentist use toflemire matrix ,, injured his finger and there is bleeding under gloves :
A/ put plaster without remove gloves
B/ remove gloves and let bleed freely then wash with water and soap ***

32/ HBV carrier :


A/ HBeAG
B/ HBsAG ***

33/ pic of extracted lower 6 ,, and 7 is menially tilted ,, what will happen :
A/ impaction of food

34/ Caries is endemic disease :


A/ it habitually occur in human population ***
Pandemic: affect large country
Epidemic: excess

35/ FPD success rate in 10 years and 15 years


A/ 92 - 80
B/ 95 - 70 85- 66
C/ 70- 60
20 y ( 69%)
15 y (66)
5 y ( 96%)

36/ least implant success in which type of bone :


A/ type l
B/ type ll the best posterior mandible
C/ type lll
D/ type lV *** posterior maxilla

36/ pt complain of movement of implant under denture and at the beginning there is difficulty in put
denture :
A/ rubber band and none parallel implant ***

37/ most common cause of failure of implant


A/ periodontal disease ****
B/ occlusal Trauma

38/ how can you deferentiate between periapical and periodontal abscess :
A/ vitality test ****

39/ features of ANUG :


A/ fetid odor + bleeding + pain +ulceration

40/ also about feature of ANUG :


A/ pain .. necrosis of interdental papilla ,, bleeding

41/ different between ANUG and primary hermetic gingivostomitis :


A/ systemic involvement **** (Fever, malaise and headache , sore throat and lymphadenopathy)y

42/ pic of crown with open margin mesially and distally most common cause :
A/ over taper preparation
B/ lake of path of seating ****
C/ expansion of core material
D/ shrinkage of metal

43/ pic of swelling in gingiva ( firm swelling ) Histopathology ( multineuear ......) the management :
A/ gingivoblasty
B/ gingivectomy
C/ incisional biopsy *** ? x
D/ excision all
. biopsy

44/ the margin during the metal try is good but during porcelain try in the margin is short what’s the
cause :
A/ over trimmed die
B/ cutting from metal
C/ cast is effected during porcelain application

45/ after chair side bleaching what’s the material can use to decrease the possibility of cervical root
resorption :
A/ put mixture of caoh and water in pulp chamber *** fristlly put 2mm of MTA or zinc polycarboxylate as
protectevie layer
the bleaching material and zinc oxide eugenol against it and after few week remove it the before final
restoration
put caoh to prevent resorption
B/ put mixture of zinc oxide with water
C/ sodium barbiturate + hydrogen peroxide

46/ impacted canine ,, the father refuse to extract canine ,, what’s the most complication occur :
A/ resorption of lateral incisor ***
B/ transportation

47/ To treat class ll malocclusion :


A/ headgear ***
B/ functional appliance if class 2 because mandibular deficiency

48/ patient with class ll with increased overjet


And lower crowding treatment :
A/ extraction of upper 4 and lower 5 ***

49/ camouflage for class lll without crowding :


A/ extraction of lower 4 ***

50/ or considered heavy smoker :


A/ >20 per day ***
B/ >30
C/ >40

51/ intraligament anesthesia :


A/ Cease decrease pulp circulation for 30 min
b/ markedly decreased

52/ patient vomit on Floor :


A/ intermediate level
low concentration ***

53/ pic of internal resorption at the apex of tooth of upper 2 ,, history of ortho treatment before 2 years ,,
what’s the cause of resorption :
A/excessive orthodontic force ***

54/ horizontal line at the Cervical third of labial surface of upper and lower anterior teeth :
A/ abrasion ***
B/ erosion
C/ abfraction V shape

55/ 3 years pt ,, no water fluoridation ,, what is the best fluoride supplement to decrease Caries :
A/ daily brushing with fluoridated toothpaste ***
B/ 8% stanous fluoride twice annually
C/ .....

56/ most common tumor metastasis to gingiva :


A/ colon adenocarcinoma +brest The majority of metastatic cases (70%) reported in the literature have
primary tumors located in the lung, breast, kidney, and colon
A/ father older than 50 ...
B/ the story different from what that find ***
58/ patient with bridge on 456 ,, feeling pain or discomfort with biting ,, in radiograph there is isolated pocket and tear drop
radiolucency in the lateral surface of root : dx
A/ radicular fracture *** vertical root fracture (teardrop appearance )

59/ according to American academy preschool child can drink juice per day :
A/0
B/1-3 for 1-3 years old half cup
C/5-6 **** ounces per day for 4-6 years old half to two third cup
D/ 9/10 for 7-18 years old one cup

60/ Cells in multiple myeloma


A/ Plasma cell ***

61/ Drug decrease saliva during taking impression ,,


A/ anticolenergic *** (atropine)

62/ Drug to incease saliva in dry mouth ,,


A/ pilocarpine ( salagen ) + cevimeline ( evocx) ***

62/ best time of splint for Avulsed tooth


A/ 2 weeks less than 1 h
B/ 4 weeks more than 1h

63/ Most common feature of Concussion :


A/ tenderness to percussion ***
64/ pt came after RCT ,, there is no pain and negative percussion and palpation .. after taking X-ray the radiolucency decrease in size .. the best description
of success of treatment that the radiolucency is :
A/ healed
B/ Healing ***
C/ cured

65/ pt vomit on the floor ,, the vomiting is considered :


A- Hazardous
B- contaminated *** x
C- infectious
.

66/ pt came with persistence bleeding after extraction ,, and still persistent after doing suturing ,, what’s
the material can help in clotting :
A/ Gelfom ***

67/ hemophiliac pt ,, has Remaking root canal tx. ,, the best management :
A/ extraction of the tooth
B/ RCT and reduce under the level of gingiva ***

68/ Etichics pricible


- competence, autonomy ,professionalism ***
- competence, autonomy , accibilty
- competence, professionalism, accibilty

69/ The least time that you can do Impression with retraction cord after crow lengthening /
A/ 7days
B/ 15 days
C/ 21 days ***

70/ The ideal taper of prepared tooth


A/6 ***
B/ 10
C/ 15 OCCLUSAL CONVERGENCE

71/ patient came for routine examination and there is discomfort in upper 4
Clinical examination there is 7 mm pocket in the mesial side ,, radiographically there is deep restoration
7mm below CEJ what’s the management :
A/ extraction and replace with implant
B/ deep scaling and root planing ***
C/ .....
72/ you did RCT for the patient and prescribe ipobrofin 600 for him ,, he came after 2 days with
moderate pain persist with ipobrofin the best management :
A/ replace ipobrofin 600 with acetaminophen 1000mg
B/ Increase the dose of ipobrofin to 1200 mg every 6 hours
C/ replace ipobrofin with Augmentin
D/ alternate ipobrofin with acetaminophen 1000mg ( same of 1st choice ) *** ?

73/ instrument to remove unsupported enamel in gingival floor


A/ GMT. Gingival margin trimmer ***

74/ class ll furcation involvement management


A/ GTR ***
B/ furcationplasty
C/ tunnels preparation CLASS 111
D/ riot resection

75/ pic of pusher and asked what’s used for :


A/ initial placement of band by dentist ***
B/ final placement of band by dentist
C/ initial placement by the patient bite on it
D/ final placement by the patient bite on it

76/ PPE put on :


A/ mask .. eyewear.. gloves ***
B/ gloves .. mask ,, eyewear

77/ factor of crown retention in descending order :


A/ dislodgment force ,, hight ,, parallelism ,, type of cement DESCENDING
B/ type of cement ,, parallelism ,, hight ,, dislodgment force *** ASSENDENING

78/ cell responsible for bone resorption and deposition


A/ osteoclasts
.
B/ osteoblast
C/ fibroblast x

79/ fist thing to check in zirconium crown try in :


A/ fitness
B/ marginal fitness *** , PROXIMAL CONTACT , MARGINAL , OCCLUSION contour esthetic

80/ during try in .. the sequence of check the crown :


A/ Marginal fitness ,, contour ,, proximal contact ,, occlusion ,, esthetic ***
B/ proximal contact ,, marginal fitness ,, occlusion ,, contour ,, esthetic >>>>
C/ occlusion ,, marginal fitness ,, proximal contact ,, contour ,, esthetic

81/ side effect of prolong use of Listrien mouth wash


I forgot the choices but there is no hairy tongue in choices ( Stain ) ***
hairy tongue + stains +desquamative epi + dry mouth
82/ patient with liver failure ,, has multiple Caries ,, and need to decrease Caries :
A/ flouride is contraindicated in this case
B/ flouride ........ not more than 2ppm *** ?
( the remaining choices about flouride dose )

83/ strawberry gingiva :


A/ wegener disease ***

84/ when you do extraction for #14 remaining root ,, you did four angle flap ,, and there is no pus or
infection in the area .. what’s the type of wound :
A/ Clean contented ***
B/ clean
C/ contaminated

85/ X-ray for space infection :


I choose ( CT with contrast ) ***

86/ Test for patient with liver Disease :


A/ prothrombin ***
B/ blood urea nitrogen (Kidney function)

are AST (SGOT) and ALT (SGPT)


87/ perio probe with .5 mm round ball at the tip :
A/ WHO ***
B/ Michigan 3-6-8

88/ UNC15 color coded at :


A/ 5 ,10 , 15 ***
B/ 3 ,6 ,9

89/ Impression record functional movement :


A/ selective pressure ***
B/ open widow technique ( flabby ridge )
C/ without pressure

89/ pt came with sensitivity ,, clinical examination reveal whit incepient Caries on the fissue of Lower 6
best measure to use :
A/ diagnodent ***
B/ bitewing X-ray
C/ electric Caries monitoring

90/ Andrew’s 6th key of normal occlusion :


A/ mesial inclination of all teeth
B/ lingual tilting of lower teeth

C/ no rotation ***

91/ best description of progression of periodontal disease :


A/ Loss of attachment ***

92/ ASA pt with myocardial infarction since 1 year :


A/ type l
B/ type ll
C/ type lll *** if more than 3 month
D/ type lV if less than 3 month

93/ feature of increase VD :


A/ gagging
B/ bruxism
C/ clenching

94/ most thing effect on phonetics in complete denture :


A/ anterior palate *** ? upper anterior teeth mor palataly
B/ small upper teeth
C/ lower teeth too high

95/ fracture with did face mobility .. involve nasal bridge and infraorbital margins :
A/ lefort l ( Maxillary bone)
B/ lefort ll *** ( Infra orbital bone )
C/ lefort lll(Mediyal orbital bone)
D/ lefort lV

96/ cutting end of universal curette :


A/ at the tip of working end
B/ at both side of working end
C/ at one side of working end

97/ Bone loss that can cause secondary occlusal Trauma :


A/ 30-50%
B/ 50-60 %
C/ 70-80 %
98/ Pic of geographic tongue ,, treatment !!!
1. anesthetic and antihistamine mouthwash.
2. oral pain relievers.
3. corticosteroid rinses.
4. vitamin B and zinc supplements

99/ implant screw and fixture fracture ,, how can you remove the implant :
A/ helix driver
B/ torohen bur hexagon driver
C/ Attaching device to the coronal part of the implant and remove it slowly

100/ feature of class l malocclusion :


A/ malaligment of teeth ***

101/ pt after doing bridge on 456 return to dentist to thank him ,, because she has sever headache and
disappeared after getting the bridge : The cause of headache :
A/ migraine
B/ trigemenal neuralgia
C/ muscle spasm and myocardial pain
D/ TMJ disorder ***

102/ brushing teq for pt with ortho :


A/ carter ***
Roll Brushing technique for child fones
Modified stillman For gingiva rescission
Normal patient Modified bass Technique

103/ endo instrument consider hand driven :


A/ GG
B/ lenthelo spiral
C/ barbed proach….. pulp extirpation

104/ best relation when VD is lost :


A/ centric relation ***
B/ maximum intercuspation

105 / 4 and 7 missing in both side ,, 8 also missing , how to mount :


A/ by hand in wax rim be Ouse VD not lost *** first molar still exist
B/ maxilary wax rim ,, transfer by face bow. And mandibular wax rim in centeric relation

106/ RPD without rest ,, what will happen :


A/ impinge gingiva and inflammation (support)
B/ loss of retention ***

107/ lower 7 tilted mesially type of clasp :


A/ ring clasp with distolingal indercut *** ring for lower
B/ circumferential clasp with .... for upper
( l can’t remember the exact choices )

107/ pt with missing 4 5 bride will be :


A/ 4 unit with 3 6 abutment ***
B/ 6 unit with 2 3 6 7 abutment

108/ pt with missing anterior teeth with bone loss :


A/ sectional Removable partial denture because bone loss I prefere RPD
B / fixed partial dengue ***
C/ implant ... A is correct, not B
109/ protect pt from FALL :
A/ autonomy
B/ non malefetience *** do no harm
C/ beneficial

110/ return the patient to the first dentist :


A/ ethical
B/Professionalism ***
111/ action of feeling you have authority on the patient and you know what’s the better for him and not
take his opinion in treatment :
A/ paternalism ***

112/ wrinkles of rubber dam :


A/ holes too far

113/ pt after scaling and root planing returns with pain in right side and has fever ,, after examination
you find deep pocket and swelling
,, best management :
A / repeat scaling and tooth planing
B/ put topical antibiotic
C/ root planing for the area ,, and systemic antibiotic ***

114/ pt with oral ulcer , genital ulcer perianal


Ulcer ,,
pathercy test ..

115/ most common flouride varnish :


A/ 5% sodium flouride ***

116/ not response to test ,, pain with percussion :


A/ necrotic pulp with acute apical periodontitis ***

117/ lingering pain with cold test :


A/ irreversible pulpitis ***

118/ no pain with percussion and no radiolucency :


A/ normal periapical tissue ****

119/ not response to tests ,, with pain and swelling


A/ necrotic pulp with acute periapical abscess ***

120/ not response to test ,, periapical radiolucency ,, with draining fistula :


A/ necrotic pulp with chronic apical abscess ***

121/ radiolucency around both root to endo treated tooth


A/ redo RCT ***

122/ obturaion teq for open apex :


A/ carrier based teq *** ??? MTA
B/ lateral compaction teq
B/ vertical compaction teq ( Warm GP)

123/ time before doing biopsy for ulcer :


A/ 7 days
B/ 14 days *** 2 week
C/ 21 days

124/ endo perio lesion /


A/ perio fist then endo
B/ endo first the perio ***
C/ only peri
D/ only endo

125/ probe for furcation


Naber’s probe ***

126/ correction of interference in protrusive movement :


Remove from distal incline of upper cusp and mesial incline of lower cusp ***

127/ pic and asked about wall defect :


A/ One wall defect
B/ two wall defect
C/ three wall defect
128/ maximum dose of lidocaine 1mg for 20kg child :
A/ 20 for lidocaine 0.2 × 1 = 0.2 × 100= 20
B/ 36
C/ 2.4 carbule

129/ using of small dimeter implant in posterior area ,, what’s the the complication that will occur :
A/ undermining of ceramic crown ***
(Fracture )

130/ what’s the periodontal fiber that pass over the alveolar ridge from cementum of one tooth to the
adjacent tooth :
A/ Transeptal fiber ***
B/

What is Cause hypercementosis : sharbey’s fiber

131/ what’s the complication that will occur when you give local anesthesia with epinephrine for patient
with hyperthyridosim
Cause increased heart rate ( tachycardia) + Irregular heartbeat ( arrhythmia) + High blood pressure level
can cause cardiac arrest .

132/ root planing will be easier when you have :


A/ divergent root with long trunk
B/ divergent root with short trunk ****
C/ convergent root with long trunk
D/ convergent root with short trunk

133/ most common feature of gingiva during pregnancy :


A/ gingival bleeding ***
B/ gingival enlargement

134/ case about diastema between two incisor and the is blenching in gingiva ,, what’s the management
:
A/ Extraction of supernumerary tooth then frenectomy
B/ frenectomy***

135/ Q about frenectomy

136/ difference between Florida probe and conventional probe

Florida is Digital save time high cost less tactile sence underestimation of probing depth
Conventional probe more time less cost

137.How much time does hepatitis b virus remains in a room with normal temperature :
few minutes
2 hours
week****
months
138. dentine that looks more radiopaque on the radio in a restaured tooth from a long time
ago :
sclerotic dentin**** hard shiny
secondary dentin ,
tertiary dentin,
reparative dentin

139. how to evaluate that a periradicular periodontitis is healed?


a. lesion is disappearing
b. no more lesion
c. no more symptoms****
d. no more sinus trac

140.Nitrous oxide affect which vitamin?

B12

141.Type of cementum in coronal 2/3rd


• Acellular afibrillar (cervical)
• Acellular extrinsic fiber****
• Cellular mixed stratified

142. What is the cell affected in diabetes :


a. macropage .
b. neutrophils****

143. Which cells in established gingivitis :

lymphocytes,
leucocytes ,
plasma cells*
macrophages

143. CHILD COME FOR DENTAL TTT, YOU C MULTIPLE SKIN RASHES ON HIS
BODY,WHAT U WILL DO:
.START NORMAL
.START WITH MAXIMUM PRECAUTIONS
.REFERE TO MEDICAL CENTER TO TREAT RASHES 1ST****
.DO NO WORK FOR HIM

144. FRACTURE AT CONDYLE..WHICH NERVE WILL BE AFFECTED OR


INJURED

Auriculotemporal

145. A little girl 5 years going to have chemootherapy and have deep caries on first primary molars without image on the
furation area what to do :
Extraction*
pulpotomy,
pulpectomy,
rct

146. PT EXTRACT ALL TEETH & WANT TO MAKE NEW DENTURE(1ST TIME) , IT IS NOTICED THAT HIS LOWER LIP IS
TOO TIGHT TO THE RIDGE.

WHAT PROBLEM THIS WILL


MAKE?
.PUSHING DENTURE
. UPWARD pushing denture upward

x .PUSHING DENTURE BACKWARD


.LOSS OF ANTERIOR FLANGES RETENTION****

147. A women with all ceramic anterior upper fpd , what happens to anterior lower
teeth:
abrasion****
attrition,
erosion,
abfraction

148. Patient having an orthodontic treatment and came back with bleeding when
brushing in a specific area :
periodontitis
, gingivitis *
gingival hypergrowth

149. Patient with upper denture not retentive , when checking every thing is good but
there is bubbles in the post dam area what to do:

remake ,
rebase
. ,
x reline ****
, something obout repairing the post dam area

150. FRACTURED UPPER CENTRAL, ONLY 1MM REMAIN


SUPRAGINGIVAL,PRPER TTT:
.SET POST & CROWN
.GINGIVECTOMY
CROWN LENGTHENING THEN POST & CROWN****

151. Child with low caries index previous check up there were no caries index with clean bitewings .. He came
for follow up .. What is best to be done :
a.2 bitewings
b.2 bitewing and 2 occlusal films
c.2 bitewings and panoramic veiw
d .No need for x ray film****

152.Child with chiken pox .. His doctor said this will affect the calcification of his
developing teeth .. In which stage this may happen :
a.Morphodifferentiation
b.Histodifferentiation ****
c.Initiation
d.Proliferation

153. Bacteries responsible for aggressive periodontitis

ActinoA

154. Patient 10 years having teeth (incisors and first molars) discoloration due to
tetracycline , at any age did he take the tetracycline :
1 ****
4,
6
,9

155. the bacteria communicate with each other by :


a.Courum sensingI****
b.courum signaling
c.courum transfer

156. .Depth of rest seat preparation from the surface of tooth and maximum
intercuspation interface is :
a.1 mm
b.1.5 mm****
c.2 mm

157. .Lady 22 years with high caries index .. Many stained pit and fissure in maxillary
and mandibular molars ..

U suspect careis in this fissures .. What is the best way to diagnose :


a.Exploratory preparation of pits and cissure*
b.Leave them and follow up after 6 months
c.Do conservative composite restorations
d.Seal the pits and fissures

158. Pleomorphic adenoma its size is 1.5 x1.5 cm on posterior part of hard palat, what
is ttt :
a.Enucleation only
b.Radiotherapy
c.Chemotherapy and enucleation
d.Resection of periostuim.****

159.Lesion on x ray surrounding the lower third molar which is unerrupted . patient
came complaining from facial assemetry with slight pain with tenderness on external
part of angle of mandble . lesion is :
a.dentegerous cyst****
b.ameloblastoma
c.Radicular cyst
d.Acute apical periodontitis
160. Best material for inter occlusal recording:
a.Additional silicon*
b.ZnO/E
c.Wax
d.Plaster of paris

161. Resin bonded bridges is commonly used with cases of :


a.Cares
b.Discoloration
c.Teeth fractures
d.Mobile teeth****

162. how much fluor in mouth rinse

163. Which anesthetic causes myosis

Cocaine*

164. Bur used for making cavity in ceramometal full coverage crown

Transmetal bur *

165. .porclain fracture is due to :


a.High ductility of poclain
b.Low compressive strength
c.Low tensile strength
. and ductility
x d.Low tensile strength and crack proppagation.****

166. Lingual frenum is removed in which situation :


a,If limiting tounge movement****
b.If toungue partieally protruded outside mouth
c.If toungue fully protruded
167. .Proximal non cavitated discoloration in low risk caries :
a,Reminiralization
b.Class II composit reztoration
c.Amalgam
d.No ttt****

168. lesion with x ray .. The lesion is unilocular causing shifting of teeth with no root
resorption ..

Patent's last vist to dentist was before 12 years .. His father has the same lesion before
and it was removed for

him many times before .. Lesion is :


a.Keratocyst ****
b.Ameloblastoma

169. External mandibular nerve block the movement is :


upward posteriorly****
downward posteriorly
upward anteriorly

170. Most common cause of FPD failure is :


a.Caries****
b.Periodontal distruction

171. Which of these factors not interfer with wound healing :


a.Diabetes
b.Antibiotics ****
c.Malnourichment
d.Long steroid medication

172. Function of primer is


a.Increase surface energy****
b.Bonding with composite

173. Careis in old patient compared to younge patient :


a.Progress slowly in adult ****
b.Adult suffers less pain compaired to young
c.More progreesing in old patient than young

174. Study on students in 5th grade compared with students of 5th grade before 10
years . name of study :
a.Case control study
b.Cohort study****
c.Cross-sectional study

175. 12 years child .. His mother brought him to ER because of fever for 2 days with
loss of appetite and sever painful mouth and difficulty of swallowing associated with
red ulcers and red fiery gingiva .. The cause is :
a.Herps simplex ****
b.Coxsackie virus
c.Herps zoster
d.Varicela zoste

176. epidermolysis bullosa on skin .. What is the cause :


a.hypophosphatasia
b.Amelogenesis imperfecta****
c,osteiogenesis imperfecta
d.Dentenognesis imperfecta

177. Anaphlactic shock in child after anestheisia give him :


a.IV epinephrin 0,01 mg/kg
b.IM epinephrin 0,01 mg/kg****
c.IV epinipherin 0,1 mg /kg
d.IM epinipherin 0,1 mg/kg

178. Concept of protaper system?


Step back
Crown down****

179. patient was referred to the othodontist to make a space for implant , what's the length
needed

: 5 mm,

7 mm,****

9mm

, 11 mm

180 PATH WAY OF EXTERNAL CAROTID ARTERY?

.AT THE UPPER BORDER OF THYROID CARTRIDGE

181. PT EXTRACT ALL TEETH & WANT TO MAKE NEW DENTURE(1ST TIME) , IT IS NOTICED
THAT HIS LOWER LIP IS TOO TIGHT TO THE RIDGE.WHAT PROBLEM THIS WILL MAKE?
.PUSHING DENTURE
. UPWARD
x.PUSHING DENTURE BACKWARD*

182. old patient received denture 2 months ago presents with severe peri-auricular pain,

severe wrinkles around corner of mouth. Pronunciation problems with some words.
Mastication problem,

reduced facial muscle tone. What is the problem?


.
. Low vertical dimension****
. High vertical dimension

183. A little girl 5 years going to have chemootherapy and have deep caries on first primary
molars without image on the furation area what to do :

extraction,*
pulpotomy,
pulpectomy,
rct

184. Lesion on x ray surrounding the lower third molar which is unerrupted . patient came
complaining from facial assemetry with slight pain with tenderness on external part of angle
of mandble . lesion is :
a.dentegerous cyst****
b.ameloblastoma
c.Radicular cyst
d.Acute apical periodontitis

185. time for hand wash:


a-10-15 sec
b-25-30 sec alchol
c-40-50 sec
d-50-60 sec****
. soap

186. Glass fiber post compared to custom made post :

A) Aesthetics
B) Tooth conservation
C) Less chances to fracture****

187. Fiber post advantage compared to metal post


a) Less chance of root fracture
b) More conservative
c) Esthetic
d) All of the above ****
e) B+C

188. GUTTA PERCHA SHOUL END AT :

.RADIOGRAPHIC APEX .

INTERNAL OPEN APEX****

EXTERNAL OPEN APEX

189. Loss of sensation and taste in right lateral side of the tongue :
1. Glossopharyngeal
2.lingual *
3.facial
4.vagus

190. which the salivary gland have this histological feature: pyramidal cells with
flattened neuclai:
Mucous**** in sublingual gland
Serrous parotid
Ductal
191how many times we should change tooth brush according to american dental
association:
1.three to four month****
2.five to six.

192. Sloughing tissue after anesthesia:


Hard palate*
Buccal mucosa
Floor of the mouth

192. Hypertensive patient use 75 mg aspirin he need to extract lower 6


1- stop aspirin and take heparin .
2- extract if INR
. 3.5 *
3- tell him to came after 3 days

193. Pleomorphic adenoma its size is 1.5 x1.5 cm on posterior part of hard palat, what
is ttt :
a.Enucleation only
b.Radiotherapy
c.Chemotherapy and enucleation
d.Resection of periostuim.****

194. according to amrican dental Association myocardial friction under what class
exactly:
1.i
2.ii
3.iv****
4.v more 3 month

195. PT COMPLAIN FROM FACIAL ESTHETIC PROBLEM (PROTRUSION OF


THE FACE ),ON CHECK UP THERE IS INCREASE OVERJET BUT CLASS 1
INCISORS.DIAGNOSIS :
.BIMAXILLARY PROTRUSION****
.MAXILLARY PROTRUSION
.MAND PROTRUSION
.MAND RETRUSION

196. The particles in enamel which give its shiny appearance

Hydroxy apetite particles

197. why we make glazing for glass inomer cement

To prevent dehydration

198. BRANCHE OF LINGUAL NERVE?


.FACIAL
.MANDIBULAR
.GLOSSOPHARYNGEAL
.TRIGEMINAL
199. what zone of the pulp does the vasodilation happen?

pulp core

200. Note: Z-plasty are effective for narrow frenum attachments. Vestibuloplasty is
often indicated for

frenum attachments with a wide base.

201.

.202- Which transformation of oral mucosa should be taken seriously


-dysplasia*
-Metaplasia
-hyperplasia
-hypertrophy

203-To prevent gingival injury place the margin of the retainer:


A. At the level of gingival crest.
B. Above gingival crest. *
C. Apical to gingival crest 1 mm.
d. Apical to gingival crest 0.5 mm.

204-Angulations of the blade of scaler to stone during


sharpening :
A. 50-60°
B. 70-80
C. 100-110*
.
.
205- blade activation angle:
a-45-90
b-70-80*
c-90-110
.
.
206- premolar with class 2 amalgam and recurrent carries causing sharp cold pain but
not lingering:
a-Remove caries and amalgam then place final restoration*
b-Start one visit endo
c-Analgesic & antibiotic

207 -Patient come with mild pain on biting in his lower right seven, treated RCT
before 3
months
On bitewing: caries under restoration -on periapical: radiolucent on periapical
area Your pulpal and perio diagnosis?
A. Previous treated with acute apical abscess
B. Previous treated with chronic apical abscess
C. Previous treated with chronic apical periodontitis
D. Previous treated with acute apical periodontitis*

208-Tooth surface after continous chewing of tobacco, hard food and vigorous
brushing

Attrition
Abrasion*

209-Chronic renal failure what is developed.


1.Hyperparathyroidism.*
2.Hyperthyroidism

210- Patient with renal transplant, lesion non-scapple,shaggy,fried:


a) hyperplastic candida
b)hairy leukoplakia*
c-idiopathic leukoplakia
d- lichen planus

211- evidence based dentistry is done by:


a. Criteria appraise*
b. Information corrected
c. Literature reading
d. Understanding statestics

212- Under amalgam true or false


- Caoh GIC varnish true
not sure
213- Preparation for FPD
On gingival crest
Above*
Below not sure
214-Rubber dam holes for primary incisor

Small

215. Anticariogenic food


High mineral content*
High protein

216. Radiograph of well defined RO lesion near to the apex (but not attached to) of the
lower 1st molar, the question was what is the lesion:
• osteoma*
• odontoma

217. Wax pattern carving grooves?


Pkt2
Pkt4

Pk3

PKT1: is used for positioning of functional and non functional cusps. The marginal, cusp and
triangular ridges are also added with PKT No. 1. PKT2: is used for eliminating voids remaining
on the occlusal surface. PKT3: Developmental and supplemental grooves are smoothened
PKT4: Smoothening of axial surfaces is done PKT5 is used to refine the ridges

218. Which muscle depress mandible?


A medial pterygoid
B lateral pterygoid*
.
C digastric

219. Old patient severe resorption of lower ridge and sublingual glands are prominent wants
to make complete denture which type of impression best
Muco compressive
Active impression
Dynamic impression*

220. TTT OF PATIENTS HAS AMELOGENESIS IMPERFECTA ?


.FILLINGS
.CROWNS*
.NO TTT

221. What is the recommended amount of flouride per liter?

1-1.5

222. MOUNTING CASTS FOR PATIENT HAS EXCTRACTED 14,17,24,27,34,37,44,47


.WITH WAX BITE REGISTRATION
.WITH ADDITIONAL SILICON REGISTRATION*

223. 62 YEARS OLD MAN NEED MULTIPLE EXTRACTI0N


.EXTRACTION WITH INTERRUPTED SUTURES
.EXTENSIVE ALVEOLOPLASTY IN ALL CASES*
.EXTRACTION & HEAL BY 2RY INTENTION
224. CONDYLAR FRACTURE XRAY

Reverse Town

225-Osteodystrophy in which disease


1-renal
2-liver
3-pulmonary
4-cardiac

226-Anti ssa autoantibody in which disease:


1-sjogrens syndrome
2-lupus A&B
3-pemphigus

227-Patient with Disseminated intravascular coagulation (DIC) pain killer >


1-acetaminophen
2- ibuprofen
3-vitamen k
4-heparin

228-RC prep compostistion :


1-EDTA , urea peroxide
2-EDTA, hydrogen peroxide
3-EDTA, carbamide peroxide
4-EDTA, sodium hypochlorite

229-O’leary plaque index:


1-disclosing agent
2-prophy paste
3-periodontal probe
4-sharp explorer

230Pain during protrusion which of the following is effected:


1-Medial pterygoid x
2-Inferior lateral
. pterigoid ?
3-Stylomastoid ligament

231-Before bisphosphonate treatment


Teeth indicated for extraction is :
1-sharp tooth
2- grade II mobility
3- purulent among probing
4- probing depth 5 or more

232-Dose of ibuprofen in children less than 12


1- 2-4 mg/kg q4h
2- 2-4 mg/kg q6h
3- 6-10 mg/kg q4h
4- 6-10 mg/kg q6h

233-Disadvantage of florida probe


1- overestimation of deep pockets
2- underestimation of deep pockets
3- improve tactile sensation

234-Minimum voxel in endodotic radiograoh :


1- 0.2 0.08-0.13 . and maximum 0.4
2- 0.5
3- 1.3

235-Minimum distance between implant and inferior alveolar canal


1- 1mm
2- 2mm
3- 3mm
4- 4mm

236 -Teeth with generalized grey-ish discoloration decided to do full veneers


in all anterior what’s the material:
-ceramic
-alumina

237- Patient with upper complete denture and lower partially edentulous with
only #35-45 remaining
While reading the PA when can u decide there is trauma from occlusion:
-bone loss 40%
-bone loss 50-80%
-bone loss 90%

238- brown periochip picture Is it


-metronidazole
-chlorohexidine
-(two more antibiotics)

239-a Patient came complaining of diffused pain cold test negative pain on
percussion with swelling and he doesn’t have the time to finish the whole
treatment:
-antibiotic and come back when the swelling subsides
x -incision and drainage
-incision and drainage with
. cleaning and shaping
-cleaning and shaping, then leave the tooth open.

240-Patient did an anterior crown on #21 and came to the dental clinic
complaining that the Incisal edge is opaque
What’s the mistake that the dentist did:
-did not prepare the tooth in two plains.
-wrong opaque shade
-wrong cement shade

241- electronic perio prob:


-Williams
-unc
-florida

242-(picture) patient undergoing ortho treatment came complaining on sever


gingival overgrowth (Lower gingiva covering the bracket) what would your
treatment be?
-gingivectomy so the patient would have access to cleaning by laser
-stop ortho treatment

243- Patient did a crown 2 weeks ago and came complaining of redness
around the crown and bleeding on probing happens around this tooth only
To prevent this problem you should:
-make the crown over contoured on the gingival side
-make is less (minimal) on the gingival side
-make it exactly like the tooth structure

244-what’s the name of the endo bur safe ended:


Endo Z

245- a pregnant lady in her first trimester came with sever pain and swelling
what antibiotic would you give her?
-(penicillin, amoxicillin and clindamycin where not in the choices)

246- cephalometric picture asking about a point:


(PNS)
247-Pedo Patient had a trauma two days ago and the X-ray shows the
primary intruded and close to the permanent bud
-extract
-Leave it
-splint

248- a lot of questions about (AUNG + NUG)

249 -a sign that an apprehensive patient is having a panic attack:


-bradypnea
-Tachycardia

250-side effects of retraction cord with epinephrine: 0.1-8%


-irritation while contacting the tissue
-gingival vasoconstriction
-systemic vasoconstriction

251- a tooth with osteoclasts on the mesial side and osteoblasts on the distal
side is it:
-ortho distalization
-ortho extrusion
-physiologinacl mesial (I think drift)

252- a Patient asked for an amalgam restoration, she came back complaining
of pain while drinking hot drinks;
-thermal expansion
-thermal insulation

253- sterilizing an instrument and leaving it are different from an autoclave to


another, in autoclave B how many days would you leave the instrument:
-21 days
30 days

253- what color is a k-file size 90:


White

254- Patient came into the clinic complaining of gingival overgrowth and
taking phenytoin:
-stop the medication
-refer to a physician ????

256- Patient with HIV came with a crack:


-double gloves mask and goggles
-ask for the DC4 count.

257-Medications that would act in reducing the saliva for a good impression:
-Anticholinergic (atropine)
-cholinergic
-beta blockers
-calcium channels blocker.

258- a missing tooth that needs to be replaced, the space from the crest to
the maxillary sinus is 12 mm. What’s the length of your implant?
-11
-10
-8

259-amount of Fluoride in water that would cause fluorosis:


1
2
5 ???
7

260-Amount of fluoride in over the counter:


-225
‫ثم اكبر ثم اكبر‬ ‫؟؟؟؟‬

261- least esthetic clasp:


-ibar more
-Aker
-ring least

262- metal try in for a ceramometal crown came with a high occlusion and
you’ll remove the interference. How would you measure the thickness of the
metal so you wouldn’t cause a perforation?
Iwanson caliper

263-a crown of a tooth got fractured, how would u regain the ferrule effect:
-orthodontic extrusion
-crown lengthening

264-why do we write down the chief complain?


-the dentist doesn’t forget
-the patients feels. like ‫عطينا المه اهميه‬
x -to compare it with another problems
265- the most common allergic to the dental care?
A-glass ionomer
B-SSC
.
x C-Antibiotics if there is no local anaethesia
D...

266-drugs use decrease saliva when take impression?


anticholinergic

267-Maximum Age to do orthodontic treatment ?


1-20
2-30
3-50
4-no limit

268- pt steak injury from needle . Pt is medically fit ?


1-wash the hand by water?
2-do nothing
3-go to disease centre. take blood samples ?
4-...

269-after extraction teeth with amalgam your put in any waste ?


fixer of radial or recycline waste ‫اتوقع هازارد‬

270-drugs cause gingival enlargement?


Nefedipine
Phenetoine
.
cyclosporine

.......
271- A case about implant for 46 and supra erupted 16 what to do?
*orthodontic intrusion ✅
*endo and crwon for 16
*remove the bone and put the implant more downward

272- Slow way if using palatal expansion appliance


Open 1 ml every other day
0.5 ml every other day
1 ml every week ✅ ????
1.5 ml every other day
273- Smoking in pregnancy may cause?
cleft lip and palate ✅ in baby

274- Cause A pain after two days of placing ceramic onlay on biting ?
x Sensetivty to the lutent cement✅
Hyperocclusion if dirctly
.
Marginal leakage

275- Fracture with movable maxilla and bridge of the nose with inferior border of
orbits??
Le fort 2 ✅
Le fort 1
Le fort 3
Zygomatic fracture

276- What's the varnish under amalgam?


Copalite ✅

277- Case about 12 with facial glass ionomer and distoincisal fracture with minimal
remaking sound tooth structure and the pt needs a long lasting solution??
Full ceramic crwon✅
Composite with pins

278- Badly interensic stained teeth and need to cover it with veneers which type?
Porcelain etched i think its A
Alumina
Casted ceramic

279- A picture of plastic currette whats the uses?


Implants scaling ✅
Sensetive roots
( For not scratching or roughing the implant surface )

280- Remove bone and put implant more apical

290- Best liner under composite Close to pulp?


Caoh✅
Zinc oxide
Zinc phosphate

291- Pt class 2 division 2 how the anterior guidance will be?


Flat
Steep ✅

292- Base under composite for deep class 5?,


Zinc oxide
Glass ionomer✅
Caoh

293- Which branch of cranial nerve supply the mandible?


1
2
5✅
7
8
9

294- Matrix band should be over margin by


1mm
2mm✅
3mm

295- Which band best for composite class 3?


Mylar✅ strip
Tofflemier with photoetched margin
Tofflemier metal matrix

296- Which matrix best for MOD amalgam?


Tofflemier metal matrex with metal band✅
Tofflemier with ultra thin band
Contoured matrix band

297- How to give Tranexamic acid before surgery?


Subcutaneous
IM✅ for hemophilic patient (antifibrinolytics)
Rinse after extraction
Gel after extraction IV or mouthwash

298- Fixed bridge with rocking what to do?


1.Remake✅
2.Cut the defected abutment and fix it then reattach
3.cement it with force

299- Bridge in try in with tiny publes when seated from one abutment why?
1.Passive fit while placing
2.increased lutent cement space✅

300- In primary impression when u remake it?


#If its displaced and moved during setting✅
# if the metal tray show through the impression
#when there is voids that be managed and corrected

301- Walls of composite cavity should be


*acute
*flat
*parallel
*well rounded✅
*convergent
*divergent

302- Soft tissue inflammation around the implant


periimplantitis
Peri implant mucositis✅

303- A picture of bone resorption around implant what is the case?


I choose periimplantitis✅

304- Weired hair cut?


unprofessional✅

305- Finger like projection in lateral border of the tongue which is painless and not
changed in size?
Filliform papillae
Folliate papillae if no increase in size
Papilloma if there is incraese in size

306- Causative of herpangina?


Coxackie virus✅
Ebstien Barr virus ????
Herpes simplix virus

307- What to give to patient with infective endocarditis?


2g amoxicillen before surgery with 1 hr✅
308- Amalgam pin length
2mm ✅
3mm
1mm

309- DIC Pain killer?


Acetaminophen ✅

310- What will response to aspirin?


Osteblastoma
Fibrous osteoma
Osteoid Osteoma✅

311- Pic of a palate with yellowish and white spots and says he was taking steroids
and then the lesion became white and painful on removing the Slough, diagnosis?
Candidiasis✅

312- Many questions about indirect retainers and partial denture designs and what
major connector to choose

313- Question about the implant cover screw and analogue parts function and names
314- A question about impression for esthetic area for an implant
after healing with healing abutment
I choose pick up impression, there was other answers that I don't
remember

315- Many questions about pulpits, periodontitis diagnosis

316-Antes lawde finision:


"the total periodontal membrane area of the abutment teeth must
equal or exceed that of the teeth to be replaced." ✅

317- Misiing 14,15 what are abutments for the FBD?


13,12,16
13,16 ONLY
13,12,16,17

318- Partial denture used for 3 years now when apply force on the
rest an apical displacement of the partial occur whts the reason?
Needs relining
Need more rigid clasp
Remove the rest
Not sure about what was other options

319- Most common defects of phonetics in denture?


Teeth are set away and higher than the lips

320- PP picture of brushing and asking what's the angle of the


brush?
0 degree
90 degree
45 degrees✅

321- Treatment of anterior open bite?


Fixed appliance with plastic postrior bite block

322- Camouflage of cl3 with no crowding?


Extraction of lower 4s

323- Headgear force?


Intermittent

324- Shade of composite was too light why?


Shade selection was with rubber dam on
325- Crwon is too dark why?
Chroma ✅
Hue
Value
Intensity
326- Classes of fremitus

327- Picture of a case with Miller class 2, abrasions ulcers in the


gingival and the patient used medium tooth brush?

*Ask pt to use soft brush and throw away the medium

*Keep the brush but change the technique from horizontal to


vertical✅

328- How to clean a FBD?


SUPER FLOSS✅
FLOSS
BRUSH

329- Pain in first three minutes of bleaching why?


Gel is burning gingival from✅
Hypersensitivity

330- What should be placed after internal bleaching to stop the


internal resorption?
Zinc oxide eugenol
non oxide eugenol
Caoh
Steroids
(Packing calcium hydroxide paste in the pulp
chamber for a few weeks before placement of
the final restoration to counteract acidity caused
by bleaching agents and to prevent resorption.
331- What should we avoid during gates glidden and files
preparation in coronal part (can't remember the exact question) ?
Stripping perforation ? Question not clear
Zipping perforatin

332- How to avoid ledge?


Precurve the file✅

333- Where should we stop the obturation


?
Radiographic apex
Anatomic apex narrowest part
minor foramina
334- Where is the comment place for denture caused hyperplasia?
Palate anteriorly✅
Lower ridge
Tuberoaity

335-The best advantage of coolant?


To reduce heat generation ✅
To clean cavity from depris ( in perio )
Better vision

336- Over protected parents effect on child ?


Make Dr uncomfortable
Worsens child behavior

337- Question about A doctor who decide a medical option without


even asking the patient or referee to him?
Medical decision
Paternalism✅
Best interest

338- Question about autonomy and consent

339- What's least important of consent?


Costs

340- A picture of through and through class (pifurcation grade 3)

341- Question about CODE 1? ???


RESTORATION
REMINERALIZATION
FLOURIDE
EXTRACTION
Fissure sealants
342- Question about A patient on bisphosphonate therapy and has
osteoporosis
Came after extraction with three months with a bare bone without
purulunt exudate or pain what's ur management?
Chlorohyxidine Mouth wash and follow up ✅
Jaw resection
Antibiotic
Vitamin D
343- which endo file system do more reciprocation motion than continuous rotation

profile -

BioRace -

** Wave one -

344-when you have cervical perforation at the beginning of the procedure , when will .
? seal it

If large perforation do it after obturation

If small perforation do it before obturation

immediately after cleaning and shaping but before obturation -

After obturation -

Immediately before any thing -

345-Pt came to you convince he has TB disease. after taking sample from the sputum , .
? )which stain will use it to see the bacteria ( waxy cells

violate crystal -

Acidic -

Endospore -

This stain is used to identify Mycobacterium tuberculosis, the causative


agent oftuberculosis. Acid-fast organisms have a lipoid capsule that has a
high molecular weight and is waxy at room temperature.

346- Which polishing paste will use it to polish GIC restoration


zinc oxide paste -

Pumice -

Calcium carbonate -

Zirconium oxide -

347-What are the ethic principles ? ( it is sth like this , i didn’t remember exactly

competence, autonomy,professionalism -

competence,autonomy,accibilty -

. competence, professionalism, accibilty -

348-When there is new brand of implant , first it should be tested on animal to be sure it
? is biocompatible , according to FDA ethic code this is match which one

B1 -

A3 -

I forgot the choices - D2

349-Pt came to you complaining of too dark newly placed single anterior crown btw
? natural adjacent teeth , where is the problem

hue -

Chroma -

Value -

350-Alot of scenarios about wear facet on natural teeth or fractured or chipped cusp of .
?PFM crown & asking what happened

primary trauma from occlusion and in PFM it is due to high point -

351-Pt with esthetic chief complain , he has pit stains and white spot ( i forgot if it is
?generalized or not ) what is the cause

hypo calcification-

enamel dysplasia-

amelogenesis imperfecta-

352-What is material which will


irritate the pulp if used to fabricate temporary crown on direct tech

Dimethacrylate-

bisacrylate-

polyethylen methacrylate-

polymethyle methacrylate-

353-Pt has active TB & he is on medication , when can you book apointment for him

after 2 week on medications-

3-

4-

6-

354-Recommended amount of fluoride in water fluoridation

ppm 1.0 -

1.0-
1ppm
2-

)ppm not mentioned 0 ( -

355-Picture of coronal view of skull CT scan , from the pic you will see fracture in orbital
? floor & zygomatic process , and asking the most common complication

diplopia-

excessive tearing-

laceration-

356-Antibiotic of choice for pregnant women

clindamycin-

i forgot the others -

Here's a sampling of antibiotics generally considered safe during pregnancy:


Penicillins, including amoxicillin, ampicillin.
Cephalosporins, including cefaclor, cephalexin.
Erythromycin.
Clindamycin.
353-Micanazole antifungal target point on fungi is

mRNA-

cell wall-

DNA-

i forgot -

353- Which cells increase in multiple myloma

plasma cell-

erythrocyte-

B cell-

359-Pt feel sharp nerve like pain when he swallow or rotate his head , the pain in pharynx
?base of tongue and neck ... what is it

**eagel’s syndrome-

trigeminal neuralgia-

..glossophyrengeal neur-

...vagus neur-

360- What type of collagen found in young in formation pulp

I-

II-

III-

XII-

361-Pt have full ceramic crown since 6 months , after clinical examination there is crack on
?the crown in the facial side , which cement have been used

resin-

polycarboxylate-

zinc phosphate-

RMGIC-
362-When placing implant and you want to have interdental papilla regrowth , you
?should leave btw base of proximal plate & crestal bone

4.4-

6.4-

7-

i forgot -

363- When you plan to place ridge lap pontic to replace #21 , you should consider

amount of ridge resorbed-

lip line level-

pt acceptance-

364-Long scenario and then said you have deep caries& want to place varnish under the
?amalgam , what is the type

copalite-

caoh2-

zinc oxide-

GIC-

365-Gingival metastasis lesion

It happen with lung adenocarcinoma

366-Pic of tooth with good crestal bone level(normal to mild) and there is RL lesion in
furcation area , in clinical examination there is 8mm pocket , what is the type of bone
?loss

vertical-

horizontal-

interradicular-

interdental-
367-Scenario of pt came with severe pain in the palate / after examination there is
bleeding & well dimensions of lesion extend from molar to 1st premolar with denuded
?palate &there is graft in lower anterior area , what is the type of graft

free gingival-

connective tissue-

transpositional- i think its B

-pedicle

368-You have autism pt with mental retardation came with nurse and you want to take
? consent before starting the tx , from whom will take it

pt -

parents-

nurse-

no need-

363-Which type of mandibular fracture most common associated with lip numbness

symphesial-

parasymphesial-

body-

condylar-

370-Most common abnormal finding after doing pulpotomy with ferric sulfate in primary
? tooth

external resorption-

...internal re-

calcification-

371-Pt came to checkup, #44 extracted ,#45 has mesial arrested caries with RO lesion
inside the canal in the middle part & surrounded by RL line you test the tooth & every
?thing normal ...what is the most clinical significance of this case

early pulp degeneration- Not clear

will make endo tx difficult-

... -
372- type of quadhelix appliance force used in expansion

skeletal-

dental-

dental, 1/2 skeletal 2/0-

skeletal, 1/3 dental 3/2-

333- Lab investigation for HBV

Read about which antibody will see in -acute stage

chronic - recovery -
immunized

374-y/o pt has class II malocclusion &lower anterior crowding , which teeth will extrat 44

upper 1st premolar-

lower 1st premolar-

upper & lower 1st premolar-

upper 2nd ,lower 1st-

)upper 1st ,lower 2nd not mentioned (

375-y/o pt has skeletal class III & 6mm reverse bite , pt main concern just the esthetic 32
? appearance

camouflage-

orthognathic surgery-

extraction-

376-Headgear force is

heavy-

intermittent-

interrupted-
377-teeth you want to make space btw them by ortho tx , what will happen to the 2
? inderdental papilla

underneath bone will resorbed then the papilla will become flat-

remain but become fibrotic-

inverted-

378-In which situation in ortho tx will have mesial osteoclast active& distal osteoblast
active

intrusion-

extrusion-

iforgot but did not mention mesialization or tipping -

379- How will hold the gracy curette when use it

the handle // to tooth long axis-

the lower shank // to long axis of tooth-

same B but // to surface to be scaled -

same b but angled toward the tooth surface-

380-Scenario about RPD pt have attrition& the hight of contour is more coronaly toward
the occlusal surface , in insertion session while placing the denture the aker clasp broke ,
? what could be the cause ??????

cyclic fatigue to the clasp-

”flasking interfence”i think-

. the metal-
weakness in

force more the distortion limit of metal alloy-

381-Pt has class II div 2 , doctor place ceramic bracket with coated wire , after a while pt
?came with upper anteriors abfraction what is the cause

ceramic bracket-

coated wire-

...-
332-in orthodontic wits appraisal show you.

??????
FH plane-

anterior post..occlusal plane-(disharmony )

....-

333-Q about Tx of unilateral cross bite i think in young pt , what is the tx Not clear

Hawley expander-

dental braces &....-

only dental braces-

functional .appliance-

-334To adjust condylar inclination& lateral shift , you should take ????

extreme protrusive-

lateral excrussion-
?????
anterior plane guidance i think-

....-

335- what is the diameter of gates glidden size 3

1.0-

1.7-

**1.0-

1.2-

336- Which L.A is contraindicated for pregnant

prilocaine-

lidocaine-

**mepivacaine-+ Bubivicain + procaine

Etidocaine-

387-Pedo pt in mixed dentetion has lesion in incisor & 1st molar white spot & weakness i
?think so what is Dx

dentin dysplasia-
amelogenisis imperfecta-

incisor molar hypominralization-

388-point of resistance in anterior teeth under ortho tx but have moderate to severe bone
?loss

middle root-

** more apically-

more coronally-

389- When you AIDs pt , the most important finding in the lab result you should concern is

CD4-

CD8-

basophile-

eosinophlie-

390-RPD metal frame try in , you find every thing is good but you notice the occlusal rest is
?smaller than the rest seat , when check it in the cast it was in same size , what will do

fill the gap by flowable composite-

do soldering on the rest then cast it-

redo impression ** i think it is the answer -

391-When you show photo of your patients to new pt but you did not take their
?permission , this violation of which ethic code

privacy-

autonomy-

confidentially-

332- CD4 level in AIDs pt is

011<-)011 ‫(اقل من‬

0011-

0211-

2111-
333-who is pt need prophylaxis befor surgery

All of these are correct answers

Prosthetic cardiac valve -0


Previous infective endocarditis -2
Comgenital heart disease -3
Cardiac transplantation recipients who develop cardiac valvulopathy -4

.)about endo diagnosis many Q.. and its easy..(read-3

334-currte activation angle

a-45-90

b-70-80

c-90-110

395-pt came with fracture in occlosal of amalgam

..High point in occlusion of restoration

306- pic... Gutta brecha trasing mean* chronic abical abscess

333- pedo pt cam with Prominence in his central incisor from platal side

Talon cusp

333-pic of tooth... the nebrs prop through the frcution with out connective tissue ressed

..Class III frcution invo

333-peptic ulcer pt which analgesics u will give him

.. Paracetamol

400- which cusp u will use it in molar classification for primary teeth

..distobuccal cusp of lower E..

401-GG used for


... Flarring coronal part

402-pt have upper 4 Equogingival what ther procured

.. I choose crown lengthening

403- wht the main purpose to do post

I choos Strength and retain the core for build up

405-Most important thing in parasymphesial fracture*

.Airways -

406- vibrating line determine by

T burnisher

407- pic. Submandibar infections space ?

I choose 2 molar

403- pic

.. Reverse bone loss


409-u want to do post in lower molar what it the dangerous thing may be occur when u
... use pesso remar

Stripping perforation

410-class III mandible. For ortho u extracted

... 1 premolar

411- asthmatic pt in endo


412- wide canal what the obuteration tech u will use

..I choose WVC

413- after bleaching by 1 day pt came with compling of pain wht is it

...Teeth sensitivey... if the pain during procedure...leakage from gingival dam

414- type of matrix in MOD

universal matrix , tofflemire matrix

-415 type of anterior teeth matrix

Cello. Matrix mylar

446-pic -25

Perio prop
443- pic:

Sicl scalar

443-attach gingival

. Gingival marginal to meccogingval line

419- many Q from oral medicine... alot of drugs

420-pt need mandible set back

.I choos BSSO

424 -for cemented crown what is type of GIS

Type 1 luting

‫*مالحظه‬
luting 0

restoration 2

liner, base 3

fissure sealant 4

orthodontic cement 0

core build up 6

422-wired hair cut

I choos unprofessionally

423-pt tell to doc don't touch my body with out my permission

Autonomy

424-wht is mean of Non maleficence

do no harm -

425-which muscle is elevated and retracted the mandible

I choos temporalis x

426- what cause of fiscal paralysis after LA ..? The Q about Needl Direction

! to far posteriorly x

427- pin depth in resto

2 mm
428- pic

Attrition

423- long stoooory .... with pic

Peri abutment

430- longg stooooory without pic

Distal abutment

434. function of artclutor

...Relationship maxill with part of art


432-distance bw 2 implant

mm 3

433- PPS (posterior palatal seal) for

Retention of upper CD

434-pic.... looong storry

epulis fissuratum

435-Pt. need RCT in tooth with PFM crown what type of bur use

round diamond in porcelain and trnssmetal in metal

436-Bulimia nervosa

.Erosion palate side of upper

437-F.. varinsh

438- F .. water

ppm 0

439- leewy space


Leeway space (maxilla 1.8) mandibal (3,4)
... Perment premolar with prymery molars

440- prmait space

Upper canin and lateral incasor

441- discoloration of single tooth

.Imporor of RCT improper RCT ,trauma.internal resorption,calcification

442-cause of ledge

...I choos fauiler of precurve of file

443- pt missing lower 6 and upper 6 supraeruption


..orth intrusion

444-nerve supply of premolar platal side

. Greater plantine

445-to break mesial side enamel class2

Hatchet

446-sequence of extraction

.Max... posterior then anterior

443-contributing to stop bleeding

.Gelfom

443-rubber dam wrinkled

Hole to far

449-pt need rct but the no seat for clamp in tooh structure

Clamp in next tooth ..split rubber dam

450.The most stable result after orthodontic procedure is


A. Widening of maxilla
B. Retrusion of mandible
C. Superior repositioning of maxilla
D. Downward movement of maxilla

452. Electric pulp test on the adults is not accurate because:


A) late appearance of fibers A ✅ ✅ ✅ ✅ ✅ ✅
B) late appearance of fibers C
C) early appearance of fibers A
D) early appearance of fibers C

453. The management of a single tooth posterior crossbite


requires
454. opening the bite to disclude the teeth
B. creating space in the opposing arch
C. correcting the functional shift of the mandible✅ ✅ ✅ ✅ ✅
D. rapid palatal expansion

454. An electric pulp tester – (EPT OR VITALOMETER) usually


elicits a response at a higher current than normal if a
tooth, being tested has:
a. Actue pulpitis
b. Chronic pulpitis
c. Open apex
d. A pus- filled canal
455. Periodontal flaps what it important
A. Acute Pericronitis ttt
B. Antibiotic
C. Remove occlusion force sub gingival
D. Rinsing✅ ✅ ✅ ✅ ✅ ✅ ✅

456. All corners of a periodontal flap should be


a. Sharp
b. Rounded ✅ ✅ ✅ ✅ ✅ ✅
c. Knife edge

457. 70 years old male patient comes to restore his badly


decayed upper second molar chance of involving pulp by
infection from dentin compared to young patient
a. Progress slowly pain in adult✅ ✅ ✅ ✅ ✅ ✅
b. Progress slowly by age
c. More progress in old then young
d. More progress and less pain in young then adult

458. Frictional recession in lower premolar the cause is:


A-inflammatory
B-genetic
C-allergy
D-tooth brushing

459. Open up the tubules to release the intra-pulpal :


* pressure✅ ✅ ✅ ✅ ✅ ✅ ✅

460. feature that describe the energy absorbed by a material


before deforming
toughness✅ ✅ ✅ ✅ ✅

461. crown to root ratio ?


2:3 *✅ ✅ ✅ ✅ ✅ ✅ ✅
☀ Optimum is 2:3
Minimum is 1:1
Ideal is. 1:2

462. linchen planes histpothalogical characteristic?


☀ Histological findings characteristic of oral lichen planus include hydropic
degeneration of the basal layer, lymphocytic infiltration in the subepithelial layer and
the absence of epithelial dysplasia; however, it is also frequent to observe hyperplasia
phenomena at the epithelial level, hyperkeratosis, acanthosis and ...

463. best radio for proximal caries ?


* bitewing✅ ✅ ✅ ✅ ✅ ✅
☀ For Posterior
Anterior is periapical

464. device to limit child extremities?


* - posey straps✅ ✅ ✅ ✅ ✅ ✅ ✅ ✅ ✅

465. most recurrent cyst?


* odontogenic Keratocyst✅ ✅ ✅ ✅ ✅

466. complete blood count test?


☀ complete blood count (CBC) is a blood test used to evaluate your overall health
and detect a wide range of disorders, including anemia, infection and leukemia.

A complete blood count test measures several components and features of your blood,
including:

Red blood cells, which carry oxygen


White blood cells, which fight infection
Hemoglobin, the oxygen-carrying protein in red blood cells
Hematocrit, the proportion of red blood cells to the fluid component, or plasma, in
your blood
Platelets, which help with blood clotting✅ ✅ ✅ ✅ ✅ ✅ ✅

467. bicondylar fracture in child without displacement or


or mat beresilience
malocclusion?
* closed treatment technique.✅ ✅ ✅ ✅ ✅ ✅ ✅ ✅ ✅

468. where does the submandibular gland open in the


mandible?
* Wharton duct
☀ The submandibular duct or Wharton duct or submaxillary duct, is one of the
salivary excretory ducts. It is about 5 cm. long, and its wall is much thinner than that
of the parotid duct. It drains saliva from each bilateral submandibular gland and
sublingual gland to the sublingual caruncle at the base of the tongue.✅ ✅ ✅ ✅ ✅
✅✅

469. area specific scaler?


- gracey
* gracey curettes are area specific to allow for deep scaling root planing and
periodontal debridmebt.

470. primary use of a dental wax ?


* * The primary use of waxes in dentistry is to make a pattern of appliances prior to
casting as many dental restorations are made by lost-wax technique, in which a
pattern is made in wax and put in the mold (investment materials).
* Cast construction of nonmetallic denture bases, registration of jaw relation and
laboratory work
✅✅✅✅✅✅✅
471. what to check last during try in:
A-aesthetics ✅ ✅ ✅ ✅ ✅ ✅ x
B-occlusion
☀ _Proximal contact
_Marginal integration
_Stability
_Occlusion
_Characterization and glazing

472. plaque consist of - bacteria ?


* * Bacteria. The bulk of the microorganisms that form the biofilm are Streptococcus
mutans and otheranaerobes, though the precise composition varies by location in the
mouth. Examples of such anaerobesinclude fusobacterium and actinobacteria✅ ✅ ✅
✅✅✅✅✅✅✅✅

473. bacteria for initiation caries and progression of bacteria ?


* Streptococcus mutans and lactobacillis ✅ ✅ ✅ ✅ ✅ ✅ ✅ ✅ ✅

474. question about a SCC on tongue poorly differentiated ... -


bad prognosis+ high recurrence
☀ Tongue cancer is a type of mouth cancer, or oral cancer, that usually develops in
the squamous cells on the surface of the tongue
Known risk factors include:

smoking or chewing tobacco


consuming alcohol in excess
eating a diet low in fruit and vegetables and high in red meat or processed foods
having a human papillomavirus (HPV) infection
having a family history of tongue or mouth cancers
having had previous cancers, particularly other squamous cell cancers
Older men are the group most at risk of tongue cancer. Oral cancers are most common
in those aged 50 or above

People with tongue cancer will usually require surgery to remove the cancerous tissue
Inaddition to surgery, some people may have radiation or chemotherapy treatment to
kill any cancerous cells that remain. ✅ ✅ ✅ ✅ ✅ ✅ ✅ ✅ ✅ ✅

475. Vertical root fracture diagnosis :


1. X ray✅ ✅ ✅ ✅ ✅
2. Vertical percussion
3. Horizontal percussin

476. recent two bridges in posterior of mand restoration pt


came after months came with sever pd disease and pain
what you will see if u left him for a year
A. -loose of abutments
B. Mobility✅ ✅ ✅ ✅ ✅ ✅ ✅
C. Connector fracture

478. Patient with bad occlusal force came back with porcelain
fracture
A. Tell lab to do stronger porcelain
B. Increase porcelain thickness
C. Night guard✅ ✅ ✅ ✅ ✅ ✅ ✅ ✅ ✅ ✅ ✅ ✅

479. Risk of vertical root fracture is a disadvantage of


A vertical condensation✅ ✅ ✅ ✅ ✅ ✅ ✅ ✅ ✅
B lateral condensation
C compaction .method
D sectional method

480. Contraindication Of Root Resection


A. Vertical Root Fracture
B.Vertical Bone Fracture✅ ✅ ✅ ✅ ✅ ✅ ✅ ✅ ✅
C.Horizontal Root Fracture

481. A patient comes to your clinic with severe pain,.there's aJ-


shaped lesion a long the root of RCT tooth inX-ray. What 's
the diagnosis
A. Periodontal pocket
B. Vertical fracture✅ ✅ ✅ ✅ ✅ ✅ ✅ ✅
C. Lateral canal
D. Secondary periodontitis

482. the fracture of root at which will be most difficult to


manage
A. Apical third
B. Coronal third
C. Middle Third
D. Vertical fracture✅ ✅ ✅ ✅ ✅ ✅ ✅ ✅

483. What kind of root fracture in a tooth has the best


prognosis ?
* . Apical third✅ ✅ ✅ ✅ ✅ ✅ ✅ ✅ ✅ ✅

484. After u did RCT to your pt he came back to the clinic after
few days with sever pain on biting, you did x-ray and it
revealed that the RCT filling is very good, but u saw
radiopaque ,thin (film like) spot on the lateral border of the
root what is the most probable diagnosis:
A. Accessory canal.
B. Vertical root canal fracture✅ ✅ ✅ ✅ ✅ ✅ ✅

485. Isolated pocket in


A. Vertical root fracture.(most
.
B. Palatogingival groove.
C. Endo-origin lesion.
D. All✅ ✅ ✅ ✅ ✅ ✅ ✅ ✅

486. RCT is contra indication in all Except


A. Vertical root fracture
B. Un-restored tooth
C. Insufficient periodontal support
D. Large preiapical area or Medically compromised pt .✅ ✅ ✅ ✅ ✅ ✅ ✅ ✅ ✅
487. best treatment to be done for tooth with vertical root
fracture:
A. bonded filling
B. porcelain fused to metal crown
C. all ceramic crown
D. extraction✅ ✅ ✅ ✅ ✅ ✅ ✅ ✅
E. Non of the above

488. most appropriate root configuration for abutment of FPD


:
A. Conical
B. Multirooted+ long divergent
C. Root that wider mesiodistally more than buccolingually

489. most difficult root canal in RCT :


A. Long – narrow – curved✅ ✅ ✅ ✅ ✅ ✅ ✅ ✅
B. Short – wide – straight

490. oblique ridge in upper 1st molar :

A. Run from distopalatal cusp to mesiopalatal cusp .


B. Run from mesiobuccal cusp to distobuccal cusp .
C. Run from distobuccal cusp to mesiopalatal cusp .
D. Run from mesiobuccal cusp to distopalatal cusp .

491. Acurate detection of dental caries :


* Diagnodent .
* True it's quantitative

492. Acurate detection of vertical root fracture :


A. Transillumination as the light pass through the tooth✅ ✅ ✅ ✅ ✅ ✅ ✅ ✅ ✅
B. C.T scan

493. percentage of ( Naf ) in fluoride varnish :


A. 0.5%
B. 5%

494. percentage of ( F ) in mouth rinse that found In pharmacy


center?
* Daily 0.05 %
Weekly 0.2%
495 _ Type of topical ( F ) that give to child of 2 years old :
A. foam
B. varnish
C. gels
D. mouth rinse

496. Major component of Gatta percha :


* . Zno
* Gutta-percha endodontic filling points were found to contain approximately 20%
gutta-percha (matrix), 66% zinc oxide (filler), 11% heavy
metal sulfates (radiopacifier), and 3% waxes and/or resins (plasticizer). The
mechanical properties were indicative of a partially crystalline viscoelastic polymeric
material✅ ✅ ✅ ✅ ✅ ✅ ✅ ✅

497. Busy dentist take the alginate imperssion and put it in


water , the alginate will develop :
A. Synersis
B. Imbibitation✅ ✅ ✅ ✅ ✅ ✅ ✅
C. Chalky appearance

498. X-ray shown open apex 0.9 mm of central incisor of 9


years old following trauma after dentist does cleaning and
shaping , which appropriate procedure to close the canal :
* Apexification✅ ✅ ✅ ✅ ✅ ✅ ✅

499. Dentist remove caries from upper lateral incisor , amonge


this he saw the shadow of the pulp , if he decid to put
composite as final restoration which is the best liner for this
case :
A. Caoh ✅ ✅ ✅ ✅ ✅ ✅ ✅
B. Gic
C. RMGIC

500. The most acidic solution of the followers :


A. 35 % Phosphoric acid✅ ✅ ✅ ✅ ✅ ✅ ✅
B. EDTA
501. patient come to your clinic with fall down ceramic bridge ,
the 1st step to cement it by resin is :
A. Applying Silane Coupling Agents to ceramic
B. Applying hydroplouric acid to ceramic as etchant✅ ✅ ✅ ✅ ✅ ✅ ✅

502. patient come to your clinic with fall down resin bonded
bridge , the steps of cementation in order is :
A. sandblasting
B. applying acid etch + bond agent on the tooth
C. cement with resin
E.all aobove ✅ ✅ ✅ ✅ ✅ ✅ ✅ ✅ ✅ ✅

503. subluxation trauma splinting for how long :


A. 2 week✅ ✅ ✅ ✅ ✅ ✅ ✅
B. 6 week
C. 8 week

504. poorest prognosis for avulsed tooth :


A. Open apex less than 24 hrs.
B. Open apex more than 24 hrs .
C. Closed apex more than 24 hrs✅ ✅ ✅ ✅ ✅ ✅ ✅ ✅
D. Closed apex less than 24 hrs

505. HPV affect which organ :


A. Skin
B. Oropharynx
C. All above ✅ ✅ ✅ ✅ ✅ ✅ ✅ ✅

506.patient with road accident come with fracture in nose


bridge and eyes :
A. Lefort I
B. Lefort II ✅ ✅ ✅ ✅
C. Lefort III
D. Zygomatic fracture

507.patient with cervical caries that extend to the root on the


upper premolar :
A. GIC ✅ ✅ ✅ ✅ ✅
B. Composite
C. Amalgam
* Root caries is rmgi ( first choice)

508.patient come with swelling on posterior mandible with


trismus and pain during chewing , on examination there is
carious lower 6 :
A. Submassteric space
B. Submandibular space

509.complication of extraction of wisdom tooth :


A. Vertical defect ✅ ✅ ✅ ✅ ✅ ✅
B. Horizontal defect
C. Furcation involvement

510.patient take pilocarpine drug , this medication for which


disease :
* Sjogren's syndrome✅ ✅ ✅ ✅ ✅
☀ This medication is used to treat symptoms of dry mouth due to a certain
immunedisease (Sjogren's syndrome) or from saliva gland damage due to
radiation treatments of the head/neck for cancer. Pilocarpinebelongs to a class
of drugs known as cholinergic agonists

511.patient with denture stomatitis treatment is :


A. Antifungal ✅ ✅ ✅ ✅ ✅
B. Antiviral
* Antifungal nystatin
Clotrimazole
* *Denture stomatitis* is common type of of chronic oral candidiasis

512.patient with Parkinson disease has grade II periodontitis


excessive plaque , you advise him to use :
A. Electronic brush ✅ ✅ ✅ ✅ ✅ ✅ ✅ ✅ ✅
B. CHK mouthwash
C. Interdental brush

513.patient loost 14 and 15 teeth , the best numbers of


abutments that achieve the ante's law :
A. 13 and 16 ✅ ✅ ✅ ✅ ✅ ✅ ✅
B. 13 , 16 and 17

514.the deepest point that currete can reach subgingivally to


remove the subgingival calculus :
A. 2-7

B. 3.7

515.patient with class II malocclusion , what is the extraction


that correct case :
A. Upper 1st premolar ✅ ✅ ✅ ✅
B. upper 2st premolar

516.patient with class III malocclusion , what is the extraction


that correct case
A. Upper 1st premolar
B. lower 1st premolar✅ ✅ ✅ ✅ ✅

517.when mesiobuccal cusp of upper 6 occlude mesial to


buccal groove of the lowe 6 , what is malocclusion :
A. class I
B. class II ✅ ✅ ✅ ✅ ✅
C. class III

518.10 years old child that receive quard helix , the mode of
expansion :
A. skeletal
B. Dental
C. 1/2 skeletal and 1/2 Dental
D. -2/3 dental, 1/3 skeletal✅ ✅ ✅ ✅ ✅

519.Functional appliance , which one :


A. Bionator ✅ ✅ ✅ ✅ ✅ ✅
B. Headgear
C. Revers headgear

520.condyle rotate around which line :


A. Hinge axis ✅ ✅ ✅ ✅ ✅ ✅
B. Midline
C. Vibrating line
521.porcelain fused to metal crown , in lap the investment
material :
A. Silica investment ✅ ✅ ✅ ✅ ( firest choice is phosphate investment if found in
choices)
B. Resin investment
C. Gypsum investment

522.In descending order put the following that affect


resistance and retention of crown when doing preparation :
A. Freedome of displacement
B. Type of cement
C. Height
D. Parallisme

☀ FORCE – HEIGHT- PARALLISM-ROUGHNESS OF FITTING SURFACE-


TYPE OF CEMENT – THICKNESS OF CEMENT

523.child 8 years old come with his brother 14 years seeking


for orthodontics :
A. Take photographs
B. Take impression
C. Ask their parents to come✅ ✅ ✅ ✅ ✅ ✅ ✅ ✅

524.patient with torus palatinus 10×10 mm in the middle of


palate , what the best of major connector :
A. Mid palatal strap
B. Horseshow strap ✅ ✅ ✅ ✅ ✅ ✅ ✅
C. Full coverage palatal strap

525.patient with class III mode I kenndy classification , best


major connector :
A. Mid palatal strap ✅ ✅ ✅ ✅ ✅
B. Horseshow strap
C. Full coverage palatal strap

526.patient receive 2 sets of new complete dentures before


one month come to clinic with lower lip numbness , the
possible cause will be :
A. Mentalis nerve
B. Pressure of mental foramen✅ ✅ ✅ ✅ ✅ ✅ ✅ ✅
C. Pressure of inferior alveolar

527.patient has missed lower 6 , have tilted lower 7 ( 30º ) ,


possible complication that occur when doing preparation of
FPD on 5 , 6, 7 :
* Pulp exposure✅ ✅ ✅ ✅ ✅ ✅ ✅ ✅ ✅ ✅ ✅ ✅ ✅ ✅ ✅

528.After doing apicoectomy for a tooth when can we place a


crown?
A. 1-2 months.✅ ✅ ✅ ✅ ✅ ✅ ✅
B. 2-3 months.
C. 3-4 months

529.what is position for exctracionlower molar of pedo.


Patient

A. Mandeble parall to flore✅ ✅ ✅ ✅ ✅ ✅ ✅


B. Mandela 45 to Flore
C. Maxilla parall to Flore
D. Maxilla 45 to Flore

530_pt complain from burning sensation in examination toung was red and free from
filiform which test used to dignose?

Cytologyical

531_diabatic pt with dray mouth and long story ... which impression material should
be avoided in this case ??
#Zinc-oxide eugenol because it will irritate the mucosa of drymouth

#Silicone impressions is preferred

532_which cardiovascular disease enhance anasthesia??


A beta blocker ✅ ✅ . Use of local anesthetic containing a vasoconstrictor should be
avoided in patients taking beta-blocker medication because of a possible adverse drug
interaction. However, when a vasoconstrictor is indicated for hemostasis, the local
anesthetic should be administered slowly and in small amounts as pulse rate and
blood pressure are being monitored. The patient should be informed that the duration
of anesthesia might be prolonged.

B calcium channel blocke

533_instrument used after root planning to check the surface ??


Explorer
534_color code of file size 6?
Pink

535_length of file size 20?


21-25-31

536_clefet lip !!
Uncomplete closer between nasal and two maxillary process
8_how to defrantiate between periodontal and preiapical abcess?
Vitality test

537_drug cuse gingival overgrowth ? ‫الخيارات مافيها فينتوين شوفو كل االدويه المسببه‬
-anticonvulsants (phenytoin)
-calcium channel blockers (antihypertensives such as nifedipine)
-cyclosporine, an immunosuppresant

538_drug link emotinal stress and initiate periodntal proplems????


539_disadvantge of condensiton silicon??
-High ploymerization shrinkage
voltaile by-product
• Low tear strength
• Hydrophic

540_larg cavity and u want to do pin to increase retention what is size of pinhole?
_same size of pin
_larger than pin
_smaller ‫اظن كدا الخيارات‬

541_tug back of gutta percha indicate of what?


Good apical seal

542_pt complain of loss sensation in right side of mandible after( 2 weeks or 2 month
) from implant procedur what happend .....?
IAN damage

....
543_blood supplay of pdl ?

Superior and inferior alveolar nerve


544_pt with missing 11,12,13 what is the kennedy class?
Class3
545_tooth with 1.5 mm fasciolingual mobility grade ??
Grade 2
18_sickle scaler used in ??

Supragingival calculs

546_tip size of k file??

According to size in question

547_ predisposing factor af ANUG?

548/ full name of Edta and concentration?


Ethylenediaminetetraacetic Acid
17%

549/ hemophilea A test?


APTT

550/ G.G. tip diameter of size 2?


Size1/50
Size2/70
Size3/90
Size4/110

551/ color of endo file 30?


Blue

552/ doctor tell to nurse take off your ring why?


- one of the answer coz when the nurse washing her hand all contamination go out
and ring can't hinder.

553/ in x-ray open marign mesially and distally why?


obstructed path of insertion

554/ tooth with amalgam restoration doctor will keep it to pre-clinical how will keep it?
- put it in formaline 10% then in saline

555 direction of collagen fibers in presence of implant?


- parallel to surface of implant.

556/ when do frenectomy in case closed upper distema btween two centerals?
-after ortho treatment✅
-during
-befor
-other answer

557/ treatment multiple radeulucency in lowe right jow?


Marginal resection and If there surgical enculation with curretrge is correct

558/ how check posterior palatal seal?


T- burnishar

559/ case of loss upper two cetrals right and left with padly decay in upper left
premolar with intact teeth of lower jow
what the kennedy class?
Class IV

560/ loss of 11,12,13what the kennedy class?


Classlll

561/ patient of active TB under medicine when u can strat to treat him?
2-3 week

562/ alternative antibiotic in patient has


allergy from pencilline?
-cephalosporine
- amxicillin
-clindamicine✅
-and other one els

563/ tooth with discomfort in lower premolar area when doctor check there is deep
pocket in mesial aspet 9mm and no response with cold test in tooth and there is
periapical radiolucency so you will strart treatment with?
-endo✅**
-perio
-and 2 other choices

564/ in x-ray upper right premolar remaining rooth above the bone how will you
treating?
- exo
-surgical endo
-non-surgical end✅

Note:- there anather tow Q about surgical and non surgical endo.

565/ pic the name of holding the instrument?


-Palam thumb holding

566/ pic what type of finger rest?

-Oppiste arch

567/ disinfection gata perch in?


-sodium hypochlorite 5.25%

568/ varnish under amalgam?


-cobalite

569/ pic of case came to check up no symptomes and good response of teeth and x-
ray analysis is good also so!!
-healthy

570/ dye to detect bacterial of TB?


-Acid fast

571/ case about generalized gray crown?


-dentenogenesis imperfecta.

572/ case about 6 molar upper right and left and incisors has incipient caries?

Fluoride application

573/ treatment of avulsion primary tooth?


extraction

574/ side effect of patient taking... ( ‫) السؤال ناقص‬


28/ Q about palate plate ( palatoplate).
29/ one Q about major connector.
30/ one Q about minor connector.

575/ 37 with mesially carved mesial root


What the appropriate tratment?
-handle file
-rotary file
-will not use G.G
-handle and rotary files

576/ healthy patient did regular vists to clinic and today came for check up doctor
decide he will do hepatitis test why coz pt has?
-pale skin
-yellowish skin with white eye
-yellowish skin and eyes ✅*
-and other 2 choices

577:Uncontroled diabetic pt have swelling in the parotid gland, the biopsy show
-acinar atroph
-acinar hypertrophy
-lymphatic ....

578:diagnostic test for bronchial asthma:


Pulse Volume
Pulse flow +EPF+SPIRPMETRY

579:bacteria found in siphilis:


Treponema palladium

580:large lesion in the ventral surface of the tongue how to treat or diagnose:
-excision biobsy
-incisional biopsy Because it is large

581: Q’s about gingival phenotype.

The term “gingival phenotype“ has been introduced to address the common clinical
observation of great variation in the thickness and width of facial keratinized tissue
[Gingival biotype is described as the thickness of the gingiva in the faciopalatal/
faciolingual dimension. Reduced gingival thickness is one of the factors that can
cause periodontal attachment loss and marginal tissue recession in a patient, which is
a major concern for periodontal disease progression
The term periodontal biotype introduced by Seibert and Lindhe categorized the
gingiva into “thick-flat“ and “thin scalloped“ biotypes Thick gingival biotype usually
depicts broad zone of keratinized tissue with flat gingival contour which indicates
thick underlying bony architecture and is more resilient to any inflammation or
trauma.

285: most important codes in ethics: ‫موجود في األسأله السابقة بس المعطيات كانت غلط‬
-competence, autonomy, .......... Five ethical principles in the code:
- 1. Veracity- truthfulness
- 2.Justice- fairness
- 3.Autonomy- self-governance
4. Beneficence - do good
583:primary first molar tooth the root shape 5. Nonmaleficence- do no harm
-short convergent
x -long convergent
-short divergent
-long divergent
.

584Doctor have needle injury and the pt is diagnosed with HIV, the maximum time
allowed to get post exposure prophylaxis:
5 weeks
8 weeks within 4 weeks
6 weeks
The incidence of human immunodeficiency virus (HIV) infection continues to rise among
core groups and efforts to reduce the numbers of new infections are being redoubled. Post-
exposure prophylaxis (PEP) is the use of short-term antiretroviral therapy (ART) to reduce
the risk of acquisition of HIV infection following exposure. Current guidelines recommend a
28-day course of ART within 36–72 hours of exposure to HIV.

585 case scenario and the Q what type if gic?

The answer is type one.

286* Q there is pic of bone defect and asking about the type of defect:
-one wall defect
-two wall defect
-three wall defect
285treatment of xerostomia:
-midazolam
-Salagen (PILOCARPINE )

588:Rheumatic arthritis pt will have


-TMJ problem
-condylar hyperplasia
-bifid condyle
589maximum dose of epinephrin for cardiac pt according to american heart
association:
-0.04
-0.2 healthy patient
-1

590the optimum of total occlusal convergence (TOC):


10-21 15-12
5-7
25-30

591Pt with missing 25 and 24 with high lip line and will do RPD what type rest will
chose:
-I bar
-twin flex clasp hidden clasp in insufficient area +esthetic
-aker clasp
-wrought wire clasp

592The worst Type of bone


Type 1
Type 2 best one
Type 3
Type 4

593 tooth start to develop in the embryo


4th week
6th week
8th week
594 pt with cleft lip and palate will have:
Maxillary deficiency Maxillary deficiency is a common developmental problem
in cleft lip and palatepatients.
Mandibular deficiency
Maxillary prognathism
Mandibular prognathism

595Q about curette scaler depth for calculus removal in healthy periodontium:
5.5. mm
x 7.5 mm
4.5 mm
2.5 mm

596 widening and loss of lamina dura seen in:


-hyperparathyroidisim
-hypoparathyroidism

597hypercementosis and ankylosed teeth seen in:


-paget disease

598 teeth with generalized yellow and brown discoloration with white opacities:
-fluorosis
.
x -tetracycline
-amelogenesis imperfecta
-dentinogenesis imperfecta

599Q about definition of Antes law


600Q about cleft lip:
The answer is: failure of fusion between medial nasal process and maxillary process.

601Gic polishing paste?


The answer aluminum oxide disc but paste pumice

602‫ صوره انتفاخ مكان ال‬submandibular gland ‫وسأل ايش اللي ورا االنتفاخ‬:
platysma muscle ‫ هايويد بون‬،‫ ستيرنوماستود مسل‬،‫مسل‬

603‫اذا بنسوي كومبليت دنشر ثاني كم الوقت اللي ما يلبس فيه البيشت الدنشر لين الموعد علشان نسوي له دنشر جديد‬
٢٧ ‫ساعه‬
٧٢ ‫ ساعه‬24-48
٣ ‫ساعات‬

604:Q about enamel hypoplasia.

Enamel hypoplasia is a defect of the enamel that only occurs while teeth are still
developing. Still, it can affect both baby teeth and permanent teeth. The condition
results in thin enamel, which makes your teeth vulnerable to dental decay.
* pits, tiny groves, depressions, and fissures
* white spots
* yellowish-brown stains
Also cases amelogenesis imperfecta,

605. AH plus better than AH 26 by??


* high sealing✅✅✅✅✅✅

606. use composite flowable??


* small class v caries✅✅✅✅✅✅
and class VI
607. fractured tooth to alveolar crest , whats the best way to
produce ferrule effect to return tooth to occlusion??
* crown lengthening ( this answer 100% right NOT post
and core)✅✅✅✅✅✅✅
608. the endochondral of skull occur in??
* synchondrosis✅✅✅✅✅✅✅✅

609. which material is mixed very slowly to ensure long


working time??
* zinc phosphate cement✅✅✅✅✅✅✅

610. what time for reepithelialization is?


5_ 14 days✅✅✅✅✅✅✅✅✅✅
7-10 epithilum ,connective tissue 21 days
611. what is the internal angle during scale??
* 70-80 degree✅✅✅✅✅
grasy 60-80 ,universal 90
612. what is bacteria cause and progression caries??
* Streptococcus mutans and lactobacillus✅✅✅✅✅✅

613. what the lesion found at junction between soft and hard
palate and surrounded with pseudoepithelium and
hyperplasia in salivary gland??
* necrotizing sialometaplasia✅✅✅✅✅
Necrotizing sialometaplasia (NS) is a benign, ulcerative lesion, usually located
towards the back of the hard palate. It is thought to be caused by ischemic necrosis
(death of tissue due to lack of blood supply) of minor salivary glands in response to
trauma

614. function of surfactant is??


* wet ability ( decrease tension wall)✅✅✅✅✅
615. what virus cause herpes infection??
* HSV type (1)✅✅✅✅✅
There are two types of herpes simplex virus,type 1 (HSV-1) and type 2 (HSV-
2). HSV-1more commonly causes infections around the mouth while HSV-2 more
commonlycauses genital infections. They are transmitted by direct contact with body
fluids or lesions of an infected individual.

616. cause of verruciform xanthoma ?? truma +allergy+HPV


* viral infection ( HPV)✅✅✅✅✅
Due to its clinical and histopathological resemblance to human papilloma virus-
induced lesions, verruciform xanthoma was believed to be caused by HPV. ... It is
sessile or pedunculated and can resemble leukoplakia or squamous papilloma

617. largest structure in enamel??


* enamel rods✅✅✅✅✅ prisms

618. instrument use for waxing pontic ?? ???


* spoon excavator✅✅✅✅ or may be pk4
no 4,5 are wax carver
619. how to make sugical mask effect??
* change it between patient✅✅✅✅✅✅
620. what the content structure of salivary gland that has the
role of control the concentration of chloride and sodium
in saliva??
* striated duct

621. use the dental floss is??


* remove the bacterial plaque✅✅✅✅✅✅
, dental floss is the only way to remove plaque between your teethand
keep tartar out of these hard-to-reach areas. Rinse daily. Use an antiseptic
mouthwash daily to help kill bacteria that

622. discoloration of endo treated teeth because??


* incomplete removal of pulp tissue✅✅✅✅✅
In compete removal of gutta percha above cemento enamel junction yellowish color
Heamorage or incomplete removal of pulp tissue black color

623. what the nerve that pass over ilium which can be injury
during surgery??
* lateral femoral cutaneous nerve✅✅✅✅✅✅

624. what cell consider the mos??


* is basal cell of epithelium✅✅✅✅✅
Parenchymal Cells
strafied squamous=stratified cell anti radio sensitive

Type I Cells are cells actively dividing that do not differentiate. They are
the mostsensitive to radiation, and are often the cause of radiation induced early
effects. They include the stem cells of most rapid turnover tissues: Basal cells of
the skin epithelium
625. from PEE??
* is mask✅✅✅✅ take on :gown mask faceshield gloves
Take off: gloved facesheild gown mask
PPE >>personal protective equipment

626. most flexible file is??


Remer ✅✅✅✅✅
Positivity of rake angle
H more
Then
K file

More cutting efficiency


H
Then
K

Flexibility
Re
Reamer
Then
K
Then
H

627. what material to eliminate gamma 2??


* 13% copper✅✅✅✅✅✅✅ 12%

628. what is name the notch in lower border of mandibular


jaw??
* antegonial notch✅✅✅✅✅✅

629. root resorption due to??


* force magnitude✅✅✅✅✅✅

630. patient with multiple caries high plaque index erythema


and edematous the gingiva pocket (2-4 mm) what is
periodontal diagnosis??
* juvenile periodontitis✅✅✅✅ ????

631. second most common malignant tumor in bone jaw is??


* chondrosarcoma✅✅✅✅✅
Chondrosarcoma is a malignant tumorcomposed of cartilage-producing cells. It is
the second most common primarymalignancy of bone, and one of
the mostdifficult bone tumors to diagnose and treat
Osteosarcoma, also known as osteogenic sarcoma, is the most
common type of bone cancer and typically starts in bone cells in the arms,
legs or pelvis. ...
Chondrosarcoma forms in cartilage cells and is the second most common
form of the disease.

632. pt with deafness and lion face and bone deposition is??
* Paget’s disease✅✅✅✅✅✅

633. ideal biological width after crown lengthening is??


* (3mm) all aspects✅✅✅✅✅✅✅
2.7

634. best graft for condylar is costochondral graft found


between ??
* junction between sternum and ribs✅✅✅✅✅✅✅

635. function of ridge mapping gauge is??


* to measurement of bone thickness without tissue
incision✅✅✅✅✅✅✅

636. most common diabetes feature is??


* periodontal abscess✅✅✅✅
most common feature of periodontal disease :-
( periodontal abscess) I think CAL
Most common in diabetes is :-
Periodontal disease *****
Gingival recession

‫ هنا بختار‬periodontal disease


‫علشان اشمل‬

637. technique for treatment zygomatic fracture is??


* GILLIES temporal approach✅✅✅✅✅

638. most common cause the trismus is??


* trauma to muscle✅✅✅✅✅

639. incision for palatal torus is??


* inverted or double ( Y ) shaped✅✅✅✅✅

640. name and function nerve number ( VIII ) is???


* name is : vestibulocochlear nerve ... function is
hearing✅✅✅✅✅✅✅

641. function of accessory nerve number ( XI ) is??


* move muscle of head and neck and shoulder The spinal accessory nerve provides
motor function to the sternocleidomastoid muscle, which extends the neck and the trapezius,
as well as the upper back and shoulder.

642. disadvantages of full thickness flab is??


* delayed secondary healing✅✅✅✅✅
uncomfortable to the patient ,sensitivity, root Caries
643. reticular form in lichen planus is?? find on the cheeks as a lacy web-like ,
* linear or network white bands✅✅✅✅✅ white threads that are slightly raised
644. rare form in lichen planus is?? (wickham’s striae)
* is bullous form✅✅✅✅✅
Bullous lichen planus is a rare variant of lichen planus. It is characterized by vesicles
or bullae, which usually develop in the context of pre-existing LP lesions. It is often
misdiagnosed and should be differentiated from other subepidermalbullous diseases
especially lichen planus

645. viral sialadenitis ( mumps ) caused by??


* paramyxovirus✅✅✅✅✅✅
646. what the gland effect by Sjogren’s syndrome??
* lacrimal and salivary gland✅✅✅✅✅
647. what the name duct of sublingual glnad??
* bartholin duct✅✅✅✅✅✅
ducts of Rivinus and the largest of all the bartholin duct .
648. rare common complication of trauma to pterygoid plexus
veins is??
* Cavernous sinus thrombosis and most common hematoma

649. kaposi sarcoma the human herpes virus number is??


* number ( 8 )✅✅✅✅
Human herpesvirus 8 (HHV-8), also known as Kaposi's ...

650. the types high risk in human papilloma virus is??


* types ( 16 and 18)✅✅✅✅✅
High-risk HPV strains include HPV 16 and 18, which cause about 70% of cervical
cancers. Other high-risk HPV viruses include 31, 33, 45, 52, 58, and a few others.
Low-risk HPV strains, such as HPV 6 and 11, cause about 90% of genital warts,
which rarely develop into cancer. These growths can look like

651. the most common cause of fungal infection in the oral


cavity is??
* candida albicans✅✅✅✅✅✅

652. most common sign in treacher collins syndrome is??


* retrognathic mandible✅✅✅✅✅
Treacher Collins Syndrome :
- Deformity of external ear & hearing loss. - Drop down of eye. - Deformity of zygoma
bone ( Malar bone or Cheek bone ). - Cleft palate. - Mandibular retrognathia. - Open
bite. - Breating problems. - Mentally normal.

653. the resin bonded bridge is??


* Maryland bridge✅✅✅✅✅✅ indicated in young and mobile teeth

654. picture for inferior alveolar nerve block?( ‫??? ) الجواب هو ساليد‬

655. picture for amelogenesis imperfecta


Amelogenesis imperfecta (AI) is a congenital disorder that presents with a rare
abnormal formation of the enamel[1] or external layer of the crown of teeth, unrelated
to any systemic or generalized conditions.[2] Enamel is composed mostly of mineral,
that is formed and regulated by the proteins in it. Amelogenesis imperfecta is due to
the malfunction of the proteins in the enamel
(ameloblastin, enamelin, tuftelin and amelogenin) as a result of abnormal enamel
formation via amelogenesis.[3]✅✅✅✅✅✅✅✅

656. picture for compound odontoma??


A compound odontoma still has the three separate dental tissues (enamel, dentin
and cementum), but may present a lobulated appearance where there is no definitive
demarcation of separate tissues between the individual "toothlets" (or denticles). It
usually appears in the anterior maxilla.✅✅✅✅✅✅

Systemic candida in pt. with AIDswhat is the best medicine:


a-amphotrecinB
b- fluconazol✅✅✅✅✅✅✅✅✅✅✅✅ ketoconazole are less effective both
systemic

657. The number of roots that are formed is determined by the


:
A- number of root sheaths developed by the enamel
organ.✅✅✅✅✅✅✅

B- number of root sheaths developed by dental sac.


C - number of medial ingrowths at the epithelial
diaphragm.
D- Thickness of the cervical loop.

658. a child 13 years old, catch on probing newly erupted 2nd


molar, other teeth caries free, which is best done:
a. fissure sealant✅✅✅✅✅✅✅
b. fluoride varnish

659. Use of water during cavity preparation is mainly to:


a. Wash debris
b. Decrease heat of hand piece
c. Decrease heat of dentin✅✅✅✅✅✅✅

660. Most prevalent site for fluorosis:


a. Upper premolars✅✅✅✅✅✅
b. lower deciduous incisors upper incisors
c. lower 1st permanent molar
* upper incisors is first choice if found in choices

661. Patient came few days after Zn containing amalgam


restoration, complaining of pain, isolation couldn’t be
achieved, what is the suspected cause:
A. Delayed expansion✅✅✅✅✅ When the saliva reacts with the zinc inside the restoration,
hydrogen gas will be formed, and this gas will cause the amalgam to slowly expand over time. This phenomena is
called delayed expansion, and delayed expansion can after weeks and months result in pain and cusp fractures.
B. -leakage
C. secondary caries

662. Which is best material used as cementation of provisional restoration


A. polycarboxylate
B. zinc oxide✅✅✅✅✅✅✅
C. -zn phosphate

663. Most conservative method to lighten incisors:


A. Bleach
B. veneer
C. crowns
D. microabrasion

664:condyl fracture

665:after surgery procedure pt has numbness of lift side of mandible


???? not clear
666:after insert the crown pt come back and Compline of pain on bit
High contact point
667:pregnant women smoker what happen for his child
( cleft lip and palat)
668:Liner under amalgam
CAOH
668: High vertical dimension caused
Clicking –tmj pain- bruxism
669:Drugs caused xerostomia and salivation

670: Tp pt activation and non activation how u will treatment


If he cured treat him like normal patient but if he has active TB just do the
emergency treatment and let him on medication for 2-3 weaks
671: Witch nerve supply condayl side
The auriculotemporal nerve is a branch of the mandibular nerve (V3)
672: More questions about reversible and unrevesible pulpitis and acute
necrotic symptomatic and asymptomatic
Read endo diagnosis
673: Pc of Pt with mobile teeth and ask u about gab between root and
alveolar it’s resorption or just gap :
Not clear
674: Question if pt with double vision :
It is call diplopia ( the nerve that will be affected abducens nerve , and if with
diplopia he can not move his eyes oculomotor nerve )

675: Pt request his extracted tooth to keep it with him ( how u will sterilization
tooth before he taken )
Saline and disinfected the tooth and give him
676:Attrition and abrasion question :

677: Ethic question of do not harm and parentlism


do not harm non maleficence
paternalism action of feeling you have authority on the patient and you know
what’s the better for him and not take his opinion in treatment
678:Over hanging restoration with resorption bone
(reason food accumulation or plaque accumulation )
679:Witch kinddy class full seated on teeth
Class 4
680:Pedo patient 12 yr with missing teeth witch prosthodontics will used
removable or fixed

681: Instruments use to detect vibrating line


T-burnisher
682: Pc of instrument use for seated the matrix band on tooth
Final set of band
683:Nerve support premolar palatally
Gretear palatine nerve
684: Overhang of end sealer apically and patient uncomfortable how to
treatment
In cases of root canal sealer extrusion, surgical treatment should be
recommended only if clinical symptoms are present or if there is radiographic
evidence of an increasing periapical lesion.
685:First sign of gingival inflammation
GCF exudate
686: Deep pocket 7 mm with calculus sup gingival how to remove:
( gracely / secle scaler / ultrsonic)
687: Pedo patient missing 11 the RPD rest on which teeth
????

688-11yrs patient came complaining of recurrent infection related to lower premolar and
there is sinus tract

?The tooth is free of caries , normal periodontium , what is the cause


Internal resorption

dens invagenatous

necrotic pulp

680-During the FPD rocking metal try in-

? What to do

, Pick up impression (I think)

, permanent cement

temporary cement

690-Supra-erupting 16 cause wedging of food in the lower space of 46 ?? Whats the name of
the cusp

? Wedging cusp

Contact cusp

More two option cusps

600 : When you give Epi + heart failure may cause

Respiratory problem

Angina

Low blood pressure

602 Most important ethical codes


.
Competence, Autonomy , compassion

profitability, Autonomy, compassion

Competence, profitability, compassion

There was no professionalisms

693Most common man fracture Condylar 29.1%


Angle 24.5%
Angle , body , parasymphesis symphysis 22%
body 16%
Ramus1.7%
coronoid 1.3%
694 The end stage of renal disease shows
petechia and osteodystrophy

695-Patient taking ca blockers and has gingival overgrowth you would give Alternative to
? nifedipine

Isradipine

696-What the most common part that get (fractured or distorted ??)
? in RPD

?? , Connectors , base

CLASP ARM

697- Patient came after 1 week with Fractured occlusal rest due to

Inadequate CR

The rest extended to the central fossa

Overextended preparation to the marginal ridge

698-There is high point b/w upper distal inclination and lower mesial
?inclination premolars ?? Whats the type of occlusal interference

Working side

Centric

Lateral

Protrusive interference

699-Patient has thin gingival biotype and healthy gingiva, what should be considered before
? impression

Crown lengthening

Laster diode for retraction

Gingival graft

711 After OD rest the Patient cannot inter the floss inter-proximally, and the floss get teared

Recountor the proximal contact


701-direct provisional crown what could cause discoloration:

Bynzoyel perioxide

702-Patient will have Ant. implant the dentist did the clinical examination , impression and
made the cast

Whats left?

Surgical splint

713-The assistant placed the alginate impression in contaminated ?? What will happen

The cast will be chalky

The impression tearing

714- Patient vomit on the floor, what to clean the floor by

Low , high , housekeeping

710- The nurse is vaccinated against HvB and got injured what to do

Test Check her antibodies

706-Patient was infected with tb and he is under treatment, the result test not yet delivered,
? what to do

Face mask

Respiratory mask N95

Face mask and ventilation

717- Patient with semicircular radiolucency above the upper premolar roots 1.5cm

Normal .anatomy

Apical Cyst

Apical scar
?

708-Mechanism of action metronidazole


It inhibits nucleic acid synthesis by disrupting the DNA of microbial cells

709-Patient under chemotherapy after the first cycle was in acute pain and he is taking
:prophylaxis antibiotic 18 days ago on the day of tx the pain is gone

Proceed with planned Tx

wait until the last cycle

710-Patient under chemotherapy have acute pain and the tooth is not restorable

Crown resection and endo the root

Extract

Analgesic & antibiotic

700- Pregnant lady in the 7th week in acute pain

Give analgesic

Take Panorama

702-Cervical enamel projection


It may be concluded that CEP occurs more frequently in ma ndibular molars

713-Calculus is

Etiological factor

Contributing factor

Risk factor

704- Chemotherapy commonly cause

Mucucitis

Dermatitis
Osteonecrosis

700- Andrew occlusal concept

Mesial inclination of the teeth

Occlusal is flat and

Lingual inclination

706- Collagen in the pulp during development

Collagen 1,2,4,6 type 1,3,4,5,6

707- Swelling of parotid gland in diabetic patient shows

Granulomatous , Acinar Hypertrophy

708-White lesion , no pain how long do you wait to take biopsy:

2 weeks

700-How long it takes to develop traumatic ulcer ?

1-2 3-4 5-6 7-8 dayes 2 days

720-Pic of Swelling on the angle of the mouth, angle of the mouth is not palpable, whats the
? inferior border of that space
??
?? Digastric muscle , platysma muscle

721-Patient in severe pain , cannot open his mouth ....


? has buccal white lesion what to do

Intralesional corticosteroid

722 Biopsy should be placed in 10% formalin

723- Partially erupted molar has caries in the fissure what to do

Restore with composite

Restore with GIC


Wait until fully erupted and place composite

Wait until fully erupted GIC fissure sealant

724-Pic of double papilla flab ? What is this flab

725-Measuring the posterior palatal seal ?? And the Surgeon ask the patient to close his
nose and blow ..?? Why ?

Valsalva Maneuver

726- To identify the glandular opening

Lateral .. pps

Medial .. pps

??

727-Zinc oxide impression + water or alcohol

Speed the setting time

728-brown tooth discoloration appear yellow with violet light

Dentinogenesis imperfecta

Fluorosis

Amelogenesis imperfecta NOT SURE

Tetracycline
. stain

729-Supraeruption 2.5mm what to do


ORTHODONTIC INTRUSION

730-Chin cup ortho


Mandibular prognathism without maxillary problem

731-Avulsion #11 patient come after 2 days the tooth is dry kept in blastic
bag:
✅Rct outside then replantation
Replace and implant

732-How does backflow occur when using a saliva ejector?


‫االجوبه كانت بين ان قوة الضغط في الفم كانت اقوى او اضعف من قوة السكشن‬
(‫)بسبب انو ضغط الفم اقل من السكشين‬

733-HIV associated ‫مع اي نوع من الهيبتايتس‬


Most common HCV

734-Missing upper 14,24 and molars how to restore?

572-‫ كيف تحدد مكان االمباكتد كناين‬:


PA with different angle
Pan
✅Cbct

736-Brushing teq for ortho:


Charter's teq

737-Straight access in endi by:


GG
Barbed broach
...

738-All metal crown finish line is:


✅Deep chamfer
Shoulder

739-softens gutta percha by:


chloroform

547-‫قوة ضخ المويه فالعياده‬:


277 ‫✅اقل من‬
1000-1500
1500-2000

741- Healing cuff in 2 stages of implant


Healing abutment from 2-5 weeks
742- Warthin’s tumor most common involved S.G
In parotid gland , Tx: parital parotidectomy
742- Depth of class II composite restoration in dentin
2mm
743- Distance between dental implant and inferior alveolar nerve block
2mm
744- IANB piercing which muscle
Buccinator
745- Caliper to measure metal thickness in FPD:
Iwanson caliper
746- Facial space infection that cause truisms
Pterygomandibular spcae, infratemporal , submassetric
747-Butterfly redness face
Malar rash , with lupus
748-The most important factor for prescribing fluoride for a child:
age
749- The most traumatic force from RPD to surrounding abutments:
horizontal force or vertical or both
Horizonral
750-Pain killer during pregnancy may cause fetus deformation
Opoied such as codien
751-Liver disease patient and amount of fluoride
Less than 2 ppm
752-Finish lines width (shoulder and chamfer)
1-1.5 shoulder
0.5 chamfer
753- Role of resonance frequency analysis (RFA):
determine stability (the level of osseointegration) in dental implant

754- The intermediate part between fixture and crown:


abutment
755-Different interpretation of transillumination in crazing and crack tooth
(which one block the light)
If the tooth is cracked, the light will be blocked, allowing only a segment of the
tooth structure to light up; if the tooth only has a craze line, the entire tooth
structure will lightup

756- The sign of bubbles during FPD try-in


Increase the space for cement

757- Maintains/follow up appointments after implant placement for a patient


with high risk of periodontists:
every 3 months or 6 or ...
3 months

528 ‫سؤال اذا فتحت فالب وكان نظيف وش الجواب‬


Clean contaminated
527 Transcemic acid
( intramuscular or subcutaneous )
IV or mouthwashor im
567-after chemotherapy
Necrosis or mucositis
Mucositis
760 .mandible protrousion
Bitlateral sagital spit osteomy
BSSO
761. Internal resorption how to obturate
Thermoplastizine
762. Working time for ah sealer
Working time 4 hours, Setting time 8 hours Working time 4-8 h , setting time 12-48 h
763. Splinting of less than 7 mm intrusion
4 weeks
764. Pemphigus ( ‫ تحط ب‬saline or formalin)
Saline
765 .end stage renal ‫وش اعراضه نسيتها حطيت‬
Petecheia on mouth
766.lesion in genital and perianal what is the test (kawaski .. pathergy ..
imunofluorsence (
Pathergy 765, Nausea
Vomiting
Loss of appetite
Fatigue and weakness
767-First choice analgesic in endo ? Sleep problems
Ibuprofen Changes in how much you urinate
Decreased mental sharpness
Muscle twitches and cramps
Swelling of feet and ankles
768-Medication used in anaxious asthma pt. Persistent itching
Benzodiazepine Chest pain, if fluid builds up around the lining of the heart
Shortness of breath, if fluid builds up in the lungs
High blood pressure (hypertension) that's difficult to control
769- Sympathatic : constric pupil or blood vessles ?
Blood vessles

770-Blood transfuion or extract without epinephrine?


blood transfusion
give adrenaline ( 1:1000 solution ) 0.01 mg/kg ، ‫ممكن يجي السؤال ايش نعطيه انتي بيوتيك ف الجواب‬

557-‫ وش السبب‬bus ‫ مريض خلعت له وجاك بعد‬٢ ‫اسابيع ب‬


subperiosteal abcsess

subperiosteal abscess , its came usually after surgical extraction of mandibular third
molar . May occur several weeks after .

555-‫جاك مريض جته ضربه ع اليسار وش يصير‬


Fracture of left body and right condyle

773- zometa ‫مريض ياخذ‬


U did extraction and it is not healing 4 weeks

‫ وش تسوي‬Surgical depridment

774-Give systemic fluoride depend on ?


Age
775- Contamination of impression .. cause
tearing marginal tearing
776- Feature of elastomeric impression is

777- Sinus left what is the flap will be used with it


Lateral window sinus elevation(direct)/ This approach provides access to the
lateral sinus wall by raising a full-thickness mucoperiosteal flap from the
alveolar crest with vertical releasing incisions. (indirect)/ at the angulation and
position of planed implant bone of sinus floor pushed upward
778 - 7277-7777 277 ‫ضغط الماء في العيادة اقل من‬-
!!!!!

779- When necrosis happaned in ANUg ?


caused by synergism between the Fusobacterium and Borrelia vincentii (an
oral spirochete) in a systemically stressed or severely debilitated
patient, ANUG occurs as an acute liquefaction necrosis of the interdental
papilla ANUG is an opportunistic infection that occurs on a background of
impaired local or systemic host defenses. The predisposing factors
for ANUG are smoking, psychological stress, malnutrition, and
immunosuppression
780- Impression can not poured for 24 hour is
polysulphide Polysulfide and condensation silicone impressions should
be poured up to 30 minutes. Polyether impressions can be poured between
30 minutes and 24 hours. PVS impressions can be poured between 30
minutes and 4 weeks.
781- In excessive occlusal wear .. we should re establish Maxmium
intercuspion or centric
Centric relation
782- Buccal lingual placement of crown is
dynamic seating
783- Solution for cleaning waterline ( chlorohexidene or hydrogen peroxide (
Hydrogen Peroxide
784- More bacteria in Ongoing waterline Ingoing water line Stagnant line
Stagnant line
785- Bacteria in odontgenic abcess I put erobes and anerobes
Mixed is correct anerobic
786- To ensure immunity after 3 dose of hbv vaccination range should be 9
10- 12 I forget it anti-HBs level ⩾10 mIU/mL

787- FPD success rate 10 to 15 90 and 72 most close answer 87% 10-year
survival rate for FPDs and a 69% 15-year survival rate
788- Furaction involovment only there is bone in apical one third What is
treatment
Tunnel approach
Root resection
Furcation plasty
Root resection
789- 6-8 probing depth what is treatment
Scaling and root planing
Open flap debridement
MWF indicated for > Shallow to moderate pocket depth with pocket depths 4-
6 mm (Moderate periodontitis)
790- Minimal intrusion .. what is the treatment.
Observe for re eruption correct
791- Differences between primary and permant teeth
792- Pit in enamel , attrition in dentin .. obliteration in the pulp what is the
disease
dentinogenesis imperfecta maybe
793- Gingival margin of complete denture high in central and canine or high in
lateral
central and canine
794- Denture come less retentive over implant ..
non parallel implant correct
795- Denture become loose what is the cause Hyperplasia syndorme ??
The most common reason for loose-fitting dentures is a process known as
bone resorption,
796- Newton syndrome ??
Capnocytophaga canimorsus is a facultative Gram-negative bacillus that is
typically a constituent of the oral flora of dogs and cats. It was first isolated by
Bobo and Newton in 1976 from a man presenting with meningitis following a
dog bite.
797- Maccot syndrome??
The most characteristic feature of the syndrome is the excessive faith in
medical technology, particularly imaging. Other components that might also
be present are the absence of clinical reasoning and of establishing emotional
links with sick people.
798- White line did not rub off
White spongy nevus correct
799- 3 line in the probe:
Michigen probe
Marques
800- Florda disadvantages
Underestimation of pocket depth
Lack of tactile sensation
Expensive
801- Inital stage of gingivitis
Interlukin 1
Prostaglandin
802- Which clasp causes more stress
I bar
Ring
Circumferential
Circumferential (aker) Others are stress releasing clasps

803- Ortho plane growth cessate last


>>>Vertical
804- Infection obliterate nasolabial fold
>>>Canine space
805- Eye redness after canine space infection
>>>Cavernous sinus thrombosis

806-GTR & GBR difference

807-New name for biological width


>>>The term biologic width was replaced by supracrestal attached tissues
808- Remaining dentin thickness for core build up ??
>>> 1mm thickness
hight> 2mm ferrule
809- Amalgam pins
>> 2mm in dentin 2mm hight
810-Histo. of sjogren's syndrome
Lymphocutic infilteration
811-the most common site of pemphigus vulgaris Hard palate/soft
palate/buccal mucosa..
>>> Lesions may occur anywhere on the oral mucosa, but the buccal
mucosa is the most commonly affected site followed by involvement of
the palatal, lingual and labial mucosa. Gingiva is the least commonly
affected site and desquamative gingivitis is the commonest manifestation
of the disease when gingival is involved
812-fissured tongue is a feature of? Lupus/erythema multiform…
>>>The exact cause of fissured tongue isn't known. However, researchers
believe it may occur as a result of an underlying syndrome or condition,
such as malnutrition, infection, or Down syndrome. Fissured tongue is
seen in Melkersson-Rosenthal syndrome (along with facial nerve
paralysis and granulomatous cheilitis). It is also seen in most patients
with Down syndrome, in association with geographic tongue, in patients
with oral manifestations of
psoriasis, and in healthy individuals. Fissured tongue is also sometimes a
feature of Cowden's syndrome.
813-how to diagnose asthma Inspiratory../expiratory../pulse rate...
>>> peak expiratory flow
814-bacteria with infective endocarditis
>>> Staphylococcus aureus followed by Streptococci of the viridans
815-picture* of modified pen grasp
816-picture* type of support during scaling
817-how much mg of xylocaine in 1 ml as xylocaine 2%
>>> 20mg
818-warthins tumor involved Submandibular gland/parotid/sublingual…
>>> parotid
819-main of gutta percha is Zinc oxide/plasticizer/gutta percha…
>>> zinc oxide
820-pt with multilocular lesion managed by Curretage/resection…
>>>resection
821-solution for cleaning waterline Saline/hydrogen
peroxide/cholorohexidine
>>>hydrogens peroxide
822-during preparation of class V pulp exposure occurred what to choose
before restoration GI/calcium hydroxide…… ‫مو متأكده لو السؤال كان انو حصل‬
‫😅 بلب اكسبوجر وال بس كان قريب‬
>>> calcium hydroxide indirect capping
823-the most important during shade selection is
Chroma/value/saturation/hue
>>> value
824-when to choose composite shade After prep./before application of
rubber dam….
>>> At the beginning, before application rubber dam
825-extract tooth how to manage before pre-clinical use Autoclave for 50
min/stored in saline
>>> If it is with amalgam : put in formaline 10% for 2 weeks If it's
without amalgam : heat sterilization for 40 min
826-mechanism of action metronidazole
>>> inhibit nucleic acid
827- types of GI cements and its uses
>>> mentioned above
828- radiograph pic* about internal resorption
829- what is rebound phenomena
>>> is the emergence or re-emergence of symptoms that were either
absent or controlled while taking a medication, but appear when that
same medication is discontinued, or reduced in dosage. Reactive
hyperemia or rebound phenomen : occur due to localized tissue hypoxia
and acidosis caused by prolonged vasoconstriction.
830- osteo dystrophy accompanied with Liver
disease/cardiac/pulmonary/renal disese
>>> renal
831-cardiac drugs cause gingival hyperplasia Statin/beta blocker/calcium
channel/ACE inhibitor
>>> calciuma channel
832- definition of veracity
>>> Veracity is the principle of truth telling, and it is grounded in respect
for persons and the concept of autonomy. In order for a person to make
fully rational choices, he or she must have the information relevant to his
or her decision
833-acute sign after chemotherapy
>>> Nausa and vomiting mucositis
834-clinical sign of bulimic nervosa
>>>Palatal erosion
835-color of k-file size 90
>>> white
836-FL in water
>>>1ppm
837-how to differentiate between acute apical abscess and acute
periodontal abscess Vitality test/radiographic…
>>> vitality test
838-Best light for color maching:
Florcent light ‫بحثت فيه اكثر من مكان حاطني الريكومندد فلورسنت بس منب متاكد‬
Incandesent light
839‫جا†كمان†جنجفا†االتاشد†حقت†الماتريال†سؤال‬
Shiller’s potassium iodide
840-Cranial base growth?
Structure development
INTRAMEMBROUNCE
ENDOCANDERIA
Endochondral ( syncondrosis )
841- Which of the following is common skeletal anamolly with cleft lip
and palata!
Mx deficency
Mand. Deficwncy
Max excess
Mand excess

842-asymptomatic white legion for nonsmoker?


Leukoplakia
843-fracure of rest in RPD is due to
Crosses marginal ridge

844-Which antibiotic for spirochets


doxycycline,
tetracycline hydrochloride, ‫طلع كل الخيارات انها تنفع بس األغلب‬
Doxycycline
metronidazole

845-Distancse between marginal gingiva to floor of the mouth :


4/6/7/8
8mm

846-Endodontic re vascularatin –
using tri antibiotics

847-poly vinyl impressions disadvantage Disadvantages: Hydrophobic,


Hydrogen gas release, inhibition of setting,
latex sensitivity retards its reaction

848-If a violete color is added on a crown with a yellow chroma what will
be the result:
- higher chroma lower value

849-the autoclave B for


21 days

850-gingival enlargement from nifedipine, what drug can you give the
patient instead: Answers were all calcium channel blocker drugs, except
one which is
the correct answer Isradipine

851-Obstructive sleep apnea sleep study result


5 event per hour
‫ على حسب الن السؤال موب كامل‬:
10 None/Minimal: AHI < 5 per hour
15 Mild: AHI ≥ 5, but < 15 per hour
20 Moderate: AHI ≥ 15, but < 30 per hour
Severe: AHI ≥ 30 per hour

852-Implant complete overdenture for lower which is the most commonly


used attachment?
Bar
Magnet
Clasp
Locator

853-Alginate impression how you take It to the lab


plastic bag
biohazard

854-Fiber responsible for sharp pain


A delta fibers
b delta
c
855-Varnish under amalgam
Copalite

856-Wide canal obturation


Warm vertical condensation
857. What is the trade name of the 5% varnish?
Duraphat
858. During RPD delivery, the rest were smaller than the rest seat itself,
what is your management?
Apply flowable composite
Redo
859.During FPD try in, you notice that the framework is rocking, what is
your management?
Pickup impression
860.60 years old patient with cervical caries and recession, what is your
Diagnosis?
Root caries
861.The principle of truth telling?
Veracity
862.When the dentist shows the record of the patient to others, he breaks
which rule?
Confidentiality
Privacy
Autonomy
863.When the dentist has the right to disclose information about the
patient?
If it is the public right to know
During consultation of another dentist
864.Which of the following is morally right?
You can seek help from another dentist when needed ??
You can get you patients phone number for the record for social activities
Book appointments for family and friend for them to get fast treatment
865.Patient is getting complete denture what is the best relation to set the
maxilla?
Centric relation
Centric occlusion
Maximum intercuspation
866.The purpose of the celluloid casting ring liner during investment
procedure?
To ensure uniform expansion
To allow easy removal
867.The lower complete denture was set so that it extends about 1\3 of
the retromolar pad, what is your management?
it should be 2\3 it’s fine
it should cover the whole retromolar pad
868.What is the most important landmark during aesthetic setting of
maxillary anterior teeth in complete denture?
Nose bridge
Median raphe
Incisive papilla
Anterior alveolar ridge
869.What is the most important landmark during complete denture set up
for occlusal plain record?
Retromolar pad
Incisive papilla
Median raphe Maxillary incisors
870.Patient reported to the dental clinic with pain in TMJ, his waif
mentioned that he put pressure in his teeth while sleeping. What is this
parafunctional habit called? Q870:
Bruxism is defined as the dynamic grinding of teeth.
Bruxism Clenching is the static contact of teeth together as power from
Clenching the closing muscles of the jaw is applied.
871.Patient came with internal stain in multiple teeth, indirect veneers
were the proposed treatment. Which type of indirect veneer will be used?
Alumina
872.Patient came to the clinic which all ceramic crown in his anterior
teeth, he complains of porcelain chipping. Upon examination, there was
porcelain fracture half of the incisal edge. What is your management?
Apply composite
Polishing
Remove and
correct in the lab Remake
873.Patient with atrophic tongue papilla and hypo chromatic RBC, what
is your diagnosis?
Iron deficiency anemia
Vitamin B12 anemia
874.Test used to examine the presence of pulp blood flow?

875.Hemophilia test?
PTT
PT
Bleeding time
876.Aspirin test?
Bleeding time
PTT
PT
877.What is the treatment of paroxysmal hemicrania and it is diagnostic
for it?
Indomethacin
878.Which of the following drugs you use when you need less salivation
during impression taking?
Anticholinergic (Atropine)
879.Which of the following drug can cause dry mouth?
Anti-malaria
Tricyclic antidepressant
880.Which of the following drugs used to treat dry mouth?
Prilocaine
881.Which of the following can cause sever elevation in blood pressure
when given with epinephrine?
ACE inhibitors Calcium channel blockers Diuretics

882.Patient have a tooth with post and core and crown, he complains of
sever pain on biting, what is your diagnosis?
Verical root fracture
Hyperocclusion from eccentric movement
883.Patient came to the clinic with asymptomatic broken tooth. upon
medical history the patient has HIV. What is your management?
Wear double gloves and mask
Refer to infectious disease specialist Defer the treatment
Request CD4 count and viral load
884.During placement of Tofflemire band, the dentist injured himself,
what is the appropriate management?
Remove gloves and let bleed freely
885.Dentist is infection free, he injured his figure and bleed inside patient
mouth, what is the management?
Assure the patient and both should go to the infection control center for
follow up
886.X-ray of medially tilted tooth #47, what is the most probable
consequence?
Food impaction
Sever periodontal destruction Furcation involvement
887.Which type of bone have the least success rate during implant
placement?
Type I
Type II
Type III
Type IV
888.What is the direction of periodontal ligament surrounding the
implant?
Parallel to the implant
889.In ANUG, the necrosis which extent to the gingival margin in which
stage?
Stage 1
Stage 2
Stage 3
Stage 4
890.During treatment, the dentist was careful and trying not to touch the
lip, what is the most probable diagnosis?
Herpetic gingivostomatitis
891.Picture of a dentist doing scaling and his figure is under the lip, why?
Lip retraction
892.What is the most type of biopsy incision helps with primary closure
of the wound?
Circular
Elliptical
Triangle
Trapezoid
893.What is the access cavity shape of the upper first premolar with 3
canals?
Ovoid
Triangle
Trapezoid
894.What is the most probable consequence after using ferric sulfate in
pulpotomy?
Internal resorption
External resorption
895.Patient came with impacted canine, the father refuses to extract.
What will be the most probable consequence?
Resorption of the lateral incisor
896.Traditional serial extraction?
Primary canine then primary first molar only
Primary canine then primary first molar and
primary second molar
Primary canine then primary first molar and permenant first premolar
897.What to extract in case of Class II malocclusion?
Upper first premolar
898.What to use in case of mandible retrognathism in class II patient?
Headgear
Facemask
Functional appliance
899.Class I malocclusion accompanies which of the following?
Teeth misalignment
900.What to extract in case of Class II malocclusion?
upper first premolar
901.Patient with Class II division II, what the overbite will be?
Steep
902.In we have sever class II, what will happen to the ANB?
Increase
Decrees
903.10 years old patient came with slight space between the incisors what
will be your management?
Nothing
904.What will be the effect of smoking in the periodontal status of the
patient?
Immunity against the periodontal pathogen
Decrease wound infection
Less blood flow and inflammatory response
Decrease colonization of periodontal pathogens
905.What is the most important factor to take into consideration while
taking a PVS impression?
Moist control
906.Flappy ridge impression?
Mucostatic impression
907.Probe that is missing 4 and 6 mm readings?
Williams
908.Probe with 3m6m9 markings?
Marquis
909.Patient vomit in the floor, which type of disinfection?
High
Intermediate
Low
910.Heavy continuous forces will result in?
Undermining resorption
911.Patient with thin gingival biotype and will go through orthodontic
treatment, when do you need to do gingival graft?
When the tooth is planned to be moved lingually
When the tooth is planned to be moved labially
When there is bleeding on probing
When there is deep pocket
912.Patient have posterior teeth with amalgam, however the amalgam is
looking like island. What is the most probable cause?
Erosion
Attrition
Abrasion
913.Child patient with no water fluoridation what is the best method to
get fluoride and decrease caries?
8% stannous fluoride
Varnish twice a year
Daily brushing with fluoridated toothpaste
APF gel
914.What is the best instrument to clean a FDP?
Super floss
Dental floss
915.Cell of the initial gingivitis stage:
Neutrophils
916.Which stage of gingivitis develop after 2-3 weeks?
Established stage
917.The most common feature in concussion?
Tender to percussion
918.Tooth displaced may be mesially, laterally, labially and palatally?
Concussion
Extrusive luxation
Lateral luxation
Intrusion
919.Hemophilia patient came with symptomatic unrestorable tooth, what
is the best management?
Extraction
Remove the coronal part and do RCT for the roots
920.Ethical principles?
Autonomy- competence-profitability
921.Follow up after complete denture delivery?
Every 6 months
Every 3 months is necessary
Every week
922.Osteointegration time?
3 months
6 months
9 months
923.Osteointegration other name?
Primary stability
Secondary stability
924.Patient came after scaling in the maintenance appointment
complaining of generalized bleeding on probing and there were localized
areas of deep pockets. What is your management?
Polishing of all the teeth and deep scaling of areas of deepened pocket
Polishing of the areas of deepened pockets and generalized scaling

925.Patient came after doing RCT complaining of persisting pain spite


the use of ibuprofen. What is your management?
Replace ibuprofen with acetaminophen
Give acetaminophen with ibuprofen
926.What instrument is used to remove unsupported enamel?
Gingival marginal trimmer
927.What instrument is used in push motion?
Chesil
928.What instrument to use to measure the thickness of the metal crown?
Iwanson caliper
929.Tooth with grade II furcation, what is the treatment?
Furcation tunnel approach Extraction Root resection
930.Depth of amalgam self-threaded pin?
2 mm
931.amount of non-functional cusp reduction in PFM crown:
1.2
2
932.which of the following provide retention of the restoration?
Pin Grooves
933.picture of an orthodontic band pusher?
Final seating by the patient
Final seating by the dentist
Initial seating by the dentist
initial seating by the patient
934.osteodystrophy in which disease?
Renal
Liver
935.side effect of prolonged Listerine mouthwash use?
Lichenoid reaction
Erythema Multiforme
Epithelium desquamation
936.Andrew’s 6 keys of normal occlusion depend on what?
Maxillary first molar
937.Patient have sealant on the top of incipient caries, what is the status
of the caries progression?
Decrease
Increase
No effect
938.Bone loss in secondary trauma?
40-50%
939.Implant screw has fractured, how can you remove the part of the
screw in the fixture?
Ultrasonic tip
Hemostat
940.Instrument fracture in the coronal part during RCT, what is your
management?
Open flap
Retrieve it
Extraction
941.Which of the following is a nickel-titanium rotary system?
Protaper
942.Which of the following material is the best and most effective for
dissolving Guetta percha during retreatment?
Chloroform
943.Which is the instrument used to retrieve Guetta percha?
H file
K file
Peaso reamer
944.What instrument is used to detect crown and root fracture?
DG-16 explorer
Tooth sloth

945-Most cost effective modality for caries prevention?


Fluoride gel
Fluoride rinses
Fluoride varnish
Fluoride supplements

946-The increase in deflection between an FPD with 1 pontic and an FPD


with 3 pointics is:
3 times
9 times
12 times
27 times

047-Hospitalization fluoride dosage


4 mg
5 mg
6 mg
7 mg

948-The healthy implant probing depth with no bleeding on probing is:


1mm *
2mm *
3mm *
4mm *

949-Mechanism of action for Metronidazole:


a. Inhibits DNA gyrase
b. Inhibits cell wall synthesis
c. Inhibits mRNA
d. Interferes with DNA and disturbs its function
950-Which of the following medications increase the effect of local
anesthesia:

a. Beta-blockers✅✅
b. Calcium channel blockers

951-Patient with multiple ulcers in the mouth and oral mucosa and
genital area: the test to confirm diagnosis is:
a. Pathergy
b. Kawasaki
c. Schirmer
d. Immunofluorescence

952. Herpangina is associated with which virus:


a. Coxsackievirus
b. Herpes Simplex Virus

953-A patient came complaining of frequently fractured acrylic base for


upper partial denture, the prosthodontist decided to use cast metal
mesh to reinforce the acrylic. The minimum thickness of the metal mesh
in mm is:
a. 0.1
b. 0.2
c. 0.3
d. 0.4
954. (PICTURE: painless slow growing large mass in posterior teeth, the
mass was not ulcerated and irregular in shape) Patient is 26 years old
pregnant with swelling:
a. Pregnancy tumor

955. (PICTURE: brownish-blackish circular discoloration on labial gingiva


between #33 and #32) The lesion is not symptomatic, Treatment will be?
a. Excisional biopsy
b. Incisional biopsy
c. Remove the cause
d. Observation ← (Looks like melanotic macule, just observe or excision
if patient complains about esthetics)

956. Patient came complaining of pitted enamel with yellowish and


brownish discoloration, patient said his teeth are getting smaller. Upon
examination the root chambers is obliterated and enamel chipping in
some teeth.
a. Fluorosis
b. Enamel hypoplasia
c. Dentinogenesis imperfecta
d. Another dentine related condition

957-#31.41.42 missing, best option to restore?


a. 5 units FPD #32-43
b. 6 units FPD #33-43
c. 6 units FPD after extraction of #32 (#33-43)
d. 6 units resin bonded #33-43 (if he said most conservative)
: 17. #14.15
008-missing best option to restore?
a. 4 units FPD 13-16
b. 5 units FPD 12-13-16
c. 5 units FPD 13-16-17
d. 6 units FPD 12-13-16-17

959-#35,45 missing, dentist decided to restore with conventional RPD


#$@! !!! ... anyways they’re asking about which lever class will be in this
case:
a. 1
b. 2
c. 3
d. 4

960-Only #21 extracted, patient wants temporary rpd, what Kennedy


class is this?
a. 1
b. 2
c. 3
d. 4

961-Patient with lower bilateral distal ext. RPD and upper natural
dentition can't tear food, denture is stable and not rocking and was done
5 years ago.
a. Reline
b. Rebase
c. Remake
d. Re-establish occlusion

962-Best management for dehydration:


a. Water
b. sport drink
c. energy drink
d. Some other stupid drink

963-Bisphosphonate patient requires 3 extractions 2 upper right and 1


lower right:
a. Extract all in 1 visit
b. Wait until symptomatic then extract
c. Extract 2 then wait 2 months and extract 1
d. Extract 1 each 2 weeks

964-Mild hemophilia A needs endo:


a. Do endo.

965-Hemophilia B which factor:

a. 9 ✅✅ ix
b. 99
c. 999
d. 8
966-Patient with end-stage renal disease:

a. Petechiae in oral mucosa✅✅


b. Bone deposition in jaw bone

967- Patient with parotid enlargement soft no symptoms and bilateral:


histology will show:
a. Acinar atrophy
b. Acinar hypertrophy
c. Macrophages
d. Lymphocytic infiltrate

968-Patient feels pain when pressure is applied against chin while


protruding mandible:
a. Stylomandibular ligament
b. Retrodiscal tissues
c. Inferior lateral pterygoid
d. Medial pterygoid

969-Frankfort plane in ortho tracing between which two points?


a. Sella - nasion
b. Porion - Orbitale

970-Patient came with protruding ortho wire that is causing


ulcerations of the cheek, lip and tongue -- what is the best
management:

a. Trim the wire

b. Cover wire with relieving wax


c. Push the wire with pencil’s eraser

d. Analgesics

971-There were two questions about patients coming back 2 days


after Scaling and root planing for periodontitis. Both cases had
teeth sensitivity and ulcerations of the marginal gingiva or buccal
mucosa...

a. Antibiotic (topical)

b. Topical steroids

c. Redo scaling and root planing

d. Analgesics and reassurance✅

972. We’re going to place an implant in a space that is 12mm away


from the maxillary sinus. What’s the maximum implant length that
can be used?

a. 8 b. 10 c. 11✅✅ d. 12

973. #34, 35, 36 are missing.. 37 is slightly mesially tilted. The total
remaining space is 14mm, what’s the maximum number of 4mm
diameter implants that can be placed to restore the 3 missing
teeth?

a. 1 b. 2✅✅ c. 3 d. 4

974. Dentist decided to go for 2 stages implant placement. Which


metallic part is used to cover the implant that is then submerged
under soft tissue?

a. Healing abutment

b. Impression coping

c. Cover screw✅✅
d. Abutment

975-In primary molars, the most common site where pathological


radiolucencies are observed:

a. Interradicular bone✅✅

b. Root apices

976. Possible complication for usage of CaOH2 during pulpotomy


procedures:
a. Internal resorption✅✅

b. External resorption

c. Radicular pulp calcification

977. Pulpotomy with ferric sulfate (Same question as the previous


one) I think the answers were exactly the same too:
a. Internal resorption✅✅

b. External resorption

c. Radicular pulp calcification

978-Primary incisor with a history of trauma (I forgot if they


specified which type of trauma 6 months ago, started turning grey
in color. Radiographically the tooth doesn't show any pathologies;
what’s your management?
a. Observation✅✅

b. Extraction

c. Endo treatment

979.How to measure gingival width


a. Probe✅✅

b. Explorer

c. Nabers

d. Clgjdsadweaiuaqwjehdz calipers .. ← I chose this cuz... u know .

980. Fractured labial ceramic veneer PFM of lower anteriors, on the


opposing there is PFM too, with deep overbite.

a. Heavy forces with mandibular ✅✅protrusion causes the fracture


b. Weak ceramic-metal bond

c. Inadequate preparation thickness

981. The preparation for a full ceramic crown includes:

a. Facial reduction of 1.4 mm in 2 planes✅✅


b. Facial reduction of 2 mm in 2 planes

c. Facial reduction of 0.5 mm in 2 planes

d. Reductio

2 planes incisal 1.5 to 2 mm & gingival 1.2 to 1.4 mm


982. Patient with large distal caries and mesial composite
restoration with recurrent caries wants an esthetic restoration,
when the caries and old restorations are removed there was little
remaining tooth structure:

a. Full ceramic crown✅✅


b. Composite restoration

983. Premolar grade 2 furc and grade II mobility

a. Tunnel preparation

b. Furcation
. plasty

x c. Root resection✅✅
d. Extraction

984-Prognosis of maxillary molar grade 1 furcation and very well


maintained oral hygiene and patient compliance:
a. Good✅✅

b. Poor

c. Hopeless

d. Normal

985. Which of the following configurations is the best in perio


prognosis:

a. Short trunk convergent roots

b. Short trunk divergent roots

c. Long trunk convergent roots

d. Long trunk divergent root


986. Axial taper for prep:

a. 12 degrees

b. 10 degrees

c. 6 degrees✅✅

d. 3 degrees

987-Short clinical crown, prepared for full crown coverage, how to


get more retention:

a. Full shoulder margin

b. Axial grooves✅✅

988. Nabers probe (pic)

989. Band pusher (pic) → intial seating by dentist

990. Cheek biting:

a. Occlusal plane too high


b. Not enough horizontal overlap✅✅

c. Use of anatomical crowns


991. Patient feels pain when stressed behind the eyes, he’s
clenching his teeth too:

a. Migrane

b. Cluster headache

c. Trigimenal neauralgia
d. Dunno.

992. Too much listerine could cause:

a. Lupus

b. Epithelial desquamation
c. 2 more conditions I’ve never heard of.

993. GIC luting is which type of GIC:

a.I ✅ b. II c. III d. IV

994-Deep class 5 liner:

GIC a.

Calcium Hydroxide✅✅✅ b.

Resin modified GI c.

995-EDTA name and conc → Ethylenediaminetetraacetic


Acid 17%

996-Which kennedy class for a patient missing only #21 →


Class III

997-Primary stress bearing area for lower denture? → Buccal


shelf

998-Tooth with large amalgam extracted. In what type of


labeled container:

Biohazard✅✅ a.
Medical waste b.

Incineration c.

999-Tooth with large amalgam and enamel fracture line →

bite test

1000-Muscle + denture lingual flange = mylohyoid

1001-Patient came with swelling but cant do complete endo


treatment, what’s the best emergency management?

Cleaning and shaping a.

Incision and drainage✅✅ b.

Antibiotic c.
Extraction d.

1002-2mm remaining Gp after post prep, what should the


prostho do?

Refer for endo to refill✅✅ a.


Proceed b.

Wait and evaluate c.


Use cast post instead d.

1003-Pic of CT scan of the face, there was a fracture of the


floor of the orbit. What’s the most probable complication:
Disturbed occlusion a.
Double vision✅✅ b.

2 other options. c.

1004-A pic of lower mandibular fracture (mid mandible:


symphysial), best management?

2 plates✅✅ a.

3 plates b.
1 lower border plate c.

1 upper border plate d.

1005-File notes should be (Specific)

1006-Veracity (Truthfulness)

1007-Autonomy (Patient has the right to make their own


decision)

1008-How to tell pt. About bad news

Stop frequently to make sure they understood a.

Try to minimize the severity of the situation b.

1009-Overprotective parents → make dentist uncomfortable


if the question about dentist make him uncomfortable if it’s about child
make the behavior worse
1010-Curette to tooth angle → 45-90
1011-FPD success rate 10 years and 15 years → 92-74
66%

1012-Dentigerous Cyst treatment:

Excision of the cyst a.

Extraction of the tooth b.


Excision of the cyst and the tooth********* c.

Observation d.

1013-Exaggerated gingival inflammation can be seen in:

Pregnancy******* a.
Hypertension b.

Epilyptic c.

Hyperlipidemia d.

1014-Cementum enamel interface, which is most common?


Gap a.
Butt-joint b.

Cementum overlaps enamel **** c.

Enamel overlaps cementum d.

1015- At which age is the completion of 3rd molar crown?


11 a.

13 b.

16 c.

19 d.
1016- A mount of GP in GP → (20%

1011-Cone size 20 = 0.2mm

1018-During cementation, the crown is rocked facially then


lingually, why?

Dynamic seating permits cement flow ***** a.


Static seating permits cement flow b.

Two other options… c.

1019-The minimum prophylaxis in weeks after needlestick


injury with known HIV positive patient is:
2 weeks********** a.

4 weeks b.

6 weeks c.

8 weeks d.

1020-Most common malignant oral tumor?

Squamous cell carcinoma**** a.

1021-Pt did not spit in “spitton” and spilled on chair:


Low disinfection a.

High disinfection b.

Home remedies c.

Follow equipment manufacturer instructions d.


1022-Alginate with moisture contamination:

Impression too grainy a.

Chalky porous casts b.

Impression tears easily c.

Impression something d.

1023-The gingival col is more vulnerable to gingival


inflammation than other places why?

High vascularization a.

Non keratinized✅✅ b.

Harder to clean c.

Not attached to tooth d.

1023-Tb can stay in air → 4 hrs

1024-Cement excess = perio problems

1025- Isolated bacteria from abscess?

Anaerobes a.

Facultative b.
Aerobes c.

Mixture of aerobes and anaerobes******** d.


1026-Mylohyoid-ridge was sharp, the surgeon will do
osteoplasty; possible injury what nerve?

Lingual nerve** a.
1021-Which artery supplies blood to the floor of the mouth?

Lingual artery__******* a.

1028-MB root 1st Molar→ MSA nerve

1029-Pedo molars → MSA + Greater Palatine

1030-IAN terminates to? → mental and incisive

1031-Enamel pearl removal by?


Ultrasonic scaler*** a.

K file b.

explorer c.

1032-At which stage of tooth development abnormalities


like peg-shaped incisors occur?
Morphodifferentiation a.

Histodefferentiation b.
Initiation c.

Proliferation d.

1033-Severe occlusion on teeth that’s enough to push it into


bone?
Necrosis of PDL and bone resorption a.

PDL widening******** b.

Bone deposition c.

1034-Necrosis stages (marginal gingiva → Stage III)

1035-Shape of small excisional biopsy:

Elliptical*** a.

Rectangular b.

Circular c.
Triangula d.

1036-Gp disinfection:

Sodium hypochlorite for 1 minute** a.

1037-Denture wearer for 24hrs a day → Denture stomatiti

1038-Crown fractured with no pulp exposure →


Uncomplicated crown fracture

1039-Lightness or darkness → value

1040-Best light for shade selection:

colour corrected incandescent light a.

colour corrected fluorescent light *** b.


1041-Shade selection time:

before prep*** a.

After prep b.

During prep c.

1042-Safe position of X-ray tech without barrier → 2m


behind tube source

1041-Paget’s disease panoramic appearance → Cotton wool

1044-Safe in asthmatic anxious patients → diazepam

1045-Pt taking chemo blood work was normal platelets


slightly decreased and needs endo:

I think we just proceed with Tx. a.

1046-Least impacted tooth:


Lateral********** a.

Premolar b.
3rd molar c.

Canine d.
1047-Complication of orifice widening with gates-glidden?
→ strip perf

1048-Water spray during prep? → for protection of pulp


from heat generated

1041-When to give prophylactic antibiotics?


History of infective endocarditis********* a.

3 other conditions (Just study them all because 2 more b.


questions were similar)

1050-Which tooth will benefit the most from Fissure sealant?

Maxillary first molar******* a.

Maxillary second molar b.


Maxillary 1 premolar c.

Mandibular 2nd premolar d.

1051-65 years old patient with increased opacity around


the some teeth apices:

Cementum remodeling with age a.

Continuous cementum deposition throughout life******** b.


1052-When trying an FPD one of the retainers could not be
seated on the crown without stress:

Redo the FPD a.


Section then solder in position***** b.

Replace the problematic retainer c.

Even if there’s stress, cement and the tooth will adapt d.

1053-Minimum duration for CaOH2 to work as an intracanal


medicament?

1 hour a.

1 day b.
1 week**** c.
1 month d.

1054-What will happen if you mixed acrylic for temporary


crown with 1:1 monomer to polymer ratio?

No excess monomer will remain a.


polymer to monomer usual ratio is 3:1
More polymerization shrinkage** b. too much monomer = shrinkage ,
too little monomer = granular cured mix
Endothermic heat generation c.

1055-What’s the least esthetic restoration:

Composite a.

Compomer b.

GIC c.

Amalgam****** d.
1056-First thing to check during crown cementation:
Proximal *********** a.
Occlusal b.
Margin c.
Fit d.

1057-Ortho patient that is 8 years old came with his 14 years


old, what’s the first step:
***** Ask for a legal guardian before any steps a.

1058-Worst tooth in terms of perio prognosis:

1st molar maxillary***** a.


1st maxillary premolar b.
1st mandibular premolar c.
1st mandibular mola d.

1059-The thinnest total mean of bone width is found with:


Thin scalloped phenotype**** a.
Thin flat phenotype b.
Thick scalloped phenotype c.
Thick flat phenotype d.

1060-Which of the following expanders is tissue borne:


Haas expander************ a.
1061- what is the lab test to detect if he has a liver disease:

A- blood urea nitrogen

B- Can’t remember

ALT- AST ****** AN aspartate aminotransferase (AST), alanine aminotransferase (ALT) test
is a blood test that checks for liver damage.

1062-Patient had adenoid cystic carcinoma, where is the suspected location of caries appear
in tooth, and what is the management?

A-occlusal caries, extraction

B-cervical caries, extraction

C-occlusal caries, restoration

D-cervical caries, restoration***

4- which one of these disease has the characteristic of Autoantibody serum?

a-lichen planus

b-erythema multiform

5- temperature no. Of endo ice?

-26.2 C ✅

6-Main artery blood supply of the floor of the oral cavity

Lingual artery

8- patient has a brest cancer disease, she did extraction before 2 weeks ago, she come back
with pain and non socking healing, what is the management?

a- curettage

b- marginal resection***

Cant remember

9- x ray result say: multi locular radiolucency in the lower jaw with root resorption, what is
the management?
A-curettage

B-marginal resection ✅

10-What is the nerve supply of the superior alveolar nerve which enervate mesiobucal cusp
of upper first molar?

a-anterior

b-middle******

c-posterior

d- i think inferior

11- endo question, patient complain from his lower molar Xray show: lower molar with
crown and radiolucency in the apex and the lateral distal surface of the root.

Pain in percussion, pocket depth distally: more than 10mm, cold test: negative

a-subgingival calclus

b-lateral periodental abscess

c-asymptomatic periapical periodontitis

13- strawberry gingivitis related to what disease

Wegner’s disease

15- drug used for asthmatic patient? In anxious asthmatic patient: benzodizipine

Asthmatic pt‫الدوا اللي يستخدمه اصال لحالته‬salbutamol or Ventolin

‫ال‬analgesic ‫اللي يستخدمه وآمن معاه‬paracetamol

17-most common organism found in endo abscess?

if chronic answer is mixed

If acute answer is Anaerobes

19- 2 questions about avulsed teeth management, one of the questions the avulsed happen
before 15 min, and the second one before 30 min

Splinting for 2 weeks ******


20-why we do bevel to to gold cast restoration?

For burnishing and adaptation ✅ (and retention)

21-depth of pulpal floor of inlay, how many mm?

2mm (1.5-2)

22- amalgam cll restoration how i choose where to stop the gingival margin?

Caries extension ✅

Thickness remaining

Depth of restoratio

23- amalgam class l was applied, after 2 weeks he came with broken distobucaal cusp, what
is the reason?

A-undermined enamel ✅

B-insufficient depth

C-heavy occlusal force

24-Disadvantage of bleching in the clinic?

A- change restoration color

B- deglaze surface of porcelain (do something to the outer surface of porcelain

C-mercury relase from amalgam

25-Optimum taper of the prepared crown?

6 degree**

26-Chlorohixidene concentration in mouth wash rinse?

0.12**

27-Artificial teeth width compared with natural teeth?

A-Greater
B-Less ✅

C- equal

28-Xray: crown with open margin mesially and distally, what is the problem?

a- insertion obstacle ✅

b-too tapred prepration

29-Question about PFM crown

a-less metal

b- less colar✅

30-Patient take asprin 4 times a day, and he want to extract teeth, what test he
should do before extraction?

a-INR

b-Bleeding time

31-Asprin related to what disease

a-Osteoma

b-Osteod ostema ✅

c-Osteofibroma

32-Pic of a red lesion in both corner of the mouth, what is the responsible micro
organism?

a-candida albecan *********

33-patient came with fatigue, flue, and he want to do something, but you defer. What
is the drug that you should give him?

a- antihistamin

B-antiviral ✅

C-topical steroid

34-Kaposi sarcoma associated with what disease:


A- AIDS ✅

B- human papilloma

C- herps

35-Doctor was working to the patient and he was very careful when he touch or reflect
patients lips, what is the disease that doctor suspect?

Herps lipialis

pemphigus valgaris **********

36-Doctor want to extract amalgam teeth and then he want to preserve it, how?

Sunmerged with formalin 10%************

38-Patient vomit on the floor, whqt is the type of disaffectent:

High

Low********

medium

39-Patient vomit, then the nurse wept it with towel paper, to which waste she should must
through it?

Hazard***

Contaminated

Infectious

Not____ the active angulation of the blade is 45-90

-----------angulation of the universal curette bwteen the blade and shanke 70-90__

--------- angulation of gracy curette bwteen the blade and shanke 90-110

43-sequence of RPD abutment preparation?

A- rest seat prep, proximal prep, occlusal prep


B-proximal surface, contour, rest seat.

C-occlusal, proximal, rest sea

44- MCV test appear less than normal, the patient should take:

A- vitamin B12 deficiency

B-folic acid

C-iron deficiency ✅

D- vitamin D

45- pregnant woman, lower left quadrant with large swelling more than 2cm, with bad oral
hygiene, what is the management?

A- oral hygiene instruction**************************

B-defer after

C- scaling and biopsy

46- quad helix, what is the type of this appliance?

A-fixed orthopedic appliance ✅

B-removale orthopedic appliance

C- RPD

47- what is code 1:

A-fissure sealant ✅

B- amlga

c-remenarilzation

46- inlay latent cement type:

a-Type 1 ✅

b-Type ll

c-Typelll
47- patient with virus i think HBV, and he touch the handle of the drawer then the doctor
touch this handle, what is the type of infection?

a-direct

contamination

b-indirect ✅

c-airsole

48- patient came with incipient caries in the proximal, what is the best tool to diagnose?

a-diagodemt

b-bitwing x ray

51-what is the component of the dentin that consist 70% of the dentin?

a-organic

b-inorganic ✅

c-water

56- vertical dimension of rest is it :

A-Greater than vertical dimension of occlusion ✅

B-Equal than

C-Less than

57- R flex file cross section:

a-Triangle ✅

b-Square
‫مع التركيز حقها ‪EDTA ٪٧١‬الاسم الكامل حق‬
‫‪Ethylenediaminetetraacetic acid 17%‬‬

‫)متى يبدا السن يتكون في الجنين داخل الرحم (‬


‫‪6th week‬‬

‫وفرشة الاسنان تدخل كم ملي في الجنفجفا وقت التفريش‬


‫‪0.5 – 1 mm‬‬

‫)والحامل لمن تدخن ايش يسير للجنين ( اساالخيارات مي سندرومات ‪ ٣‬وكلفت ليب‬
‫‪Cleft lip‬‬

‫الستيج والقريد حقت البريو الجديدة يعطيك المعلومات كلها وتحدد هوا ايش‬
‫‪Read about new perio classification‬‬

‫‪not‬الفكسد وقت التراي ان طلعت فقاقيع من الابتمنت ايش السبب ياترى ‪ ،‬ووحدة من الخيارات كانت‬
‫‪passively fit‬‬
‫‪Increased space of cement‬‬

‫وطريقة التصحيح دورو عليها ‪post insertion problems of RPD‬اسئلة كتير على ال‬

‫للبريو ديزيز جابو حالة وعندها فيڤر ‪ ٣٣‬حرارتها ومرصوصة ‪combination of antibiotic‬ايش افضل‬
‫انتي بيوتيك كل خيار اتنين‬
‫?‬
‫لكل الحالات ( الطبيعي ‪ ،‬المصاب ‪ ،‬المتطعم ) ومتى ‪needle stick‬دورو على ايش تسوو بعد ال‬
‫‪immunoglobulin‬ياخد‬

‫‪removal of impaction wisdom‬كومبليكيشن بعد‬


‫الخيارات فيرتيكل ديفكت وهوررنتل دفكت‬
‫‪Vertical defect‬‬

‫‪ossifying‬من الخيارات بشكل افضل ؟ الخيارات وحدة منهم ‪lesion‬الاسبرين حياثر على ايت‬
‫وحاجة كدا دورو عليها ‪osseous‬والباقية كلها فيها ‪fibroma‬‬
‫‪osteoid osteoma‬‬

‫مااتكررلتلي كتير من اسئلة الشهر اللي فات بس اتكرر من حقت يناير وفبراير وبرضو كان التكرار قليل‬
‫نوعاً ما‬

‫كمان عن ربط الامبلانت بالسنة الطبيعية ايش ممكن يسير بعدين‬

‫ايش ممكن يكون ‪solid mass nodules in the neck‬وكمان‬


‫‪SCC‬‬
‫‪basal CC‬‬
‫حاجة كدا ‪fibroma‬‬

‫‪ CT‬وبعدجاب صورة من ‪. Diagrams‬فيالبريو ‪ bony defects‬جاب صورعن تصنيف ال‪. 1-‬ساء الخير‬
‫‪.‬وقالوشتصنيف البوني ديفيكتس هنا‪ .‬يمكن ‪ ٥‬اسئلة‬

‫‪.‬في البريو‪ .‬يعني وش االنقل خالل السكيلنق وكل االنقلز يعني ‪ angles‬ذاكرواال ‪2-‬‬

‫‪2-Answer: Angulation during scalling: 45-90‬‬

‫‪Activitionangle : 70-80‬‬

‫‪Sharping (blade to stone): 70-80‬‬

‫‪Sharping(lower shank to stone): 100-110‬‬

‫ولكن الي تجي كثيرة يعني‪ .‬مااتوقع ‪ subgingival calaculas‬نوع االنسترمنت المخصصة لل ‪3-‬‬
‫‪ hoe.‬اتوقعانهاال‪gracycurrate .‬اجابتها‬

‫‪Hoe: supragingival‬‬

‫جت أسئلة منه‪ .‬يمكن ثالث اسئلة‪ .‬يعني الفترة بين كل زيارة وزيارة وعلى ايش ‪4- periodontal mantinance‬‬
‫‪.‬تتحدد‬

‫على كامل الفم؟وشالعالج؟تحولها لدكتورتتحكم في السكر؟اوتبدا ‪ periodontal deases.‬مريضةعندهاسكروعندها ‪5-‬‬


‫بيريو؟‬

‫‪: artaiciane, lidocaine,‬للمرأةالحامل‪ .‬االجوبة ‪ class c‬يصنفك ‪ anesthetic solutions‬أي انواع ال ‪6-‬‬
‫?‪etedocaine, prilocaine‬‬

‫‪.‬سؤالواحدبس‬
6-Answer: Class B is allowed for pregent lady which is : Lidocaine , Etiocaine , prilocaine

Class C Not allowed :Mepivacine , buvicaine , procaine

7- ‫ االبتيورشن كويس"جايب راديوقراف لك" بس السيلرطالعه في‬.‫جاك بيشنت يشكو من الم بعد االندو بفترة وجيزة‬
apical aera. ‫االجوبة‬: reassuring, endosurgey‫وخيارات غيرها‬.

7-Answer: Wait and Follow up

8- ‫ مانجمنت ال‬angular chillitis ?

8-Answer: If there is a denture with low vertical dimention u have to correct it

Topical Nystitine (antifungal)

Nuturitonal supplement (B2 , Iron)

9- infection in this space will lead to cavernous sinus?

-Infraorbital

-Submental

- subminbilar

The correct answer should be ( inftatemporal space , canine space infection)

10- ‫وش تركيزال‬mouth wash ‫ الي ياخذها البيشنت‬over the counter ?

-0.05% Daily

-0.2% Weekly

11- pain killer with asthma patient?

11-Answer:Acetamenophine

12- pain killer that you don't use with asthma patient?

12-Answer: Iburofen , Naproxen , asprisin

13- ‫ بس بعدين جا يشتكي انها‬.‫ اول ماركبتها كان مبسوط‬.‫بيشنت جا بعد فترة وجيزة من تركيب كللسيراميك كراون‬
dark. ‫وش السبب؟‬

- chroma

- value
- hue

14- ‫ ال‬CHX ‫و‬Naocl. ‫ والفرق بين هنوليش نستخدم‬.‫مميزاتهن‬


‫ سؤالين بس‬.‫ذاومانستخدم الثاني‬.

15- ‫ وشالمكونفي‬retraction cord ‫ الظاهرال‬.‫الييمكنيسويمشكلةمعمريضالقلب‬epinphrine. ‫ماالظاهر قال انعنده‬


‫ لكنذكراسمعالجياخذهالمريضفتوقعتانهالعالجعالجقلب‬.‫مرض قلب‬.

16- ‫ ال‬sickle cell anemia. ‫ وشال‬oral manefstation‫لها؟‬

16-Answer:

The most common oral manifestations of sickle cell disease are mucosal pallor,
yellow tissue coloration, radiographic abnormalities, delayed tooth eruption, disorders
of enamel and dentin mineralization, changes to the superficial cells of the tongue,
malocclusion, hyperce-mentosis, and a degree of periodontitis
17- flabby ridge .

What is the technique you use to take impression for him?

17-Answer:

Mucostatic impression Material,Selective impression technique

18- ‫ وشتختارك‬base ‫ الظاهراالجابة‬.‫ بيسقويةاوبيسيعنيمرنهوخياراتاخرى‬.‫ تحتالحشوة‬rigid base.

19- ‫ وشتسوي؟‬.‫بيشنت جا بعد كم يوم ماهوراضي عن لون الحشوة ويقول لونها يختلف عن االسنان الي جنبها‬

-Resurfacing

- change restoration

20- ‫ سيناريوات عن تشخيص ال‬vertical root fracture .‫ بعضهاجايب صوراشعه معها‬.‫يمكن ثالث اربع اسئلة‬.

20-Answer: Exploratory Surgery is the best for diagnosis Followed by CBCT

21- ‫ اشعةل‬upper 6. 3 ‫ مسواابتوريشنفقطل‬canals. ‫السؤال انه جاك المريض يشتكي من الم استمرلشهورمن بعد‬
‫ وشالخط أالي سواه الطبيب الثاني؟‬.‫ماسوا االندوعند طبيب ثاني‬
-Missed canal. ‫( اتوقعها ذي الصحيحة‬YOU have to see the x-ray , Mostly it would be missed MB2)

- overfilled.

- underfilled
- ‫خياررابع‬.

22- ‫ جاسؤالعنال‬rests ‫ فيال‬removable. ‫وجايباختصاراتمثل‬.‫وشاليمنهنتسببمشكلةمعينةذكرهابالسؤال‬: RPA.


‫ والخيارالرابع‬.‫ماهوشرطهلالختصاريكونصحيحبسجايبكذااختصاراتيعنيماهوذاكراسمهاكامل‬aker.

23- ‫ مريضةموسوسة تفرش اسنانها كثير وتسوي‬flossing ‫ تشتكي من‬.‫ اقل شيء مرتين فياليوم‬bleeding gums.
‫وش الشيء الي معها‬:

-Non plaque induced gingivits‫اتوقعهاذيالصحيحة‬.

- self reflected harm ‫او‬truama.

24- ‫ وشنوعالمايتيرال الي توقف‬internal root resorption. ‫ ذكر‬MTA ‫بس ماذكرال‬Caoh. ‫لكن ذكراسم‬
‫ مادةواتوقع انهاال‬brand name ‫لل‬Caoh.

24-Answer: Caoh

25- ‫ وشال اسنان الي نبتت له وهوفي هذا العمر؟‬.‫ سنوات‬01 ‫طفل عمره‬

25-Answer:Max. & mandible 1stmolar , Max. & mandible incisors (cental , Lateral) All theses
below 10 years , First and second premolar From 10-12 years

26- ‫ سؤالينعنال‬biologic width ‫ و‬ferrule effect. ‫جايب المسافةبين المارجنوالعظم ويذكرفي السؤال ان السن‬
badly distructed‫ اذامكسورالسن بالمرة معناته مافيه‬.‫ اوال‬ferrule. ‫ ماهومتعمق‬.‫السؤال عامتقريبا‬
‫فيالسيناريوواالجابات واضحة‬.

27- ‫ سيناريوعن‬cd 2 ‫ مدعومةب‬implant‫صارت‬، loose ‫ وماعندها‬retention ‫ وش السبب؟‬.‫بعد كم شهرمن الموعد‬

27-answer: non parallel implants and loss of rubber

28- ‫ وش اكثرسن يجي فيه الي‬cervical enamel projection ?

28-Answer:The mandibular second molars showed the highest incidence of enamel


projections(51.0%), followed by the maxillary second molars (45.6%)

1- Easily palpated central incisors with 1 mm horizontal. Which grade ?


Grade 2

2- missing 46 and 47 is
sever mesially tilted what can occur in that area ?
Food impaction

3- what is the relation between stress and gingival condition ?


Stress was found to increase the inflammation of the gingiva.
4- what type of major connector should be used in case of palatal torus ?
Horseshoe major connector.

5- missing 44,45. what is the abutments ?


46,43.

6- nerve supply to the TMJ ?


- auricutemporal ( mainly)
- deep temporal
- massetric

7- nerve supply to the lower canine ?


Incisive nerve.

8- what is veracity ?
Truthfulness.

9- what is nonmalificince ?
Do no harm

10- patient 9 years old came to the clinic with his brother whi is 14 years old
and want to do ortho treatment. What should the doctor do ?
Ask for guardians approval and attendance.

12- access cavity for #14 with 3 canals ??????


Modified T shaped access cavity. (Triangular??)

13- hand scaling for the mesial side of #44 with which instrument ?????
5-6
11-12

14- picture of naber probe

15- pedo patient drink a bottle of mouth wash containing fluoride. What to do
?????

16- patient entered the clinic and suddenly he fumet on the floor and the
house keeping whipped it. Is that enough ?????

17- what is the color of file size 45 ?


White.

18- what is the amount of reduction in the buccal and palatal cusp in upper
molar for PFM crown ?????
2 mm in the palatal cusp
1.5 mm in the buccal cusp

19- how to remove gutta percha from the canal ?


H file
20- after giving a nerve block the patient feels he can’t see or open his eye.
What is your mistake ??????

21- patient came after trauma with lower jow fracture and feels numbness in
the lower lip. In Which part is the fracture ??????

22- upper primary molar. Which nerve to give the anesthesia for ?????

23- patient with cardiac disease and you want to give him anesthesia. What is
the maximum dose in milliliter ???????

24- which file is used for calcified canals ?


C file

25- what is the benefits of cephalometric ??????

26- cleft lip what is the causes ?????

27- antibiotics before treatment given to who ?????

28- 2 type of fissure sealant and asking about why one is better than the other
???????

-alot of cases and you should know the pulpal and periodontiom state (so you gotta know it
by heart)

-an xray pic ( and there’s a furcation resorption and a story of a patient who underwent RCT
) and asked about what’s the mishap happened during treatment ?

Direct perforation or furcation perforation * it’s the same

-gates glidden size 3 is equivalent to?

0.9**

-when preparing in the danger zone with gates ,what do u fear ?

Strip perforation **

-The use of explorer in RCT ?

Examine the pulp floor and locate canals**

-read about surgical and non surgical rct , there was lots of questions about it
I don’t remember alot about endo it was mainly cases , u won’t know it unless you see it

-a patient with heamofilia , and he needs extraction what’s the method used to stop the
bleeding post extraction ( and there was a pic of xray , badly decayed tooth with bone
resorption) ?

The main aim of providing what is known as coagulant cover is to raise plasma factor levels
to normal in order to attain adequate hemostasis following dental extractions.1 In general,
for minor surgery, patients with hemophilia A and patients with hemophilia B require a
plasma factor level of 50%-80% preoperatively and 30%-80% for 1-5 days postoperatively.4
Patients requiring general anesthetic must have factor replacement therapy in anticipation
of endotracheal intubation, which can cause trauma-induced bleeding.1 For major surgery,
hemophilia A patients need a plasma factor level of 80%-100% preoperatively and 60%-80%
for 1-3days postoperatively.4 Hemophilia B patients require a plasma factor level of 60%-
80% preoperatively and 40%-60% for 1-3 days postoperatively.4 Guidance from and
collaboration with the patient’s hematologist are essential to meeting the individual needs
of the patient based on the severity of the hemophilia, the degree of trauma anticipated,
and the dental treatment plan. Antifibrinolytic agents are used in conjunction with clotting
factor replacement therapy. These agents include lysine analogs such as tranexamic acid
(Cyclokapron, Pfizer, Inc) and aminocaproic acid (Amicar). Both agents can be taken orally or
intravenously.2 These drugs prevent postoperative bleeding by inhibiting the activation of
plasminogen to plasmin, which works to degrade fibrin clots. Thus, the administration of
antifibrinolytic agents stabilizes clots.

-paget disease of bone ,what’s the radiographic feature?

Cotton wool appearance***

4-5 questions about hepatitis know it by heart

-asked about the titer to know you’re immune ? ANTI-HBS

-a case of a nurse who got needle stick injury from a patient with history of hepB ? Check her
antibody

-sterilization of instrument used for a hepB patient

The centres for disease control and prevention (CDC) recommends Sterilisation or high level
disinfection of HBV, HCV or HIV contaminated devices.

High level disinfection does not reliably kill all bacteria endospores, only acceptable
alternative when sterilization equipment is not available.

-2 more questions about HBsAg ,I don’t remember what about it ! But study them all
2 questions about HIV

-the most common manifestation in oral cavity?

candidaisis

-and number of CD4 to get infections?

Under 200

-Symphysial fracture (picture) and asked about what plates are used to treat in numbers ?

2 plates

-mandibular deficiency , what do u use to treat it?

BSSO (most common)

Trans vertical ramus osteotomy used exclusively for mandibular prognathism

-Pemphigus valgaris most common site ?

Buccal mucosa***

-Proliferative verrocus something ???


Proliferative verrucous leukoplakia (PVL) is a rare type of oral leukoplakia, where
white patches that have a high risk of becoming cancerous develop inside the mouth. It
mainly involves the lining inside of the cheeks (buccal mucosa) and tongue. It starts as
a white plaque of thickened skin (hyperkeratosis) that eventually spreads and forms
rough, wart-like (verrucous) lesions that may look like cauliflower. The lesions are slow-
growing and progressive, and more and more difficult to control over time. The risk of
becoming cancerous is high, especially, of transforming to squamous cell
cancer or verrucous carcinoma.

-strawberry mouth ?

Wegner granulomatosis *

-cleft lip and palate patients will have ?

Deficient maxilla
-cephalometric analysis picture and u name the point ?

It was PNS**

-picture of band pusher

Definition of class one occlusion

The mesiobuccal cusp of the upper first permanent molar occludes with the mesiobuccal
groove of the lower first molar, but line of occlusion is incorrect because of malposed teeth,
rotations or other discrepancies.

Picture of maxillary teeth showing class 2 , but the question was about molar relationship I
don’t know what’s the point bruv

(Paedo was all easy )

A case of 16 years old boy with a retained canine what’s the effect ?

May cause resorption of lateral

Alottt of perio cases with treatment options , and new classification of perio *which I’ve
never seen in my life *

-furcation involvement grade 2 treatment (was a bit tricky*

GTR and u can do root resection or tunnel preparation in severe cases.

(Ethics was easy you’ll know it right away)

-professionalisms defi

-do no harm 2questions

-professionalisms (scenario)

-another scenario when the patient was referred from dentist A to B and then B did a
treatment plan and sent him back to A then the patient-decided to do the treatment with B
then A refused ? (Paternalism)

Proffesionalism
-confidentially

Restorative was kinda hard (tricky questions and alotttt)

-lab procedure questions 2

Removable (know the kennedy classification)

-know fluoride by heart

-retention in fixed

Factor affecting retention: magnitude of dislodging force, geometry of tooth preparation,


roughness of internal surface of restoration, thickness of luting cement and type of cement.

Alot of fixed cases

Pregnant women cases

-what local anesthetic agent is contraindicated Class C is contraindicated which is:


mepivecain, bubivecaine and procaine.

-position for a 9 months pregnant lady?

Left lateral decubitus position with the right buttock and hip elevated by 15ْ.

-which analgesic Acetaminophin

2 more case based questions about if u treat her or wait

‫لسالم عليكم‬
‫ سبتمبر‬٨ ‫انا اختبرت اليوم بتاريخ‬
‫االختبار ماكان صعب وماكان سهل‬
‫فيه اسئله كثيره تكرررت ليا‬
‫راح اكتب االشياء الي جات جديده‬

‫اش‬-٧ autoimmune disease SNC‫الي لها علاقه ب‬

a dementia

Bmultiple sclerosis

lipelipe S

D stroke

٢– ‫ اش يسبب‬allergy from La ‫ الجواب الصح كان‬methparben

methylparaben
pippl>>>‫اش البوزشن الي يسبب للحامل هيبوتنشين‬٣

‫ مابين الشانك لبريو بروب مع اللونق اكسس لسنه‬epgel ‫ اش‬-٤

١٧ ‫ ولا‬٤٧ ‫ولا‬٣٩

The lower shank should be parallel with long axis of the tooth

The angle between the shank and the blade of gracey curette is 60-70

‫سوال عن الكونسنتريشن للفلوريد في‬-٧ mouthwash ‫ ومو‬1.2 ‫ و‬1.10 ‫وكانو حاطين من الخيارات‬
‫محددين اذا يومي او مره كل اسبوع‬

Fluoride in daily use mouthwash: 0.05% (230 ppm)

Fluoride in weekly use mouthwash: 0.2% (900 ppm)

‫سبلنتيق لل‬-٦ avulsed tooth ٤ ‫ دقيقه والخيارات كانت اسبوعين او‬٦٩ ‫وما حددو اذا قبل او بعد‬

2 weeks according to new guidelines

‫وكمان جاني سوالين عن اش افضل‬-١ storage media for avulsed tooth

‫ موجود‬SSCC ‫اول واحد كان‬

‫والسوال الثاني كانت الخيارات سلايفا ولا سلاين و حاجتين ثانيه‬

SBCC

kpem SleC

temperature milk mllk

Ceeppl

Ceepae

Water
‫تعريف‬-٨ subluxation

Dental subluxation is a traumatic injury to the periodontal tissue[1] in which the tooth has
increased mobility (i.e., is loosened) but has not been displaced from its tooth socket.

‫ لمريض معه‬pSfirlulp ‫ ادا عطينا‬-٣ ischemic heart disease

‫اش راح يصير معه‬

aecrecerCpe

arrhythmia

uncontrolled hypertension

IDIN D’T FOUND ANYTHING AND THE SURGEON SAID IT IS NOT CONTRAINDICATED

٧٩- avulsed tooth ‫بنحطها في ايش علشان يمنع الانتيرنال رسوربشن‬

AIDE

ADE

sodium hypochlorite

‫والاسم الرابع كان شي غريب بس مدري اذا هو اسم تجاري للكالسيوم هيدوكسيد او لا‬

Ledermix

‫ ليش نكتب‬-٧٢ chief complain ‫للمريض‬

‫علشان الدكتور لا ينسى‬

‫ولا علشان المريض يحس انو اعطيبنا المه اهميه‬

‫ولا علشان نسجلها في الفايل‬

To compare it other patient’s complaint

٧٣_ primary molar ‫تكون‬

long and diverge roots

‫اصعب شي في السكيلنق لما تكون الروت‬-٧٤

Convergent tooth with long trunk **

Convergent tooth with short trunk

Divergent tooth with long trunk

Divergent tooth with short trunk


‫في الابيكل ون ثيرد ‪-٧٧accessiry canals‬كم نسبه ال‬

‫الخيارات كانت ارقام‬

‫‪%74‬‬

‫‪ -٧٦ resorbable‬اش هو السيلر الي ممكن‬

‫الزنك اوكسيد اوجينول او ‪.. rlppp‬‬

‫‪Resin and Zinc oxide eugenol sealer both of them can be resorbed so I don’t know the‬‬
‫‪correct answer‬‬

‫‪ -٧١‬ارسلو بيشنت للعيادات الاندو علشان يعمل افاليوشن لاندو معمول لبيشنت قبل ‪٤‬سنين كل شي‬
‫بس انو البلب تشامبر كان مكشوف لمده ‪ symptoms‬كان فيه كويس وحتى الاكس راي وماكان فيه اي‬
‫اكثر من شهرين‬

‫‪-‬نحولو يعملوله كروان على طول‬

‫‪-‬نحط تي اف ونتابعه لمده‬

‫‪-non surgical reteatment‬نعمل‬

‫‪-‬نعمل اريقاشن ونحط تي اف ونحوله يعملوله كروان‬

‫‪٧٨-main advantege of quad helix‬‬

‫انو مايعتمد على البيشنت‬

‫انو يعمل اكسبانشن بسرعه‬

‫‪-٧٣‬شي عن‬

‫كيف بيكون شكله ‪washing Snik for instrument before sterilization‬‬

‫‪-‬يكون فلات علشان ماتتجمع فيه المويه‬

‫‪-‬انو يخلي الانسترمنت تتغطى بالكامل‬

‫‪- minimum depth‬انو يكون‬

‫‪I DON’WNNK D‬‬

‫‪-٢٩‬فيه سوالين زي بعض انو راح نعمل كروان لبيشنت وكان المارجن حق الرستوريشن للكرست اوف‬
‫بون ‪ ٧.٧‬والسوال الثاني ‪٧‬‬

‫وكانو نفس الخيارات‬


-Sufficient biological width and furral. (In case of 5 mm of restoration away from crest of
bone)

-unSufficient biological width and furral

-Sufficient biological width but unSufficient furral

- UnSufficient biological width but Sufficient furral. (In case of 1.5mm restoration away from
crest of bone)

‫ لما نعمل بردج ويكون الابتمنت امبلنت والثاني ناتشرال تووث‬-٢٧

‫ماراح يصير فيه‬- osseointegration

‫راح ينكسر الكونكتور‬-

‫راح ينفك السمينت من عند التوث‬-

BOTH ANSWERS CAN HAPPEN

‫سنه عملنا لها برب وكان اوفر ردكشن وجاني بعد فتره ب‬-٢٢ Porcelain chipping ‫اش السبب‬

‫انو كان عند ثيك بورسلين وماكان له سبورت‬

‫مشكله في البوند مابين السرميك و المتل‬

‫ مشكل في‬pickling

‫كيف اتاكد انو فيه‬٢٣ uniform thicknss for ceramic

‫ والاجابات كلها لها علاقه ب‬wex pattern

I found only it is measured with caliper

‫سوال عن‬٢٤ ethic principles

‫بس ماكان في الخيارات البروفشنليزم‬

‫ بالمقارنه مع الاسنان المجاوره‬Cerm ‫البيشنت عملنا له كروان وجاء بعد كم يوم يقول الكروان‬٢٧

‫هوا اش يقصد‬

Hue

Value

Chroma

‫انوفيه‬٢٦ bilateral creamy granules on buccal mucosa ‫وكانت من مده طويله وماسوت مشكله‬
‫اش هيا‬
‫ليكوبلكيا‬

‫‪grepfelp elrCecl‬‬

‫وخلعناله وجاء بعد اسبوعين وماصار هيلنق وحط صوره لمكان ‪٢١ bisphosphonate‬بيشنت كان ياخذ‬
‫كيف راح اعالجه ‪ symptoms‬الخلع وكان واضح البون بس ماكان فيه اي‬

‫‪ mouthwash and AB‬اعطيه‬

‫‪ hyperbaric oxygen‬او اعطيه‬

‫او اعمل رسكشن للبون‬

‫واعطينها توبيكل سترويد لمده ثلاثه شهور وحصل امبروفمنت وقل ‪٢٨ lichen planus‬بيشنت كان عنده‬
‫وحطو صوره ‪ burning sensation‬الليجن بس صار عنده‬

‫‪ burning sensation‬اش سبب‬

‫‪Seilrplpppapapae‬‬

‫‪Eeelrge‬‬

‫‪Candidiasis‬‬

‫‪٢٣‬بيشنت كانو عند ناقص فايتمن ‪ B02‬اش راح يكون عنده‬

‫‪)eplkpe AlgeelSeepapc( eplkpe reilrcrrlkpc kecrlceapc‬‬

‫‪ -٣٩ osseointegration‬متى راح يصير فيه‬

‫‪-‬ثلاثه شهور‬

‫سته شهور‬

‫سنه‬

‫‪klparp 6 : Maxilla‬‬ ‫‪klparp 4-3 : SfelAepCp‬‬

‫وقالو ذا نستخدمه لايش ‪٣٧ gracey curetta‬جابو صوره‬

‫‪40 eeSpee‬‬

‫‪lingual 21‬‬

‫‪Alppe40‬‬
Ippaee

‫الختبار متوسط الى صعب‬

🛑‫ سؤال بالكثير‬٣٣‫إعاده االسئله تقريبا‬

🛑‫ اول جزء سهل عباره‬:

‫ سهل جدا واضح بس الزم تكبر الصوره النه بعضه مو واضح‬/‫اندو دايقنوسيس‬
‫سهل سهل مره‬

1- what type of bone is formed in the socket ?


Sequestrum

2- rhomatiod arthritis is seen in which syndrome


Sjogren’s syndromes

3- what is name of the test for the tongue papilla lose ?


Exfoliative test ?????

4- treatment for patients with warfarin INR 4 ?????????

5- local anesthesia for pregnancy ?


Class B

6- first sign in gingival inflammation ?


Exudate of fluid from the sulcus.

7- gingival enlargement around all crown surface which class ???????

8- x-ray with open margin in right and left what is the cause ?????

9- FPD case in metal try in good but after porcelian is not sitting what is the cause
????

10- how to determine the physical occlusion ?????

11- picture of lower anterior with RO line ?????


12- what first Toto for cast post and core ?????

13- picture of RPI clasp ??????

14- picture of gingival swilling and the histology. What is the treatment plan ?????

15- the most common place for pemphigus vulgaris ?


Strat in the mouth. Buccal mucosa

16- bradykinesia symptoms of ?


Parkinson’s diseases.

17- brushing technique for ortho ?


Charter’s method

18- if we placed implant with small diameter what will happen????

19- patient with pain on biting what is the test used ?????
Tooth slooth

20- recession of lower teeth ?


High frenal attachment

21- sitting of teeth and phonetic ??????

22- what is the cause of denture fall down when laughing ?????

23- herpangina caused by coxackievirus.

24- blood supply around the implant ?


Is lesser than around the tooth

25- what will happen when patient close his mouth on the saliva ejector ?
Cross contamination from the backflow.

26- size of GG 2 ?
0.70

27- what is the treatment of RL in lower jaw ??????

28- carried based ENDO ?????


Like guttacore

29- patient with pulp exposure from trauma 3 days what is the treatment ?????
RCT

30- picture of fingur rest in perio????

31- function of lactoperoxidase lysozyme found in salivary glands ?


Antibacterial
32- indication of systemic antibiotics????

33- prophylactic for patient with infective endocarditis ?


2 grams of amoxicillin given orally as a single dose 30-60 minutes before the
procedure.

34- management of asthmatic patient ?????

35- drug contraindication for renal failure ??????


aspirin, gliclazide, nitrofurantoin, and spironolactone.

36- when to take the biopsy ?


If the lesion didn’t heal for 2 weeks

37- prevalence of osses carters in periodontitis ?????

38- practice of determined caries risk assessment during active perio therapy ?????

39- why do we write the chief complaint ?????

40- impression should be placed in what for the infection control ??????
Airtight plastic bag ?

41- sequence of PPE put in ?


Gown
Mask
Goggles
Gloves

42- if you extract a tooth what is the best place ?????


Medical waste container

43- what is the master apical file ?????

44- what will happen if you work at the dangerous zone ????
Strip perforation

45- what step to use for avoid losing the working length ??????
Recapitulation

46- advantage of chlorohexidin in ENDO ?


Effective against E. Faecalis

47- picture of perio probe measure the gingival keraatenis ???????

48- causes of cheek bitting ?????? Different than w have.


‫ سبتمبر‬٨ ‫ختباري االحد‬
‫ سؤال تقريبا كلها اندو و سيرجري‬٣١١ ‫اول جزء كان‬
‫ سؤال كانت اغلبها بريو‬٣١١ ‫ و ثاني‬، ‫ والسيرجري حاالت تروما كثير‬, ‫اسئلة االندو اغلبها الدياقنوزيز و تريتمنت‬
‫تريتمنت و الفولو اب لكل حالة بعد كم اسبوع او شهر‬
‫ سؤال تقريبا االسهل كانت كلها ريموفبل و فيكسد وريستو‬٣١١ ‫اخر‬
‫ وبعض االسئلة عن االنسترمنت‬:

1- Best irrigant for smear layer removal


EDTD
2-irrigant system that cause apical discharge of the arrigant

3- file system with the most resoprication effect

Wave one

4-file with cutting end

K file cuuting tip

H file cutting effeciency

5- Gates gliddin sizes

From 50 -150

6- gate glidding lead to

strip perforation

7-gates glidding size 2, correspond to which file size

70

8-antibiotic for pregnant

Amoxicilline

9-antibiotic for sensitive to penicillin

Clindmycine 600 mg

10-antibiotic for HIV pt

11-shape of biopsy

( elliptical )

12- stafne bone defect ( panorama pic )

Angle of mandible

13- Solid ameloblastoma treatment


14-post and core for Upper molar

( palatal root)

15- post and core prep was done , and bleeding from the canals :

MTA or Radiograph first or continue procedure

16- pic of endo treated teeth with little sealant outside the canals

Reassurance and follow up

17- another pic of endo treated but with alot of sealent outside the canal .

endo surgery treatment

18- alot of Q about perio diagnosis and treatment of different cases and timing of follow up

19- disadvantage of florida probe

Underestimation of probing depth

Lack of tactile sensation

Expensive

20- angle of scaler to the tooth

21- sickle scaler function

Subragingival scaling

22- gingival enlargment covering two thirds of the crown , grade 3 enlargement

23- at which stage of inflammation erythema appears

Early

24- pain upon releasing from the bite , cracked tooth

25- the use of translumination to detect cracked tooth

26-definition of , intrusion , subluxation,extrusive luxation


27-best analgesic for pulpal and periapical pain

Ibuprofeine

28- blood test for hemophillia

PTT

29- pic of band pusher and asking about the function

30- maybe 4-5 questions from fluoride file

31-also ethics file 3 Q

32-wearing a lower kennedy class I RPD without rest seats , asking about the consequences

truma to soft tissue

33-teeth without undercut and you want to use it as abutment I choose

( dimpling ) is enamloplasty to modify retentive undercut

34-component in the acrylic denture cause irritation to the soft tissue

monomer

35- post in upper maxillary molar which root

( palatal root )

36- we use post for what purpose:

Retention - resistance to fracture or give strength

37- why the temporary restoration change color over time

Because change in benzoyel peroxide

38- mix of 1:1 ration of acrylic temporary crown ?

Polymaraization shrinkage

39- quad helix :


•half dental half skeletal •two thirds skletal •two thirds dental and on third skletal

40- deep class V , what liner under composite

CAOH

41- when doing alveolar plasty fear of injury to what nerve ? Lingual or mylohyod

42- splinting of intruded teeth less than 7mm ?

4 weeks

43- mesial surface of premolar which gracy number to use

11-12

44- best way to clean 3 unit FPD ,

super floss or proxy brush or regular brush

45- perio endo lesion , what to treat first perio or endo ?

Endo

Down syndrome with learing difficulty what to avoid : fluoride mouth wash

Cranial base form from ? Endochondral

Iron def anemia ? Microcyctic

-Partial pulpotomy for traumatic exposures (Cvekpulpotomy):

The partial pulpotomy for traumatic exposures is a procedure in which the inflamed pulp
tissue beneath an exposure is removed to a depth of one to three millimeters or more to
reach the deeper healthy tissue.

- dental trephination: surgical creation of a fistula by puncturing the soft tissue and bone

overlying the root apex to provide drainage. Called also apicostomy.


-Minimum diameter ( taper ) of accepted endodontic treatment is 0.06 ( file size 30 )

-GTR treatment indications :

-Mandibular fracture :

Condyle

Angle

Symphesis

body
-if you want to do crown for pt and the margin was 1.5 mm to the crest of bone :

-Sufficient biological width and furral

-unSufficient biological width and furral

-Sufficient biological width but unSufficientfurral

-UnSufficient biological width but Sufficientfurral

-if patient has vit B12 deficiency , he will get macrocytic hyperchromic anemia .

- ‫ وش المكون االساسي لل‬Gp

🍃Zinc oxide

- ‫ وش الي يخلي‬MTA radioopaque

🍃bismuth oxide

- ‫وشال‬mishaps during obturation

🍃over or under extended obturation-sealar extrusion-vertical root fx- nerve paraesthsia

- injectable warm GP which device :

🍃obtura II

- after crown cementation pt feel sensitivity: from cement

- pt with over counter crown and there is inflammation around it what is the tx: scaling and
remake crown

- pt with ant crowns and complain about black space what is the problem : violation
superacrestal ...

- many many question about disinfectants and vomit in the floor or spitt

- overprotective parent : on child : worsen child behavior , on dentist make dentist


uncomfortable

- pt with allergy to sulfate which impression avoided : PVS

- assistant put water in alginat what tha expect problem:imbibtion

- dr put her hand in the chair ? with medium disinfectant

- teeth extracted and need it for preclinical:

🍃if tooth doesn’t has amalgam: autoclave for 40 min, while amalgam containing teeth: 10%
formalinfor 2 weeks

- sickle cell what to avoid ? i forgot the options :(

- many Qs about perforations like very wide post and discharge from pocket
- many Qs about vertical fx.

- Reasonse implant analyses : use to evaluate the osteointegration of an implant

- cavernous sinus thrombosis affect which space ?canine space , Pterygoid plexeus of vien in
the infratemporal space

- when you put your finger extraoraly you can elevate whic ms :) ? tempral , masseter, M or
L ptyrgoid

- when you do border molding in mylohaoid area you activat which ms :) ? the option like
hundreds of ms in each option I can't even remember

🍃see the link below

https://www.longdom.org/open-access/anatomy-of-the-lingual-vestibule-and-its-influence-
on-denture-borders-2161-0940.1000122.pdf

- discoloration of provisional because ? benzyle peroxide

- Q about vital tooth which to avoid from provisional restoration? polymethyle methacrylate

🍃I think the answer is correct because the reaction of this material is exothermic.

- what is the cause ? like harmful to the pulp

- Q about Esthetic almost all from the previous Q except professionalism def.

- pin point exposure? MTA

- lateral luxation time for splint ?

4 weeks

- white lesion in the lateral of tongue not painful with pic ? exsional biopsy i think

- Tb Qs form the previous Q like scarfula and acidfast

- pt had history of Tb but he said he is not infectous what to do ? I choose deffer until you
have clearance because it is not emergency

- biopsy from minor slivary gland with sarcoidosid what you will see ?
🍃in srciodosis: salivary gland present with painless enlargement of parotid gland,
xerostomia. Biopsy will show noncaseating granuloma !

- sub mandibular duc where to empty ?sub mandibular gland duct drain through (wharthon
duct) at caruncle at the base of tongue

- ostitis need Ab or not .. Idk :'(

- osteoporosis take bisphosphanate need extraction what to do ? stop Tx before 3 months

- ledwig angina

- CBCt and what is the deffect ? 1,2,3 wall defect

- pt take diagoxin and you give epinephrine what will happen? angina or increase Bp

🍃cardiac exitation and arrythmia.

🛑notes if pt is taking betablocker and receive LA with epinephrine this will lead to acute
hypertensive episode

- hemophilia pt ? RCT or extraction

RCT

- end stage renal disease what to see ? petechia or hypotension

Petechia.

- marginal preservation on PM during prep

- how much should be left from the gingiva in Cl ll prep

- hight of core ?

- mand fx ? problem in occlusion

- teeth with different occlusion ? alveolar fx

00# - inteuded minmally ? observation and follow up

- after extraction oro antra commincation "2mm" what to do ? Ab, platal , buccal
advansment ( in case of more than 0.5 mm) or gelatin foam and figure 8 (in case of less than
0.5 mm).

- peri abutment Qs

- cause allergy :nickle

- why we write cc : Routine step

‫ ي سوال اجا انه لما بغير المريض اسنان الريموفبل ل نوعيه اسمها‬: PMMA. ‫ االسنان بيصيرلها‬wearing ‫بعد ما‬
‫ينطفها المريض‬
‫شو السبب‬
‫‪ poor solubility of PMM A in organi‬وكانت الخيارات انه‬

‫‪Low abrasion resistance‬‬

‫‪High‬‬

‫‪ : f or v‬كمان سوال عن حرف‬

‫‪Laboi dentalty‬‬

‫‪Palitlolingually‬‬

‫وكمان خيارين برضو‬


‫‪ :‬في كمان سوال عن انواع الفراكشيرر بالسن‬
‫كومبليكيتد و انكمبليكيتيد‬
‫ايمنامل اودنتين‬
‫او واصل البلب‬
‫او اينامل دنتين والسيمنتم‬
‫بعد ما ‪ wearing‬االسنان بيصيرلها ‪ PMMA.‬في سوال اجا انه لما بغير المريض اسنان الريموفبل ل نوعيه اسمها‬
‫ينطفها المريض‬
‫شو السبب‬
‫‪ poor solubility of PMM A in organi‬وكانت الخيارات انه‬

‫‪Low abrasion reisestan‬‬

‫‪High‬‬

‫‪ f or v‬كمان سوال عن حرف‬

‫‪Laboi dentalty‬‬

‫‪Palitlolingually‬‬

‫وكمان خيارين برضو‬


‫‪ :‬في سوال اجاني عن الجنجفا جروث انديوسيد درج معهم كمله ديفولبتمنت‬
‫يعني كيف عنا بالعربيه شفوي لساني‬
‫او حلقي او هيك‬
‫‪Duriticc‬‬

‫‪ pheoityin‬والخيارات كانت‬

‫‪Ac inhibitor‬‬

‫‪Calcum canal blockers‬‬

‫اي ماده من االريجيشن عندها خاصيه بروتواليتك في التشو‬


‫ا هذا بديل لشو ‪isradipine‬‬

‫‪alternative for nifidepine‬‬


‫في كمان سوال‬
‫بنت عندها حساسيه باسنانها وعمرها سبع سنوات راحت مع امها للدكتوو الحظ انه في االنسايزيل ثيرد‬
‫هايبوكاليسيفيكسشن باللنسايزور والموالرز البيرميننت‬
‫شو المشكله الي عندها‬
‫‪MIH‬‬

‫في موالر انسايزير هيبوكالسيفيكيشن‬


‫وفي دينتال دسبليزيا‬
‫وفي ايمبريفكتا وكمان اشي‬
‫مع العلم ‪ food cutting effcincy‬سوال بالبرستو عن افضل نوعيه اسنان الزم نحطها لمريض بيشكي من عدم وحود‬
‫انه الريموفبل او الدينشر الي مركبه كثير مرتاح ومبسوط عليه والريتينشن عالي‬
‫الخيارات كانت اناتومكل كراونز‬
‫وال نن اناتومكال كراون‬
‫وال سيمي اناتومكل وكمان شغله برضو‬
‫سوال بيحكي شو اهم اشي للمريض الي بده يعمل برستو ترتيمنت‬
‫من ضمن الخيارات كان اشي عن االجتماعيه بسن الناس يكون حلو‬
‫او فانكششيناال‬
‫وفي حواب حلو جامع الخواص كلهن معبعض يعني سوشيل وفانكشيننال‬
‫سوال عن كالس ثري كندي كالسيفيكيشن وفي الم للمريض عند االنسايزر والكناين باللثه‬
‫شو المشكله الموجوده بالديزاين تبع الريموفبل‬
‫‪Non stress brakers‬‬

‫‪Non rest in design‬‬

‫وكمان خيارات بس مش ذاكرها منيح وهاي الخيارات برضو مش كامله انا مش عارفه ‪Gingival stripig design‬‬
‫اوضحها‬
‫‪ tmj capsule‬شو اسم المسل الي بتعمل اتاشمننت مع ال‬

‫‪lateral pterygoid‬‬

‫اجا سوال عن جينيراليازيد بيرودونتايتس ويعاجلها الدكتور بشكل كامل‬


‫وبعد االيفاليويشن فيزيت الحط تحسن ولكن في خمسين بالميه بليدنج‬
‫شو وضعها‬
‫‪ high risk of periodontits progression‬هاي‬

‫‪Or recerrunc of Periodontitis‬‬

‫وكمان خيارين‬
‫وال‬
Implant overdenture

‫ السؤال كان انو‬loos ‫؟‬.. ‫ايش السبب‬

Non parallel implant and looseness of the rubber

Mishaps.. X-ray ( missed canal )

What is the test for Fracture root or tooth I think ( bite test )

Access cavity of premolar 3 canal

Triangular , Modified T shaped

Most of endo q about diagnosis and it’s so easy

Over protective parent ‫كيف بيأثر على الطفل‬

Worsen the behaviour the child

Makes the dentist uncomfortable

What is the impression material that we can pour it after 24 hours

Addition silicon

Ethics the q about / ethical - moral - professionalism ‫انو بالترتيب‬

Ethics code:

Ethics-Law-proffesionalism

‫ صورة لل‬band pusher for pedi

2 q about splitting for subluxation and avulsion

Mentioned before

Ortho q / 1 q about the classes another 2 q about what to extract upper or lower premolar

Class II: extract upper 4


Class III: extract lower 4

(And the last q about the appliances

‫)^ كانت كيس سيناريو و بعدين ايش حنستخدم‬

‫سؤال واحد بس عن الفلورايد‬


‫ كم نسبه ال‬Flouride varnish

5% (22600 PPM)

‫بريو كانت اغلبها كيس سيناريو بس مو مره صعبه‬


‫و من التصنيف الجديد سؤال واحد‬
‫ مثال كالم كالم كالم و بعدين‬stage 1 grade b

‫ زي كدا و بعدين الخيارات‬..

‫ فكسد جا سؤال عن ال‬taper ‫ ب ال‬mm

6 degree as I know

‫ سؤال عن‬ppe I think ( mask/ gloves ‫و كدا بس بالترتيب‬

Put in: Gown, Mask, eyewear and gloves

GETRID OFF: Gloves, eyewear, gown and mask.

‫ في سؤال انو‬pt have a crown on 11 or 21 I forgot ‫المهم انو هيا عاملته قبل أسبوع و بعدين جات تشتكي انو‬
‫ في‬bleeding

May be over contour or violation for biological width

‫ سؤال عن ال‬chroma and value

Best treatment of fracture root

Apical third root fracture: u can do RCT for coronal part

Middle third: u can do the same

Cervical third: going for extraction.

Vertical root fracture: in most cases extraction.


‫ في كيس عن‬geographic tongue

‫ كيف نقيس ال‬width and thickness of the ridge ‫كان سؤالين‬

Wanson calipr

How to detect the proximal caries

Bitewing x-ray

How long the Tb stay at room

4 hours

ANUA ‫جا كم سؤال ذاكروه بس كانت سهلة‬

Paget’s disease in the x-ray:

Cotton wool appearance \

‫ ماكان في اشياء عن ال‬tumor and bacteria ‫كتير يمكن سؤال او اتنين يعني األشياء اللي تلخبط مافي‬

‫ لو افتكرت اي شي تاني حرسلكم‬..

‫ الله يوفقكم يارب و يرزقكم بالدرجات اللي ترضي خاطركم‬..

Pregnant women she did scaling and rot planning after that she noted there is
lesions palatal to the 25 and 24 what’s the treatment?
A-scaling and rot planing and systemic antibiotics
B-scaling and rot planing and topically antibiotic
C-topically antibiotic

Pt did two implants 24 25 after 4 months there is distal mobility for the 24 why
Periimplantitis or implant near to each other
What is the depth of the preparation in the dentoenaml junction in the class 1
1.5-2 mm
2 0.8 2.5

Pt with complete dentuer of maxilla and rpd in mandible after 8 years what will
be the kind Of maxillary ridge
Flat Flabby

First thing to look at it in the insertion of the crown Proximal contact


Case sinario ......attachment loss is 3-4 Periodontitis stage 1 grade a
Periodontitis stage 2 grade a Periodontitis stage 3 grade a
4 grade a
Case sinario ......attachment loss is 2-3
‫ فية سوال كان عن‬unc prop
‫ وسوال عن‬prop ‫ سوال‬7 6 7 ‫يقيس كل‬
case sinario ‫ وكان ذاكر‬isolated pocket ‫سوال عن‬
7 days hbv ‫كم يقعد في الغرفة ايضاء ال‬

tb
‫سوال عن‬
most costly effective floried
‫سوال‬
‫ عن‬pns
‫سوال اصعب سن يخدر‬
Mandibular 1st molar in cases of irreversible pulpitis
‫سوال عن النيرف المسوال عن تخدير اللور كناين‬
‫سوال ايش المادة الي تستخدمها عشان تميز ال بالك‬
Disclosing tablets

‫سوال عن التبر االفضل للكراون‬


6%
‫سوال كيف تنظيف خط الماء في العيادة‬
It should be less than 500 CFU/ML (colony-forming units per milliliter)
‫صورة لالبتمنت حق االمبالنت‬

Patient on warfarin had medical clearance last week for dental extraction, but on the day of
extraction the INR was 4:

a) Refer to physician to keep INR below 3.5

b) Extract with local measures

During denture processing, there are discrepancies that can’t be avoided due to material
properties, what step will minimize those discrepancies?

Clinical remount

b) Laboratory remount

Selectivegrinding

During protrusive record there was an interference in the upper premolar, from where will
you remove?

* a) Mesial incline of upper cusp

* b) Buccal incline of lower cusp

There was no Distal incline of upper in the choices

The question or answers are incorrect,


maybe the question about centric interference (forward slide) can be corrected by grinding
the mesial inclines of maxillary teeth and distal inclines of mandibular teeth.

While Protrusive interferences generally occur between the distal inclines of the facial cusps
of maxillary posterior teeth and mesial inclines of the facial cusps of mandibular posterior
teeth.

Reference: dental decks

Best emergency treatment to do for symptomatic irreversible pulpitis?

* a) Pulpotomy

* b) Direct pulp capping

* c) Indirectpulpcapping

No pulpectomy in the choices

Patient with liver failure needs extraction, what test will you request?

a) D dimer

b) PT

c) BUN

years old patient with aggressive periodontitis, best management22

a) Scaling and root planning

b) Scaling and root planning with amoxicillin


c) Scalingandrootplanningwithclindamycin
d)

Scaling and root planning with combination of metronidazole and amoxicillin

Minimum time to run waterlines after each patient

a) 10-15 secsb) 20-30 secs c) 30-40secs

Space btw marginal ridge of molar and class 1 cavity?

2 mm

8) Space btw marginal ridge of premolar and class 1 cavity?

1.5 mm

28) Best method for growth determination?

a) Hand wrist
b) Lateral cephalometric

Optimal fluoride dosage:

a) 150 ml rinse 50 mg gel

b) 200 ml rinse 30 mg gel

14years old boy complains of hypersensitivity related to lower left molar, upon examination
there was yellow patches and pitting in the tooth, what is the diagnosis?

a) Dentinogenesisimperfecta

b) Dentine dysplasia

c) Hypocalcified

amelogenesisimperfecta

)00Personal protective equipment for everyday dental procedures includes:

a) Gown, mask, eye goggles, gloves

b) Gown, mask, eye goggles, gloves, headcover

)02Consent should be:

a) Always written

b) Can be verbal

Situation where it’s acceptable for the dentist to disclose information from the patient’s
record?

a) When consulting a colleague

b) When it’s the public right to know

Smoker patient complaining from white area in upper posterior gingival margin that could
not be rubbed off, best management:

a) Biopsy

b) OHI

c) Reassurance

d) Scaling and root planning

Quad helix appliance is:


a) Removable rapid expansion

b) Fixed with dentoalveolar component

c)Fixed with dentofacial orthopedic component

Best way to prevent caries for large population (2qs):

community water fluoridation

Patient needs crown on #25 but tooth is equegingival, how to place rubber dam? (with x-ray)

a) No need for RD

b) Cotton roll isolation is enough

c) Extractthetoothit’snotrestorable

d) Place clamp on26 andto #24 extend RD

)08Management of Ludwig’s angina:

a) Incisionanddrainage secondary

b) Antibiotic primary

Congenital heart disease seen in:

Down’s syndrome

)20Breast cancer patient with poor prognosis and questionable tooth needs post core and
crown, what is your management?

a) Extract the tooth

b) Just RCT for the tooth

c) RCTpostandcorethencrown

Metastasis seen in the mandible most commonly in patient with:

a) Colon cancer if there is no lung cancer

b) Squamous cell carcinoma

Deference between enamel and dentin in primary and permanent teeth:


a) Enamelanddentinearethickerinprimary

b) Enamel and dentine are thinner in primary

c) Enamelthickeranddentineisthinerinprimary

d) Enamel thinner and dentine is thicker in primary

For proper shade selection of the luting cement for veneers:

a) Mix 1:1 ration of base and catalyst

b) Usemorebase

c) Usemorecatalyst

Heavy continuous force will cause:

)0Bone necrosis

2) Bone deposition

3) Undermining resorption

Lateral luxation splint for how many weeks

a) 2 weeks

b) 3 weeks

c) 4weeks

)27Pink tooth of mummery is due to:

a) External resorption

b) Internalresorption

c) Calciumhydroxide

)28Why COL first area to get inflamed?

a) Hard to clean
b) Non-keratinized tissue

)20What type of the following causes abrasion?

* a) Tooth paste

* b) Tooth gel

* c) Tooth powder

)31Posterior teeth wore out but amalgam looks like an island, what is the cause?
a) Attrition b) Abrasion c) Erosion

)30Worst gingival phenotype:

a) Thin scalloped

b) Thick scalloped

c) Thinflat

d) Thick flat

35-years old patient with thin gingival phenotype and generalized bleeding on probing with
40% bone loss, treatment?

a) Scaling and root planning

b) Polishing

c) Don’t treat because scaling and root planning will cause gingival recession

)33Worst type of bone: IV

The most important risk factor for periodontitis reoccurrence:

a) Heavy smoking

b) Poor OH

Patient known to be high risk of periodontal disease had an implant, after complying with
recall appointments for the last 12 months, what would be the next recall interval

a) Every1-2months

b) Every3-4months

c) Every6-9months

d) Every9-12months

)30Best x-ray to locate impacted canine:

a) Occlusal

b) CBCT

c) PAwith2angulations
Scaling mesial to premolar?

Gracey 11-12

2types of root planning scalersavailable, which are:

a) Gracey, sickle

b) Universal, Florida

c) Universal,areaspecific

)38Diabetic patient presented with bluish purple discoloration of the face, they also
mentioned gangrene in the question, diagnosis?

a) Viral infection
b) Fungal infection c) Fasciitisbacteria

)30Patient with knife edge mandibular ridge and large tongue, best impression technique:

a) Admix

b) Mucostatic

c) Selectivefunctional

Patient with insufficient intercuspal distance and clenching his teeth, best restorative
treatment?

a) Ceramic onlay

b) Metallic onlay

During try in of gold FPD, there was rocking, what to do to correct problem?

a) Pick-up impression

b) Fix soldering index

c) FinalCementationofFPD

d) Provisional cementation

Location of Stafne bone cyst:


a) Angle of mandible

b) Mandibular ramus

Mandibular foramen location in children:

a) Above occlusal plane

b) Below occlusal place

c) Atthelevelofocclusalplanemedially

d) At the level of occlusal plane laterally

)44Patient feels pain in left eye and is stressed and clenching his teeth, what does he have?

a) Temporal neuritis

b) Cluster headache

c) Trigeminalneuralgia

d) Migraine

)40NI-TI rotary files rarely cause canal transportation or perforation, this is due to:

a) Low modulus of elasticity. b) High tensile strength

)46Patient says his complete denture teeth are wearing off when he cleans it with brush,
this Is due to?

a) Low abrasion resistance of acrylic

b) High PMMA content

Boy with class II div.1 malocclusion, what is most important relationship to be taken:

a) Cusp groove relationship

b) Functionalenvelope

Universal probe tip size:

0.5 mm

70) Medication increase saliva:

Salagen
‫ لبيريو كثيرة اسئلته و مدققين على‬CAL and percentage of bone loss ‫يجيبون حاالت و معلومات كثير‬
‫ويطلبون العالج‬

‫هذي بعض االشياء اللي اتذكرها‬

Shortesht attached gingiva width:


Molars
Anteriors
Canine
Premolars

Shortest papillary length buccolingually:


Canine
Molars
Anteriors
Premolars

Hardest tooth for scaling:


Long trunk convergent roots✅
Short trunk convergent root
...

Patient after tooth prep. Provisional crown will be made by indirect chair-side
technique. Tooth diagnosis is normal pulp tissue, which material to use:
PMMA ( i chose this because of exothermal setting reaction)
PEMA
... idont remember the rest

Factors affecting crown retention form in order:


Parallelism, height , type of cement, dislodging forces, i don’t remember the rest

dislodging force-hight-parallesm-the roughtness of internal surface of the restoration ,


cement space , type of cement

Medication that cause gingival enlargement ( they didnt write the names of the drugs
only the categories)
-Calcium channel blockers**
-Beta blockers
-

Main disadvantage of resin cemented fixed partial denture compared to


conventionally cemented FPD

Ludwig’s angina ( read everything about it and its management)

Facial space infections and causes ( they got a picture of someone with a facial
swelling with absence of nasolabial fold and asked which space infection caused this
)
canine space infection

-What to do after needle-stick injury from someone who’s HIV positive:


- do the test and wait for results
- Take prophy immediately ✅
- I forgot the rest

Tooth with internal discoloration which kind of crowns should be placed:


- etched porcelain
- Aluminum
- Veneer
- I dont remember

RPD kennedy class 1 was fabricated without occlusal rest, what is expected?
Gumstripping

Which force in RPD affects the abutments the most?


- horizontal✅
- Vertical
- Horizontal and vertical

During denture processing, there are discrepancies that can’t be avoided due to
material properties, what step will minimize those discrepancies?
a) Clinical remountb) Laboratory remount
c) Selective grinding

Patient with liver failure needs extraction, what test will you request?
a) D dimer
b) PT✅
c) BUN

Situation where it’s acceptable for the dentist to disclose information from the
patient’s record?
a) When consulting a colleague
b) When it’s the public right to know

1Smoker patient complaining from white area in upper posterior gingival margin that
could not be rubbed off, best management:
a) Biopsy✅
b) OHIc) Reassuranced) Scaling and root planning

-Tug back definition : Tug-back is a slight resistance to pull of the GP master cone
when it is removed from the canal.

You had an asthmatic patient and wanted to check if its controlled which test would u
do? ( theres no spirometry or PEF) its a different question
- Check oxygen levels
- Chest xray
- Dont remember the rest

Asthmatic Pt requires RCT on tooth #21:


- use rubberdam throughout the treatment
- Cotton roll isolation
- Cotton roll isolation and them RD for obturation only.

What can u see in a down’s syndrome pt:


- NO periodontal disease
- NO caries✅
- NO dental anomalies

11 years old pt came 9 months after trauma on tooth #21, pulp was non vital. What is
your tx?
- RCT✅
- Direct pulp capping
- Indirect pulp capping

‫نا اختباري كان اليوم‬


‫ركزو على الفالبس بالبريو و‬
Medical compromised patient ‫جاني كثير اسأله عليها‬
‫اورال ميدسن ركزو على الريديوقرافك ابيرنس لليجن‬

Thrombocytopenia :
CBC
Surgical procedure can be performed if it is 50,000 or higher

Hemophilia + von willebrand disease


(The difference between VWD and hemophilia is that VWD is caused by
decreased or defective von Willebrand factor and hemophilia is caused by
decreased or defective clotting factor VIII or IX (clotting factor VIII in hemophilia A
and clotting factor IX in hemophilia B))
PT and APTT
Avoid use NSAID and aspirin

Warfarin:
INR
If it is 3,5 or less we can perform extraction and dental hygiene .
Non invasive procedure can be performed without any precautions

Sickle cell anemia


Treat patient normally but the aim is to prevent stress and infection

Stress reduction protocol


1-short morning appointment
2-discuses any question with patients
3-clear explication of procedure and expected feeling
4-good pain control
5-promedicate with benzodiazepine *if needed*

HIV:
CBC within 5-7 days prior to surgical procedure
Treat if WBC more than 2000 , platelets more than 60,000 and CD4 more than 200

-if CD4 less than 200 prophylactic antibiotic is indicated prior to invasive procedure

For chemotherapy extraction should be done at least 2 weeks before IV


chemotherapy
If there is any emergency and You want to do surgery or scaling you should request
CBC within 24 H if platelets more than 50,000 you will perform necessary dental
procedure also we have to check absolute neutrophils count less than 900 we have
to postpone procedure or consider prophylactic if it is urgent treatment

Radiotherapy extraction at least 2-3 weeks before radiotherapy follow up every 3-6
months

‫ يوم‬٣٢ ‫جاني متى الوقت الي تسوين فيه للبيشنت اكستراكشن قبل ما ياخذ كيمو وماكانو حاطين باالسابيع الجواب الصح‬

‫جاني كمان سؤال في البريو اذا كان عندك شورت روت ترنك وسويتي كراون لنقثنق ايش ممكن يصير‬

1. Double papilla flap (pic)


Double papilla flap is an alternative technique to cover isolated recessions and
correct gingival defects in areas of insufficient attached gingiva, not suitable for a
lateral sliding flap

2. Case scenario about patient did scaling and root planning and came with
periodontal abscess after few days what is the management
3. Who probe
The probe that has small round tip 0.5 mm
4. Staging and grading simple case scenario
5. Management of hemiseptum one wall defect
Remove remaining bone wall
6. Case about patient came with periodontal infection (pain) with fever what to
do?
7. The angulation of periodontal probe to measure periodontal pocket?
8. Type of root that is easy to do scaling and root planning regarding root
trunk?
Short trunk and long divergent
1. Depth of pulpal floor in cast inlay
1.5 mm
2. Contraindications of composite resin restoration

3. Dentist made esthetic anterior restoration but after few days patient
complain from discoloration what to do?
Surface change
4. What causes discoloration in temporary restoration?
Benzoyel peroxide
5. Read about amalgam failure type : isthums, blues, ditching.
1. They focus on pemphgus velgaris test
Tzanck test
Direct imunoflorecent test
2. Shape of excesional biopsy
elipticall
3. Patient have transplanted organ what medication cause gingival
enlargement
cyclosporine
1. Common cause of over-denture failure?

2. Pic of epulisfissure
3. Patient complain from upper complete denture upon examination there is
nodular like enlargement in anterior palate?
Pappilary hyperplasia
4. Case about kennedy classification (easy)
5. Patient have lower RPD complain from sorness over ridge he is kennedy
class 1 upon examination there is apical displacement of denture what to do?
Relain
change indirect retainer location
change major connector
choose more rigid clasp
6. Sequence of preparing abutment tooth for rpd?
0 ‫ اليوم كان اختباري‬.. ‫ السالم عليكم‬sep ‫ جزء االندو مرره سهل اصعب‬.. ‫تكرر أسئلة صراحة مو شوي من سبتمبر‬
‫ شي كان البروسثو وبكتب اللي تذكرته وبالتوفيق للجميع‬..

1-R flex file cross section = triangle

2-Dentin most of inorganic content = hydroxyapatite crystals

3- HBVpt touch handle = indirect

4-onlay (gold) luting = type 1

5- Mandi deficiency with class 2 skeletal

What’s proper appliance ! Fixed appliance : Herbest, Removable : Twin block

6-middle under cut = I bar (RpI)

7- fluoride in water =1 ppm

8- curette for distal molar = 13-14

9-vomiting on chair = ‫ماكان بالخيارات ميديم بس اخترت اتبع تعليمات المصنع حق الكرسي‬

10- Pt vomit then wept it with towel paper , which waste !! = hazard

11-open margin x Ray where’s the problem = insertion

12-taper from cervical to occ = 6 degree

13-avulsed tooth more than 5 hours we splint it for how many weeks !! = 4 weeks

14- listerine complications = epithelium desquamation

15- class ii with upper and crowding with lower = extract upper 4 and lower 4

16-complication of gates glidden = strip perforation

17-paget’s disease = cotton wool

18-short implant fracture how to remove it !!

19- how to magerment gingival thickness = Perio prob or ......... caliper ‫ابحثواعنه اسمه غريب‬

20-dangerous area with malignant = mandible

21-col= not keratinized

22-GP in Gp = 20%

23-EDTA =ethylenediaminet tetracetic acid 17%

24-deep class V = ca hydroxide

25- differences between primary molar and perment = thin enamel thin dentin with primary

26-caOH2 with pulptomy = internal resorpthion

27- 2 stage implant placement = cover screw

28- end stage renal disease = petechae in oral mucosa


29-Liver disease = defi factor 8

30- best mangement for dehydration= water

31-length of fiber post from = 2mm above gingival margin (not sure)

32-articulations = hanau

33- pic of double papilla flap

17-9-19

Endo: very very clear.

1- pic of bone defect> two walls bone defect.

2-probing under LA:

- transgingival ”something but was not probing” to crest ridge. Or

- Sounding of crest ridge.

3-cbct of impacted canine, where is the location:


- mid of the ridge

- Buccal impacted

- Lingually impacted

4-down syndrome pedo pt and has mental difficulties, which is avoided:

- tooth paste

- Flouride supplement

- Mouth rinses

- Water fluoridation

- 5-what is the most in retention and resistance:

- Cement- maybe

- Increased the prep taper

- Reduce the height of the prep

- Increase the rough of intaglio surface crown

- 6-skeletal class III, needs upper and lower compelet denture what to do? Regarding the
teeth setting

- 7-most tooth to have fractured root in ortho treatment


Teeth with conical roots )incisors)

- 8-why to write CC.. To compare it to other chaif complain

- 9- band push by whom? Initially by the dentist or finally by the pt

band pusher : initial seating by the dentist

band seater : final seating by the patient

- 10-Hiv less duration… 10 Years is The length of time can vary widely between individuals.
Left without treatment, the majority of people infected with HIV will develop signs of HIV-
related illness within 5–10 years, although this can be shorter. The time between acquiring
HIV and an AIDS diagnosis is usually between 10–15 years, but sometimes longer.

- 11-many Kennedy classifications

-Frankfort line

The Frankfort-mandibular plane angle (FMA) is formed by the intersection of


the Frankfort horizontal plane and the mandibular plane. This angle can be traced and
measured by means of a diagnostic overlay. An FMA of 25 +/- 5 degrees is within normal
range

-definition of autonomy and veracity

medical practice autonomy is usually expressed as the right of competent adults to make
informed decisions about their own medical care.

Veracity is defined as being honest and telling the truth and is related to the principle of
autonomy

- HBV which carrier ? HBsAg

The virus is transmitted mainly through blood and sexual contact. Although hepatitis B
infection does not usually cause any symptoms, the disease can be severe.
After infection 5% of patients remain chronic carriers (HBsAg carrier) and therefore
infectious. ... The virus cannot be transmitted through intact skin
- Nerve sensation of posterior third of tongue ?

- Tx of multiloculat radiolucent in lower jaw with root resorption?

Marginal recection

- MB root of Maxillary 2nd molar supple by which nerve?

- Strawberry gingiva?

WegenersGranulomatosis

- Safe drug for asthmatic pt? Paracetamol

- Pt with peptic ulcer contraindication use ?Ibuprofen

- Test used to detect if pt asthma? Expiration test(Peak expiratory flow ,Spirometery)

- Most common organism in endo abscess?Mixed

- Scenarios of Trauma child : tooth extrusion without complete remove from sulcus in upper
ant incisal :Reposition the tooth and splint it for 2 weeks , RCT if there is signs of necrosis

- Pt did extract of sound tooth and u need it in preclinical? No amalgam :Autoclave for 40
min , with amalgam : Put in formalin 10% for 2 weeks

- Pt with fracture tooth, after extraction he want to take the tooth? A.Autoclave.
B.Unnecessary process. C. Low disinfection. D.intermediate disinfection

- West for an Extracted tooth?extracted teeth to be potentially infectious material that


should be disposed of in medical waste containers.
Extracted teeth containing amalgam should not be placed in a medical waste
container that uses an incinerator for final disposal (e.g., regular garbage, sharps
containers, biohazard or red bags).
- Which instrument will achieve straight-line access? A.k file B. H file c.gatesglidden
- Stages of gingivitis

-snowflake" appearances in x-ray? adenomatoid odontogenic


Tumor AOT

- X-ray of mishaps in endo? Perforation

- The mm thickness of gingival margin when u do cl I


composite?

- Which type is restoration is effected by temperature changing? A. Composite.


B.amalgamc.GIR. D. Resin (I think Amalgam because of Thermal expansion) but still need to
confirm from Dr.Raghe

- Which material used under amalgam? Copalite varnish

- Save ended bur? Endo Z

- Instrument fracture at coronal part of root, what is the best management? A. Remove it. B.
Bypass and obturation. I don’t remember the rest of answers

- Ortho case: pt wit generalized moderate bone loss and will undergo of ortho tx, where will
be the Center of Resistance? A.center of root. B.apical of root. More apical

- Case periodontitis after scaling and root planing he came with ulcer, what’s the
management?

Supporativemanagement : Soft diet , analgicis , topical anesthesia

- Pt after scaling and root planning came with complaint of sensitivity, what is the
management? A.do scaling again. B. desensitization therapy and reassure the pt. C. Do
nothing

- Pt with amalgam restoration he feel pain after he eat by metallic spoon, what is the cause
of this pain? galvanic effect

- drug use to decrease saliva? Anticolenergic drug

- Pt did implant and after healing the implant mobile, what should u do?

- What is the most part of mandible affected by a fracture? Condyle

- Case: pt he received a punch in the left side of his face, what will happen? A. Fracture left
body ofmandible with right condyle. B. Fracture right body with the left condyle. C, D I don't
remember

- How to measure the BL width of ridge? ridge mapping caliper :Wanson caliper

- Pt with intrinsic discoloration what is the tx? Alumina Crown

- Few days After FPD cementatiopt came with complain of bad smell and durning
examination by dentist he see bubbles under pressure, what the reason for that?Increase
luting space
- During the access cavity the canals are classified so how can u located it?

- Papillon–Lefèvre syndrome (PLS)

- Which cancer is associated with HIV? Kaposi sarcoma

- Pie abutment

- Case with missing #11,#12,#13 ? kennedy cl III

- Pedo with immature apic with non vitaltooth , MAC was 90, what the tx? A.Obturation
using large GP, B. apexification .

- Calcium hydroxide in pulpotomy can cause? Internal resorption

- Durning instrumentation of danger zone can cause? Stripping perforation

Important points:

- new classification of perio( stage and grade)

- Classification of furcation

- miller's classification of gingival recession

- Diagnosis of Endo

- Medical compromise pt

- Trauma of pedopt

- Antibiotic Rx in perio especially aggressive


periodontitisTetracyclines,doxycycline,
Metronidazole, Amoxicillin, Ciprofloxacin, Azithromycin

‫ وهذي االسئله الي جاتني‬0-06 ‫انا اختبرت اليوم‬

‫اجى سؤال الي جوابه ال‬collar less


* ‫تعريف ال‬fremitus - automony - non-maleficent - parentsim

‫ اجاني تعريف جديد باالثيكس انو‬reasrch have to done to use it in moral values somthing like that

dentology

* About smile analysis : upper incisal edge should be parallael to?

lower lip curvature.

* cavosurface angel of

composite (more than 90)& amalgam(90) .

* The most malignant tumor in the lower lip?

Squamous cell carcinoma

* Effect of Over protected parents on the child’s behavior?

Worse the child attitude

* Multiple Qustions about indication of antibiotic

* Most common site for bemvigous volgaris?

Buccal mucosa

* Associated with rhmatois arythritis?

Condylar hyperplasia

* Most common feature in mandibular fracture?

Malocclusion - ulceration - chain edema

* Advantage of sodium hypochlorite on Chx ?


* advantage of calcium hydroxide?

Form dentin bridge

* Caoh in primary teeth will cause?

Internal resorption

* Best material for perforation?

MTA

* Pic of primry tooth and asked about what the procedure done for 4 years pt have truma on
tooth #21 berofe month?

Apixification- Apexigensis-full pulpotomy

* Pt have grade 2 furcation with grade 2 mobility?

Root resection - furcation plasty

* Tooth with Poorest perio prognosis?

Upper 1st Pm or upper 1st molar

* Difference between ANUG And herpatic gingivostomitis?

Extent of inflammation or systemic inflammatory interluckin 1

* Pt with generalized bleeding and pocket 1-3 mm and generalized bone loss 20%

do ortho or not - do scalling and and OHI then re-evaluate after 6 weeks

* No bleeding on gingiva indicate?

Healthy gingiva

* Missing 38-37-35-45-44 where to put indirect rest?

I choose distal of 33 not sure

* Missing 36-46 what lever class?

class 3

* Prognosis of furcation class 1?

Good or normal or poor

* Prognosis of vertical root fracture?

Poor

* Tx of vertical root fracture ?


Extraction

* Splinting of avulsed tooth?

2 weeks or 4 weeks

* Consequane of reimplantation of avulsed tooth?

Aper resorption - external cervical resorption - external replacement resorption - internal


resoption

* Healthy pt free of caries but have sinus track?

Dens invagenatous

* How to confirm it is perio lesion not endo?

I choose Horizontal percussion(pulp test )

* Atrophy tongue?

Iron - folic acid - B12

* Pt have pd 5 mm and recession 2 mm ,how much attachment loss?

Cal= 5+2=7

* Pt take phyntoin and have gingival overgrowth >>

change the drug from his physician

* Xerostomia >>

salagen

* Sjogren syndrome >>

high caries risk

* Pt with pleomorphic adenoma which most caries will have and what tx? Occlusal,
extractiion - cervical , extracion - occlusal, restoration - cervical-restoration???!

* Pt have crown and stagnation of food in cervical area?

OHI - smooth the crow profile – remake

* Pt with class 2 restoration and multiple crown what will cause perio problem?

Well countored crown- Remaining cement

* Band seated subgnigivaly what will cause?

Recession or overgrowth

* Which give retention& resistant in cavity ?

Pin - groove - undercut - micro-mechanical of etching

* Crohn’s disease

Crohn's disease is an inflammatory bowel disease (IBD)


* Renal end stage :

PETECHEIA ON MOUTH

* Radiopacity around the apex :

cementum deposition continuously throughout life

* Primary enamel and dentin have thinner thickness than permanent tooth

* Pt do scaling and after few days have hypersinsitivity came with ulcer and red gingiva what
to do?

I choose put desinsitizing paste and reassure the pror redo scaling or give him antibiotic

* 1st thing to check :

proximal contact

* Put and membrane in the socket and suturing it >>

GTR

* If you do implant and there is only 4 mm interocclusal space what to so?

Remove the implant and put it deeper in bone or make RPD instead of implant

1-Class 2 malocclusion with

lower 6 mm crowding:

-extract upper4 s

- extract lower 5s

- extract upper 4s and lower 5s

- extract upper5 and lower 4

2-pt complain of pain after 2 days of scaling on examination gingival ulceration related to 34
35 36

Teeth are clean ttt:

- redo the scaling

- metronidazole gel

- put desisentizing agent and reassure

3- same problem as previous but ulceration was generalized

- antibiotic in infected area

- reassurance
- re do scaling

4-pt came with deep pocket with periulent

- scaling and amoxicillin

- scaling and other drug ( may be metronidazole)

6- to measure thickness of attached gingiva which sloution ??

- seliver nitrate

- Schiller's potassium iodide solution

- etheline blue

- cant remember

7- thikness of attached gingiva

- perio probe

- explorer

- celvojio caliper ( somthing like that )

8- most abrasive

- tooth paste

- tooth gel

- tooth powder

- tooth dentifrice with fluoride

9- allergy from complete resin acrylic denture from ??

- excess monmer

-pegmentation material

- polymer of methyl methacrylate

- benzoyl peroxide

‫صراحة مو متاكده اذا نفس الخيارات اخاف اكون دخلت سؤالين في واحد لكن ان شاءالله كدا كان‬
10- granular stomatitis from denture where occure:

-hard palat

- buccal mucosa

- floor of mouth

- dorsum of tongue

11 - curate make tooth surface clean in

-2.7mm

- 3.7 mm

-4.7 mm

-5.7 mm

12- periodontitis and the bacteria show dark bacterioid and spirochete ttt?

- metronidazole

- tetracycline

13-

Patient with bleeding on probing 60% and plaque index 10%

-He knows how to brush and He’s brushing

-He know how to brush but he’s not brushing

-He doesn’t know how to brush but he’s trying

-He doesn’t know how to brush and he’s not trying

14- vesciolonodular lesion how to exam:

- somthing like swap

- exisional biopsy

- incisional biopsies

15 semilunar radiulocency on rutin examination related to upper pre molar roots

- periapical healing

- periapical lesion
- periapcal ...

- normal structure

16- O'leary plaque index >> depends on dicclosing agent

17- pt want To restore here to centrals upper with bone desorption

- Fixed partial denture

-Sectional removable denture

-Every denture

-Crown dependent removable denture ( somthing like that)

18- pt with upper fixed prothesis two months ago came with inflammation and bleeding on
probing no predental bone loss what is the couse

- violation of biological width

cement excess

19- accurate indecation of lack of Osseo integration ?

Mobility

20- during metal try in the margin was not fit solution :

- remake

-soldering with metal

- close the gap with restoration

21-Why doing Bevel With cast gold crowns

-to increase adapt ability with the tooth

22 -31 32 41 missing fixed bridge design?

-6 unit on 32 33 43

- extract 32 and 6 unit pn 33 43


23- impacted lower 8 extraction will lead to?

-Vertical bone defect

-horizontal bone defect

- furcation involvement

24- dental burs become corroded ask assistant to?

-immerse in h2o2 in closed container

- h2o2 in non closed container

2% sodium nitrate in perforated container (Protect them from corrosion)

-other solution in 2 options of container maybe( NaOH )

25-alginate was contaminated with moisture during storage what will happen to
impression?

-tear easily

-grainy ?

26-When to make steep cusps:

-condylar angle is shallow -

-when over jet is large

-overbite is deep

25- Order in mouth operation for removable denture is

-undercut prep then proximal plate then rest seat

-undercut prep then rest seat the. proximal plate

-Proximal surface, contour and rest seat

Tow other sequence of those 3

26-Composite restoration contraindicated when?

-class five cavity prep


- when isolation is compromised

1. most common site of dense invagnatius

-mx molars

-mx premolars

-mx incisors

2- how many days can storage the instruments after autoclave type B :

-21

-30

-60

3. Reason of internal resorption:

-dento alveolar abscess

-fast ortho mavement


-

4. Patient take (Zindronic acid ) for osteoporosis 5years , When give him prophylaxis
AB?

-before extraction 1 h

-before extraction 7 days

-paste for socket

-when sign infection

5. Pic of red & inflamed tongue 👅, what the test need?

-serology

- exfoliative cytology

- culture

6. Interaligamentry LA ?
-bevel toward root

-bevel opposite ..

-back pressure

7. Inferior alveolar LA for women , can’t close left eye

-vasovagal

-allergy

-needle to paroted

-optic nerve

8. Test of asthma:

-inspiratory

-expiratory

9.lower molar previous treated rct good , clinical swelling buccaly :

-Endo lesion

-perio lesion

-1ry Endo 2nd perio

-1ry perio 2nd Endo

10-most difficult site infiltration LA for tooth with irreversible pulpits?

-max molars

-mand molars

11. Fracture in the mandible cause loss sensation of lower lip?

-parasymphesial

-symphesial

-
-
Body of the mandible
12. Start Endo tx with Inferior alveolar LA , patient has pain with drilling , u gave him
2nd Inerior alveolar, patient has pain when reach pulp ?

-give 3rd Inferior A

-give intraligament
If there is intra pulpal injection it’s the correct , if not then inraligament

13. AIDS , creamy lesions orally , give :

-Nystatin

-Acyclovir
-

14.
‫اإليرقيشن قد إيه تكون بعيدة عن ال‬

apex

-2mm (not sure)

-4mm

-in pulp chamber

15. Wipping for canals by :?

16.radiolucency between upper centrals

I think (nasopalatine duct cyst)

17. 19 years old patient with severe class II malocclusion and perio disease , what
the step before prognathic surgery ?

-treat perio

-make sure for complete growth

18. 15 years old boy 👦🏻 had trauma & dentoalveolar fracture , fixation duration?

-1-2 weeks
-3-4 w

-12-14 w

19- ferric sulfate cause :

-internal resorption

20-
‫كم يبتعد الفيلم في‬cephalometry ‫عن البيشنت بالسنتيمتر ؟‬
Distance between patient and film 15cm
Distance between patient and xray source 152cm or 5 feet

27- Tooth that can be extracted with rotation :

upper incisor

28-La in periodontal ligament

- if it causes injury pain will continue

- if was given with vasoconstriction it will decrease pulp blood flow✅

29- when to close cervical perforation

- delay

- immediately before continuing ttt

- after obturation

After locating the canals and clean it then close perforation

30-Pt come with exiss wire (ortho) dentist decide he will cut it what should the patient
where during the treatment :

—eye protector✅

-rubber dam

31-Cephalo Pic and pointed on PNS want u to name it


32-Fibers that runs apically from vistebular and crystal bone to attached gingiva

-circular fiber

-periostogingival

-dentogingival

-alveolgingival✅

33-Rapid palatal expansion applience activated:

- once a day✅

- once weekly

- twice weekly

- once monthly

34- pt with pd from 3-5 mm with missing 1 tooth Need replacement, there is
generalized bleeding on probing

-SRP then reevaluate

- CAD CAm and proceed with implant

35- pic of 2 implant with stud( not parallel)

Pt complain of reduced retention

- wear of rubber bc of non parallel implant

- metal wear ( of stud)

36- to decrease external resorp. Of avulsed tooth

-NaHO

- CHX

- sodium fluoride

- ..... mix

37- what’s used for internal bleaching?

- CaHO
- sodium perborate

38- 7 years pt came complaining of pain in molar area

On examination it showed ( something abnormal i cant remember wxactly) affecting


permenant incisor and molar ‫اتوقع شي كانها كانت صغيره اوشي‬

MIH

- dentogenesis imperfecta

- amelogensis imperfecta

- dentin dysplasia

39- what is the purpose of removing smear layer in endo

- to facilitate removal of sealer for reendo

- to facilitate sealer penetration of sealer to dentinal tubule

-to inhance antimocrobial effect of sealer

- to inhance adhesion of sealer to dentin, also better adaptation of filling material to the
canal wall

40- chloroform function : dissolve GP

41: Highest perforation during post perp in

- mesobuccal root of maxillary molars

- mesial rootsin mandibular molars

42-: minimum biofilm in clinic water system

-<500 ( im sure) bc it was written in onther question

43- ‫بالنسبة للموية فيالعيادة المفروض اشغلها اكترمكان فين‬

- stagnation area

- outward going

-inward going
44- whats the indication of mandible fracture

- change in occlusion

- laceration

- luxation of mandiblar teeth

45- when to suspect scleroderma in young pt

- truisms ✅

- bilateral analysis of tmj

- xerostomia more then 6 months

46- recission in one tooth in shape of line picture

- improper use of dental floss

47- pt come to ortho clinic

In the canine there gingiva with thin biotybe and thin buccal bone

When tto coseder augmentation

- when to move labially✅

- when to move lingualy

48- pt with peridontal disease with thin biotype

-non surgical STP ✅

- augmentation first

- poilshing

49- purpose of facebow

1- Cross-section of H file? Round ✅

2- Irrigation tip should be away from apex by?? 2mm or less /4mm
3- perio-endo lesions ‫سؤالين بسيطه‬

4- picture of perio chip and asking about what is the active ingredient in it? Chlorhexidine ✅

5- ortho asking about molar relationship?

6- class 3 extract lower 4

7- class 2 extract upper 4

8- epoxy resin sealer? AH-26 and AH-plus✅

9-patient with severe periodontitis after scaling in the recall visits you found residual pockets
2-3mm this indicates?

Patient have high recurrence or low recurrence or normal finding

10- the most striking feature in pregnant women is? Gingival bleeding✅

11- angle of the shank of the perio probe with the tooth?

Perpendicular

or make 45 degrees

or 75 degree

or parallel

12- during metal try in there was good marginal fit of the crown then during insertion
appointment the crown can't be seated properly?

excessive trim of wax

or cast distortion during porcelain application

13- occlusal trauma leads to what in periapical radiograph?

Funneling of PDL crest ✅

14- connecting implant to natural tooth what is the complication?

Implant mobility

or no osseointegration- maybe

or cement failure

15- you did bleaching, when you should place composite restoration to avoid weakness of
bonding strengh?

1 week after bleaching✅


Search of founded mean the you can act and make moral decisions on it? consequentialism
or principlitism‫واختيارين ناسيها‬

17- clamp for partially erupted molars?

A14✅

18- patient with desquamative gingivitis what you will do?

Take biopsy for histopathology and immunoflurecence✅

19- sealer best placed by ?

Lentulo spiral✅

‫ كيس سيناريوطويل وبعدين يقول‬-21

You used tooth sloth and the patient had severe pain on releasing?

Crack tooth ✅

1-What determines the depth of periodontal probe penetration in the tissue?

A-sizs and shape of perio probe

B-lenght if the shank

C-wight of the probe

D-The grade of stainless steel used to manefactureing the probe


2-reason for rinsing the mouth with anteseptic MW beforperio surgery?

A-to remove food debris

B-to prevent postop infection

C-to reduce the amount of microorganism in the aerosol.

3-pt has multiple missing teeth and generlized attrition, dentist need detailed analysis of the
occlusion which articulator is the best to use in this case?

A-Hanue articulator

B-fully adjustable articulator

C-name then mean value ..

D-whip max ... semiadjustable articulator

4- which of the following is the most difficult to be done with convintional CD?

A-incisoin or tearing food on canine area

B-chewing extremely hard food

C-soft food...

D-creamy food...

5-what is the cytokines responsible for tissue destruction in early gingivitis?

A-IL-1----- >maybe

B-TNF

C-PGE2

D-i forgot the choice

6-chemomechanical loss of teeth is

A-attrition

B-abrasion
C-erosion

D-decalcification- maybe

7-the most natural and effective way to increase tooth resistance to caries is?

A-drinking floridated water during tooth formation

B-topical floride after tooth eruption

C-vitamin D and Ca supplements

D- I forgot the choic

8-extracted tooh where to through?

A-contaminated waste

B-dental waste

C-infectious waste

D-regular waste

9- disadvantage of gutta percha

A-difficult to remove

B-shrinkage with heat and solvent

C-difficult to adapt with compaction

D-

10-type if headache that is relieved in 10min after 100% oxygen administration

A-cough headache

B-anxious

C-cluster

D-migraine

11-after placement of rubber dam pt started suffering from sensitivity ( something like that)
which type of sensitivity is this?

1-2-3-4
12-x-ray pic of immature tooth with open apex and asking about the treatment that has
been done? Apexification

13-dental assistant vaccinated( she took all 3 doses )for HBV, git needle stick injury from pt
+ve for HBVsAg, what to do?

A-immediately give Ig

B-measure the antibody titr for antiHBVs

C-only follow up

D-

14-pt complaining that since he recived the fixed crown in #11 the gingiva became tender
and swollen, examination showed BOP around this tooth only, there is no plaque
accumulation, no clinical attachment loss, what is the possible cause?

A-luting cement dissole

B-encroachment on the supracrestal fiber

C-caries in the tooth below crown

D- i forgot the choic

15-which type of occlusion allow freedom in protrusive movement?


A-long centric
B-neutro occlusion
C- ‫نسيت‬
D-‫نسيت‬

16- one tooth smaller than usual, with vertical notch along the labial surface of
the tooth, radiographic examination showed one small root, what is the
diagnosis?
A-fusion
B-gemination
C-peg shape ****
D-microdontia

17-cross section of H-file?


A-round***
B-trinagle
C-trapezoid
D-?

18-3 questions about tooth wear

19-RPD with intracoronal attachment, where to put the (matrix part=keyway)


A-near to indirect retainer
B-within the conture of crown of abutment tooth*****
C-
D-

19-how to increase retention of crown?


A-increase tapering of prep
B-decrease hight of preparation wall
C-use zinc phosphate cement
D-increas roughnes of the intaglio surface of crown*****

20- what is the minimum thickness of dentin wall to withstand core build up?
A-0.5mm
B-1mm*****
C-2mm
D-2.5mm

21-what is the ideal hight from alveolar ridge to the opposing occlusal surface
to receive implant supported fixed crown?
A-5
B-6
C-7
D-8*****. 8-12mm

Thickest mucosa
The highest values were found in the region of the buccal mucosa (294 μm)
and the hard palate (239 μm), whereas the thinnest epithelium was measured
at the floor of the mouth (99 μm).
Maxillary molar buccal
Mandibular molar buccal
Maxillary incisors labial
Mandibular incisor labial
‫وجا كمان العكس‬

‫ جا كمان ايش فايدة‬widge


preventing gingival overhang

1-What determines the depth of periodontal probe penetration in the tissue?

A-sizs and shape of perio probe

B-lenght if the shank

C-wight of the probe

D-The grade of stainless steel used to manefactureing the probe

2-reason for rinsing the mouth with anteseptic MW befor perio surgery?

A-to remove food debris

B-to prevent postop infection

C-to reduce the amount of microorganism in the aerosol.

3-pt has multiple missing teeth and generlized attrition, dentist need detailed analysis of the
occlusion which articulator is the best to use in this case?

A-Hanue articulator

B-fully adjustable articulator

C-name then mean value ..

D-name... semiadjustable articulator

4- which of the following is the most difficult to be done with convintional CD?

A-incisoin or tearing food on canine area

B-chewing extremely hard food

C-soft food...

D-creamy food...

5-what is the cytokines responsible for tissue destruction in early gingivitis?


A-IL-1

B-TNF

C-PGE2

D-i forgot the choice

6-chemomechanical loss of teeth is

A-attrition

B-abrasion

C-erosion

D-decalcification

7-the most natural and effective way to increase tooth resistance to caries is?

A-drinking floridated water during tooth formation

B-topical floride after tooth eruption

C-vitamin D and Ca supplements

D- I forgot the choic

8-extracted tooh where to through?

A-contaminated waste

B-dental waste

C-infectious waste

D-regular waste

9- disadvantage of gutta percha

A-difficult to remove

B-shrinkage with heat and solvent

C-difficult to adapt with compaction

D-

10-type if headache that is relieved in 10min after 100% oxygen administration

A-cough headache
B-anxious

C-cluster

D-migraine

11-after placement of rubber dam pt started suffering from sensitivity ( something like that)
which type of sensitivity is this?

1-2-3-4

12-x-ray pic of immature tooth with open apex and asking about the treatment that has
been done? Apexification

13-dental assistant vaccinated( she took all 3 doses )for HBV, git needle stick injury from pt
+ve for HBVsAg, what to do?

A-immediately give Ig

B-measure the antibody titr for antiHBVs

C-only follow up

D-

14-pt complaining that since he recived the fixed crown in #11 the gingiva became tender
and swollen, examination showed BOP around this tooth only, there is no plaque
accumulation, no clinical attachment loss, what is the possible cause?

A-luting cement dissole

B-encroachment on the supracrestal fiber

C-caries in the tooth below crown

D- i forgot the choic

Important Notes:

Bleaching

Non vital tooth:


0. Walking bleaching: 10% sodium perborate , can cause external cervical
resorption , then place RMGI liner on Gp, chang weekly , calcium hydroxide +saline as temp
resto

0. In office bleaching: 30-35% hydrogen peroxide paste or gel

Vital bleaching:

0. In office bleaching: 30-35% of hydrogen peroxide gel or past

0. Home or Rx dentist: 10-15% carbamide peroxide

Asymptomatic irreversible pulpitis:

1-deep restoration

2- internal resorption

3-pulp polyp

hand wash:

by soap 40–60 seconds.

Alcohol 20-30 seconds

Cleft lip: failure of maxillary process on one or both sides to meet and fuse with the medial
nasal process week 7

Cleft palates:

Lack of growth or failure of fusion or interruption after initial fusion between the lateral and
medial palatine processes and the nasal septum


normal range

CD4 500-1500

Platelets 150,000 -450,000

WBC 4,000 and 11,000


Asthmatic patient save to use

Acetaminophen as analgesics

And for anxiety use diazepam

Thrombocytopenia :

CBC

Surgical procedure can be performed if it is 50,000 or higher

Hemophilia + von willebrand disease

PT and APTT

Avoid use NSAID and aspirin

Warfarin

INR

If it is 3,5 or less we can perform extraction and dental hygiene .

Non invasive procedure can be performed without any precautions

Sickle cell anemia

Treat patient normally but the aim is to prevent stress and infection

Stress reduction protocol

1-short morning appointment

2-discuses any question with patients

3-clear explication of procedure and expected feeling

4-good pain control

5-promedicate with benzodiazepine if needed

HIV

CBC within 5-7 days prior to surgical procedure

Treat if WBC more than 2000 , platelets more than 60,000 and CD4 more than 200
-if CD4 less than 200 prophylactic antibiotic is indicated prior to invasive procedure

For chemotherapy extraction should be done at least 2 weeks before IV chemotherapy

If there is any emergency and You want to do surgery or scaling you should request CBC
within 24 H if platelets more than 50,000 you will perform necessary dental procedure also
we have to check absolute neutrophils count less than 900 we have to postpone procedure
or consider prophylactic if it is urgent treatment

Radiotherapy extraction at least 2-3 weeks before radiotherapy follow up every 3-6 months

Tetracyclines used in treating refractory periodontitis, including localized aggressive


periodontitis (LAP) Tetracyclines have the ability to concentrate in the periodontal tissues
and inhibit the growth of Aggregatibacter actinomycetemcomitans. In addition, tetracyclines
exert an anticollagenase effect that can inhibit tissue destruction and may aid bone
regeneration.

Metronidazole has been used successfully to treat NUG necrotising ulcerative gingivitis

Metronidazole is not the drug of choice for treating A. actinomycetemcomitans infections.

Amoxicillin may be useful in the management of patients with aggressive periodontitis, in


both localized and generalized forms

metronidazole and amoxicillin to be clinically effective in treating LAP, although 50% of


patients harbored A. actinomycetemcomitans 1 year later.

Metronidazole-ciprofloxacin combination is effective against A. actinomycetemcomitans;


metronidazole targets obligate anaerobes, and ciprofloxacin targets facultative anaerobes.

Orthodontic

CL III Treatment

Early orthopedic treatment:

1- Protraction Face-Mask:
◦ Most successful if carried prior to the pubertal growth spurt.

◦ Depend on patient cooperation (400 g per side, 14 hours per day)

2- Bone anchored maxillary protraction (BAMP).

◦ Screws or mini-plates in the posterior maxilla and anterior mandible for Class III elastics.

◦ Evidence show a greater degree of maxillary advancement is achieved compared to face-


mask therapy

alone.

◦ A combination of techniques

3- Chin-cup: rotating the mandible downwards and backwards with a reduction of overbite
(not

used).

Tissue-borne expander : Haas Appliance.

Tooth-borne expander : Hyrax or Biedermann Appliance

Tissue -borne functional appliance: frankle appliance

Class II

0. Cervical headgear : cause extrusion of upper first molar which result open
bite

0. High pull headgear : not do extrude upper molar

Class III:

0. Reverse pull headgear ( face mask) : downward backword rotation of


mandible and lingual tipping of incisal lower

0. Chin cup

Dental infection:

Hepatitis A virus:

Anti-HAV : detectable at onset of symptoms

IgM anti-HAV: recent infection positive up to 4-6 months

Hepatitis B virus:
HBsAG ( surface antigen):

+ve infectious

-ve noninfectious

HBeAg ( antigen) : hepatitis B virus that circulates in infected blood when the virus is actively
replicating.

HBcAg ( core ): commercial not available

Total anti HBc:

Indicate acute and chronic

Anti HBs: past infection with immunity

Anti HBe:

reactive (positive): indicates the onset of recovery in an individual with hepatitis B.

non-reactive (negative) the infection is very recent and viral replication has not yet peaked.

Biologic width is the distance established by "the junctional epithelium and connective tissue
attachment to the root surface" of a tooth. In other words, it is the height between the
deepest point of the gingival sulcus and the alveolar bone crest.

Pocket depth PD: distance between the base of pocket and the gingival margin

Level of attachment: the distance between the base of pocket and CEJ ( fixed point)

CAL ( rescession + ) = PD + from CEJ

Four Different cell types that can repopulate the root surface after periodontal surgery


Melcher described the four types as follow:

Long junctional epithelium : if these cells populate first there will be no bone regeneration
Gingival connective tissue cell: May lead to root resorption

Bone cell: Resorption and ankylosis

periodontal ligament : New cementum forms

after scaling and root planning = Long junctional epithelium take 1-2 weeks

Healing after Surgical Gingivectomy : After 5 to 14 days: surface epithelialization is generally


complete.

• During the first 4 weeks after gingivectomy, keratinization is less than it was
before surgery.

• Complete epithelial repair takes about 1 month.

• Complete repair of the connective tissue takes about 7 weeks.

After apical surgery when we place crown? After one month

After perio surgery when we plcae crown? After 21 days (3w)

(...General note :

⁃ pilocarpine drug ( salagen) cevimeline / xerostomia


( sjogren syndrome) to increase saliva

⁃ Anticolenergic drug “ Atropine”( to decrease


saliva

⁃ Protamine sulfate ( to reverse heparin)

⁃ Listerine long term complication( taste


disturbance)

⁃ Acetaminophen (pain killer for disseminated


intravascular coagulation)
⁃ Tranexamic acid for hemophilic pt ( IM before
surgery) Antifibrinolytic

⁃ 2g amoxicillin before surgery with 1 hr (


infective endocarditis)

⁃ Acetaminophen( DIC pain killer)

⁃ Ethosuxmide( petit mall epilepsy)

⁃ Metronidazole( contraindication for epileptic


pt)

⁃ Opoied “codien”( fetus deformation in


pregnancy)

⁃ Benzodiazepine( anxious asthma pt)

⁃ Zometa “ bisphosphonates” (to treat high blood


calcium levels (hypercalcemia) that may occur with
cancer. Zoledronic acid is also used with cancer
chemotherapy to treat bone problems that may occur
with multiple myeloma and paget’s disease

⁃ Transdermal nitroglycerin( angina)

⁃ Carbomizapine (600-1200 mg) trigaminal


neuralgia

⁃ Dylantin(phynotoin) don not give with


metronidazole
⁃ Slfonamide ( aplastic anemia)

⁃ Benzodiazpines( pre-anesthetic , anti


anxiolytic

LA Maximum Dosage
Step 1: Maximum recommended dose Calculation
Maximum allowable dose x Weight in kg = Result A
Step2: Amount of LA agent in the Carpule Calculation
(concentration% x 10) x 1.8 (Carpule) = Result B
Number Calculation Step 3 : Maximum Carpule
Result A/Result B = Maximum Carpules Number

Maximum Recommended Dosages of Local Anesthetics:


Local Anesthetic Maximum allowable dose in mg/kg
Lidocaine with Epinephrine 7.0
Lidocaine Plain 4.4
Mepivacaine with Epinephrine 6.6
Mepivacaine Plain 6.6
Prilocaine with Epinephrine 8.0
Prilocaine Plain 8.0
Bupivacaine with Epinephrine 2.0
Bupivacaine Plain
Articaine with Epinephrine 7.0
Articaine Plain

Exercise 1: Patient: 22 Years Old, Healthy, Female, 50 kg


Lidocaine 2% with Epinephrine:
Step 1:
7 x 50 = 350 mg Maximum recommended dose.
Step 2:
(2% x 10) x 1.8 =
20 x 1.8 = 36 mg Amount of Lidocaine 2% in the Carpule
Step 3:
350/36 = 9.7 Maximum Carpules Allowed

Exercise 2: Patient: 40 Years Old, Healthy, Male, 90 kg


Articaine 4% with Epinephrine:
Step1:
7 x 90 = 630 mg Maximum recommended dose.
Step 2:
(4% x 10) x 1.8 =
40 x 1.8 = 72 mg Amount of Articaine 4% in the Carpule
Step 3:
630/72 = 8.75 Maximum Carpules Allowed

Exercise 3: Patient: 6 Years Old, Healthy, Male, 20 kg


Mepivacaine 3% with Epinephrine:
Step 1:
6.6 x 20 = 132 mg Maximum recommended dose.
Step 2:
(3% x 10) x 1.8 =
30 x 1.8 = 54 mg Amount of Mepivacaine 3% in the Carpule
Step 3:
132/54 = 2.4 Maximum Carpules Allowed

MOST COMMON :
1. Most common impacted anterior tooth--- maxillary canine
2. Most common supernumerary tooth—mesiodens
3. Most common tooth involved in garres osteomyelitis--mandibular 1st molar
4. Most common malignancy of oral cavity—squamous cell carcinoma
5. Most common benign tumour of oral cavity—fibroma
6. Most common retained tooth – primary mandibular second molar
7. Most common recurring cyst— odontogenic keratocyst
8. Most common cyst in oral cavity— periapical cyst
9. Most common lichen planus- reticular lichen planus.
10. Most common dermatosis to affect oral cavity- lichen planus
11. Most common chemical burn in oral cavity –aspirin burn
12. Most common topical fluoride in adults – stannous fluoride
13. Most common topical fluoride in children—1.23 APF gel.
14. Most common burshing technique-scrub technique
15. Most common developments cyst-Dentigerous cyst
16. Most common complication of GA (op)-nausea
17. Most common used drug for petitmal epilepsy-ethosu ximide
18. Most common used drug for grand mal-phenytoin
19. Most common drug used for temporal epilepsy- carbomezepine
20. Most common treatment for cyst – enucleation
21. Most common used clasp-simple circlet clasp
22. Most common used face bow in fpd- kinematic
23. Most common complication of RA involves TMJ-fibrous ankylosis
24. Most common salivary malignancy in children – mucoepidermoid
carcinoma. 25. Most common salivary malignancy in palate area-ACC
26. Most common type of haemophilia--- haemophilia A
27. Most common type of gingivitis in children--- eruption gingivitis
28. Most common type of cerebral palsy is –athetoid/ spastic.
29. Most common nerve involved in C sinus thrombosis – abducent nerve
30. Most common type of impaction ---mesioangular
31. Most common benign epithelial tumour---- papilloma
32. Most common complication of surgical extraction of lower third molar—
loss of blood clot
33. Most common used instrument grasp—pen grasp
34. Most common susceptible tooth for caries—mandibul ar first molar
35. Most common contrast media - iodine in oil
36. Most common cause of light radiographs — exhausted developer
37. Most common cause of failure of RCT— incomplete obturation
38. Most common isolated yeast strain from RCT— Candida
39. Most common bacteria found in root canals --- gram positive
40. Most common part of oral cavity affected by L planus –buccal mucosa.

You might also like